ENARM COMPENDIO

533
COMPENDIO Dr. Alejandro Sandoval G. Travesi CMN La Raza ENARM

description

Tienes que descargar la expo, es dinamica !!! ... Los tips generales mas importantes previos al examen, les deseo exito

Transcript of ENARM COMPENDIO

Page 1: ENARM  COMPENDIO

COMPENDIO

Dr. Alejandro Sandoval G. TravesiCMN La Raza

ENARM

Page 2: ENARM  COMPENDIO

Consejos

1. Leer diariamente y en bloques

2. Adiós Partys un tiempo

3. Has ejercicio y come bien durante el estudio

4. Toma algún curso bueno si tienes la posibilidad

5. Ten Fe.

Page 3: ENARM  COMPENDIO

HEMATOLOGIA

Page 4: ENARM  COMPENDIO

Hemoglobina Hematocrito

Recién nacido19.5 ± 5.0 54 ± 10

Mujeres14.0 ± 2.0 42 ± 5

Hombres 16.0 ± 2.0 47 ± 7

Cifras normales de hemoglobina y hematocrito al nivel del mar (Wintrobe)

Page 5: ENARM  COMPENDIO

• BANDAS• 1-3%

Page 6: ENARM  COMPENDIO
Page 7: ENARM  COMPENDIO

QUIMICA Y ELECTROLITOS• GLUCOSA• 70- 105

• UREA• 10- 50

• ACIDO URICO• 2.5- 7.7

• NITROGENO UREICO• 6- 20

• CREATININA SERICA• 0.5- 1.2

• COLESTEROL• 120- 200

• TRIGLICERIDOS• 10- 160

• HDL• 40- 90

• LDL• 0 – 120

• VLDL• 0- 50

• ALBUMINA• 3.5- 5.5

• SODIO• 135- 145

• POTASIO• 4- 5.3

• CLORO• 98- 106

Page 8: ENARM  COMPENDIO

PFH Y ENZIMAS CARDIACAS• BILIRRUBINA TOTAL

• 0 – 1.5

• BILIRRUBINA INDIRECTA• 0 – 1.50

• BILIRRUBINA DIRECTA• 0 – 0.5

• AST/TGO• 6 – 38

• ALT/TGP• 0 – 35

• DHL• 100 – 190

• GGT• 7 – 50

• CPK• 0 – 226

Page 9: ENARM  COMPENDIO

GASES ARTERIALES

• PH• 7.35--7.45

• PCO2 • 33-45mmHg

• PO2

• 75-105mmHg

Page 10: ENARM  COMPENDIO

OSMOLARIDAD SERICA Y LCR• OSMOLALITY SERUM• 275-295mOsmol/Kg

• LCR• CELLCOUNT• 0.5cells/mm3

• CHLORIDE• 118--132mEq/L

• GAMMA GLOBULIN• 3--12% total Proteins

• GLUCOSE• 40--70mg/dL• PRESSURE

• 70--180mm h2O• PROTEINS,TOTAL

• < 40mg/dL

Page 11: ENARM  COMPENDIO

ANEMIA POR DEFICIT DE HIERRO

Page 12: ENARM  COMPENDIO

ANEMIA POR DEFICIT DE HIERRO

Page 13: ENARM  COMPENDIO
Page 14: ENARM  COMPENDIO

ANEMIA MEGALOBLÁSTICA# de Eritrocitos Otros

estudios

Aspirado de médula ósea:

HIPERCELULAR

bilirrubina

indirecta = lig.

DHL =

Hemoglobina

Hematocrito

V.G.M.

H.G. M.NC.M.H.G.

Reticulocitos N o lig.

Leucocitos

Diferencial L=

Frotis N=

Plaquetas

Page 15: ENARM  COMPENDIO

VITAMINA B12

Page 16: ENARM  COMPENDIO

A que se refiere el termino POLICITEMIA ESPURIA?R = AUMENTO DEL HTO por aumento de la concentración del VOLUMEN PLASMÁTICO NO POR EL > DE LA

MASA ERITROCITARIA Cual es el cc de la TROMBOSIS ESENCIAL?R = El primer signo puede ser trombosis mesentérica, hepática o portal. Eritromelalgia. Cuales son los datos de laboratorio de la trombosis esencial?1) PLAQUETAS >2, 000,0002) FSP CON > DE PLAQUETAS GRANDES3) MO CON > DE MEGACARIOCITOS4) GEN FILADELFIA AUSENTE que descarta leucemia mieloide crónica Que otras enfermedades pueden causar > plaquetario sin ser TE?R = AR, CUCI e infección crónica Cual es el manejo de la TE?R = HIDROXIUREA y ASA hasta reducir a < 500, 000 Que es la MIELOFIBROSIS?5) Trastorno mieloproliferativo caracterizado por FIBROSIS DE LA MO, esplenomegalia y cuadro leuco

eritroblastico en FSP con POIQUILOCITOS EN LAGRIMA.6) HEMATOPOYESIS EXTRAMEDULAR

Page 17: ENARM  COMPENDIO

Cual es la etiología de la mielofibrosis? R = Se desarrolla en respuesta a un > EN LA SECRECIÓN DE FACTOR DE CRECIMIENTO DERIVADO DE LAS PLAQUETAS Cual es el cc de la mielofibrosis?

1) Fatiga, distensión abdominal por esplenomegalia, DOLOR OSEO PRINCIPALMENTE RETROESTERNAL, hepatomegalia.

2) Hemorragias por secuestro de plaquetas

3) HIPERTENSIÓN PORTAL Y VARICES ESOFÁGICAS POR HEMATOPOYESIS HEPÁTICA

Cuales son los datos de laboratorio y punción obtenidos en la mielofibrosis?4) Anemia

5) FSP con POIQUILOCITOS EN LAGRIMA, formas inmaduras mieloides y PLAQUETAS GIGANTES desgranuladas

6) PUNCIÓN SECA por aumento de fibras reticulares

Cual es la triada clásica de la mielofibrosis?7) POIQUILOCITOS EN LAGRIMA8) SANGRE LEUCO-ERITROBLASTICA9) PLAQUETAS GIGANTES10) Supervivencia de 3-5ª

Cual es el manejo de la mielofibrosis?R = Transfusiones continuas, TETOSTERONA. TMO + talidomida Cuando se considera LEUCEMIA según la OMS?R = Cuando hay al menos 20% DE BLASTOS EN SANGRE O MO.

Page 18: ENARM  COMPENDIO

LEUCEMIA AGUDA LINFOBLASTICA

Page 19: ENARM  COMPENDIO

LEUCEMIA AGUDA LINFOBLASTICA

Page 20: ENARM  COMPENDIO

Cuales son las bases para el dx de LLC?

R = Es una malignidad ORIGINADA EN LOS LINFOCITOS B, en la cual cursan asintomáticos, CON INFILTRACIÓN DE LB EN LOS ÓRGANOS

Cual es el cc de LLC?R = Fatiga, LINFADENOPATIA Y HEPATO-ESPLENOMEGALIA Cual es el hallazgo de laboratorio de la LLC?

1) LINFOCITOSIS AISLADA > 5000

2) MARCADOR CD 19 EN LB Y CD 5 EN LT Cual es el manejo de la LLC?R = Clorambucil Cual es la estirpe histológica de la que proviene la LEUCEMIA DE CÉLULAS VELLOSAS?R = LINFOCITOS B Cual es el cc de LCV?R = Fatiga, ESPLENOMEGALIA o asintomático

Page 21: ENARM  COMPENDIO

PUNCION SECA

I. MIELOFIBROSIS• Cual es la etiología de la mielofibrosis? R = Se desarrolla en respuesta a un > EN LA SECRECIÓN DE FACTOR DE CRECIMIENTO DERIVADO DE LAS

PLAQUETAS

• Cual es el cc de la mielofibrosis?1) Fatiga, distensión abdominal por esplenomegalia, DOLOR OSEO PRINCIPALMENTE RETROESTERNAL,

hepatomegalia.2) Hemorragias por secuestro de plaquetas3) HIPERTENSIÓN PORTAL Y VARICES ESOFÁGICAS POR HEMATOPOYESIS HEPÁTICA

II. LEUCEMIA DE CELULAS VELLOSAS• Cual es la estirpe histológica de la que proviene la leucemia de células vellosas?R = LINFOCITOS B

• Cual es el cc de LCV?R = Fatiga, ESPLENOMEGALIA o asintomático

Page 22: ENARM  COMPENDIO

Que es el MIELOMA MÚLTIPLE?

1) Es una neoplasia de células plasmáticas que sustituye la MO.

2) Hay DESTRUCCIÓN ÓSEA con formación de PARAPROTEÍNAS plasmáticas IgG o IgA

3) Hay HIPERVISCOCIDAD debido a paraproteínas

4) Puede provocar IRA por las paraproteínas

 

Cual es el cc del mieloma múltiple?

5) DOLOR ÓSEO 70% que aumenta con los movimientos, principalmente en la espalda CON EVIDENCIAS DE FRACTURAS VERTEBRALES

6) Edad > 65ª , ANEMIA, dolor óseo, infección

7) Amiloidosis manifiesta: macroglosia, neutropatia, ICC o hepatomegalia

Cuales son los datos de laboratorio del MM?

8) HIPERCALCEMIA, Sx anémico con ANEMIA NORMO-NORMO por efecto supresor de eritropoyetina por IL6

9) NEFRITIS intersticial con pérdida DE CADENAS LIGERAS

10) Biopsia de MO con infiltración normal o difusa

 

Cual es el manejo de mieloma múltiple?

11) Vincristina, doxorrubicina y dexametazona

12) TMO autologo

13) TALIDOMIDA + DEXAMETAZONA PARA RECAÍDAS

14) Dx dif. Con plasmocitoma solitario que responde bien a la radioterapia que no presenta la clínica típica del mieloma

 

Como estadificas la carga tumoral en el MM?

15) BAJA: con espiga IgG <5 gr/dl sin hipercalcemia ni IR

16) ALTA: con espiga IgG > 7 gr, Hto <25% y calcio > 12

 

Que es la MACROGLOBULINEMIA DE WALDESTROM?

R = Enfermedad MALIGNA DE LB, con hibrido de LB célula plasmática con PARAPROTEINA IgM

Page 23: ENARM  COMPENDIO

Cuales son las causas secundarias más comunes de PTI?

1) Fármacos como la heparina

2) LES y LLC

 

Cual es el mecanismo por el cual la heparina causa trombocitopenia?

R = Auto anticuerpos IgG vs factor 4 plaquetario

 

Cual es el manejo de la PTI?

3) PREDNISONA 1-2 mg/kg/dia con respuesta en una semana

4) INMUNOGLOBULINA iv para aumento de plaquetas

5) ESPLENECTOMÍA

 

Que es la PURPURA TROMBOCITOPENICA TROMBOTICA?

R = Es una enfermedad caracterizada por ANEMIA HEMOLÍTICA MICROANGIOHEPATICA, TROMBOCITPENIA y AUMENTO NOTABLE DE DHL SÉRICA (a diferencia de la PTI donde no hay elevación de la DHL)

 

Cual es la etiología de la PTT?

R = Deficiencia de “PROTEASA” que divide al factor de VON WILLEBRAND (VIII), por lo tanto se acumula dicho factor originando aglutinación de plaquetas en endotelio vascular. TAMBIÉN LO PUEDE DESENCADENAR EL EMBARAZO, LOS ESTRÓGENOS ELEVADOS Y LAS INFECCIONES.

 

Cual es el cc de la PTT?

6) PENTADA CLASICA: Anemia hemolítica microangiopatica, Trombocitopenia, Afección neurológica, Falla Renal y Fiebre. ATARF

7) Fiebre, palidez, PETEQUIAS

8) Anemia, hemorragia.

9) Síntomas neurales como HEMIPARESIA QUE DURA MINUTOS

Page 24: ENARM  COMPENDIO

DEFICIT DE ADHESION PLAQUETARIO

I. Que es la tromboastenia de GLANZMAN?A. Es un trastorno AR que produce hemorragias ( trastorno

plaquetario). LAS PLAQUETAS NO PUEDEN AGREGARSE por que no hay RECEPTORES (GLUCOPROTEINAS IIB Y IIIA) para el fibrinógeno.

B. Se maneja con desmopresina

I. Que es el síndrome de BERNARD-SOULTIER?A. Es un trastorno AR que OCASIONA HEMORRAGIAS. Las PLAQUETAS

NO SE ADHIEREN por FALTA DEL RECEPTOR IIB para el factor de vWFB. Se maneja con transfusion de plaquetas

Page 25: ENARM  COMPENDIO

A que se debe la HEMOFILIA TIPO C y cual es su principal cc?

R = Déficit del FACTOR XI y se manifiesta con HEMORRAGIAS LEVES. Que es la HEMOFILIA TIPO A?

R = Es un trastorno hereditario AR LIGADO AL X con déficit del FACTOR VIII y SOLO

AFECTA A VARONES Cual es el cc de la hemofilia tipo A?R = HEMORRAGIA EN ARTICULACIONES de rodilla, tobillo, codo, músculos y aparato GI Cuales son los datos de lab en la hemofilia tipo A?

1) TPT SE PROLONGA únicamente2) FACTOR VIII DISMINUIDO Y VWF NORMAL Cual es el manejo de la hemofilia tipo A?R = Concentrados de FACTOR VIII 4000 UI para 70 kg Como puede transmitirse la enfermedad de anticuerpos vs el factor VIII?R = Puede desarrollarse POST PARTO o sin una causa, hemofilia A que han recibido concentrados

plasmáticos.

Page 26: ENARM  COMPENDIO

Cual es el cc de la enfermedad de anticuerpos vs el factor VIII?R = Hemorragia grave Cual es el manejo de elección en la enfermedad de anticuerpos vs el factor VIII?R = CICLOFOSFAMIDA Que es la HEMOFILIA TIPO B o enfermedad de Christmas?

R = Trastorno AR LIGADO AL X con déficit de FACTOR IX en el cual se eleva el TPT Cual es el manejo de la hemofilia tipo B o enfermedad de Christmas?R = Concentrados de FACTOR IX 6000 para 70 kg Que factores de coagulación se ven afectados en COAGULOPATIA POR ENFERMEDAD

HEPÁTICA?1) Se fabrican todos los factores de coagulación en el hígado EXCEPTO EL FACTOR VIII2) Primero se ven afectados los vitamina K dependientes II, VII, IX y X y el factor V3) Aumenta la fibrinólisis por la disminución de plasmina

Cual es el manejo de la coagulopatia por enfermedad hepática?R = PLASMA FRESCO CONGELADO

Page 27: ENARM  COMPENDIO

Cual es el cc de la coagulación intravascular diseminada?1) HEMORRAGIAS Y TROMBOSIS2) Hemorragia +++ por punción, heridas o espontanea3) Trombosis + como isquemia digital o gangrena Que es el SÍNDROME DE TROSEAU en la coagulación intravascular diseminada?R = Coagulación intravascular diseminada manifiesta como TROMBOSIS VENOSA SUPERFICIAL Y PROFUNDA POR CÁNCER

principalmente de manera recurrente. Cuales son los datos de lab para la coagulación intravascular diseminada?4) HIPOFIBRINOLEMIA 5) Aumento de productos de degradación de la fibrina, TROMBOCITOPENIA y AUMENTO DEL TP Y TPT6) DÍMERO D como producto de degradación de la fibrina que aumenta si hay fallo hepático. Cual es el manejo para la coagulación intravascular diseminada?7) TRATAR EL TRASTORNO SUBYACENTE8) Heparina, S- AMINOCAPROICO (AUMENTA EL FIBRINÓGENO)

9) El ÉXITO del tx consiste en que AUMENTE EL FIBRINÓGENO10) CRIOPRECIPITADOS PARA FIBRINÓGENO Como se diagnostica la disfibrinolemia?

R = Aumento del TIEMPO DE REPTILASA Que medicamento se utiliza de elección en los estados de hipercoagubilidad?R = WARFARINA

Page 28: ENARM  COMPENDIO

PATOLOGIA CELULA PATOGNOMONICA

ANEMIA DE CELULAS FALCIFORMES Cuerpos de Howell-Jolly:Estructuras únicas o dobles, pequeñas y redondas. Se observan como gránulos densos y de color azul rojizo o violeta. Localizados excéntricamente

DEFICIT DE 6 GPDH Cuerpos de Heinz: Citoplasma de los glóbulos rojos , aparecen como pequeños puntos oscuros bajo el microscopio

LINFOMA DE HODKING Reed Stemberg cells: Celula B modificada por lo general es muy grande dando la pariencia de ojos de Buho con nucléolos prominentes en forma de inclusiones, la variedad mononuclear tiene un solo nucleo y un nucléolo prominente.

LEUCEMIA AGUDA Bastones de Auer: Son estructuras filiformes dentro de los blastos

ANEMIA SIDEROBLASTICA Cuerpos de Papenheimer: Son acúmulos de hemosiderina unida a proteínas. Consisten en gránulos basófilos, con las tinciones habituales, que además, se tiñen también de azul con el colorante de Perls (azul de Prusia).

Page 29: ENARM  COMPENDIO

PATOLOGIA CELULA PATOGNOMONICA

INTOXICACION POR PLOMO, LEUCEMIA Y TALASEMIA.

Punteado basofilo: Pueden ser agregados ribosómicos originados por una degeneración vacuolar del citoplasma o precipitados de cadenas globínicas libres. Consiste en puntitos basófilos, con las tinciones habituales, de tamaño variable y dispersos por toda la superficie del hematíe.

ANEMIA MEGALOBLASTICA Anillos de cabot: Están formados por restos de la membrana nuclear o de microtúbulos. Consisten en una especie de hilos basófilos, con las tinciones habituales, que adoptan una forma de anillo o de ocho y que pueden ocupar toda la periferia celular.

CÉLULA WARTHIN-FINKELDEY Sarampión: Célula gigante multinucleótica con citoplasma eosinofílico e inclusiones nucleares

Page 30: ENARM  COMPENDIO

PATOLOGIA• De que patología son característicos los CUERPOS DE HEINZ?R = Déficit de glucosa 6PDH

• De que patología son patognomónicos los cuerpos de HOWELL-JOLLY?R = Anemia de células falciformes

• De que patología son patognomónicos los BASTONES DE AUER?

R = Leucemia mieloide aguda en la que además la distingue la presencia de “MIELOPEROXIDASA”

• Que es la LEUCEMIA MIELOGENA CRÓNICA?R = Se caracteriza por SOBREPRODUCCIÓN DE CÉLULAS MIELOIDES (principalmente LEUCOCITOS), presenta EL COMOSOMA FILADELFIA

(translocacion de los brazos 9 y 22) BCR/ABL que produce “TIROCINA CINASA”.

• Que diferencia la LEUCEMIA MIELOIDE AGUDA de la leucemia linfoblastica aguda?R = Mieloperoxidasa que no se produce en la LLA

• Cual es el hallazgo patognomónico de linfoma de Hodking?R = RED STEMBERG CELLS (CÉLULA B MODIFICADA por lo general es muy grande dando la apariencia de ojos de Búho con nucléolos prominentes

en forma de inclusiones, la variedad mononuclear tiene un solo núcleo y un nucléolo prominente).

• Que patología te da trombos hialinos en la biopsia?R = PTT y SUH

• Citogenetica reconocible con T (18:14) de que enfermedad es patognomónico?R = Linfoma no Hodking

Page 31: ENARM  COMPENDIO

ENF. DE HODKING• Característica de la Enfermedad de Hodgkin?• Linfadenopatia indolora, Síntomas constitucionales, no se conoce su origen, diseminación

ordenada

• Cuadro clínico?• MASA INDOLORA (CUELLO), mas frecuente en varones, diseminación ordenada, fiebre de larga

evolución, DISMINUCIÓN DE PESO, PRURITO INTENSO, sudación nocturna, DOLOR GANGLIONAR (ALCOHOL)

• Diagnostico definitivo?• Biopsia del ganglio linfático afectado, CÉLULAS DE REED-STENBERG, no hay anormalidades

cariotipicas

• El virus Epstein Barr con que enfermedad tumoral se ha relacionado?• Enfermedad de Hodgking

• En la Enfermedad de Hodgking cual es la variedad histológica mas frecuente?• ESCLEROSIS NODULAR

Page 32: ENARM  COMPENDIO

• En la Enfermedad de Hodgking cual es la variedad de mejor pronostico?• VARIEDAD LINFOCITICA

• En la Enfermedad de Hodgking cual es la variedad menos frecuente?• Depleción linfocitaria

• En la Enfermedad de Hodgking que se relaciona con VIH?• DEPLECIÓN LINFOCITARIA

• Tratamiento de la de la Enfermedad de Hodgking?1) MOPP (mecloretamina, Vincristina, procarbacina, prednisona)2) ABVD (adriamicina, bleomicina, Vincristina, Dacarbacina)

• Clasificación de ann-arbor?• Etapa 1 una sola cadena ganglionar • Etapa 2 2 regiones ganglionares (mismo lado del diafragma)• Etapa 3 regiones ganglionares en ambos lados del diafragma• Etapa 4 metástasis • (A asintomático, B síntomas constitucionales)

Page 33: ENARM  COMPENDIO

ENF. NO HODKING• Característica de la Enfermedad no Hodgking?

• No tiene cél de Reed Stemberg, si hay citogenética reconocible T (8:14), correlación con proto-oncogen C-myc

• Cuadro clínico de la E no H?• Linfadenopatia desordenada, a menudo diseminada al momento del diagnostico, síntomas

constitucionales

• En que consiste el termino de ganglio de Richter?1) Es un ganglio generalmente SUPRACLAVICULAR .2) GANGLIO que comparte una LLC q posteriormente se convertirá (EVOLUCIONARA) en LINFOMA NO

HODGKIN

• Tratamiento de los linfomas no Hodgkin?1) 1 solo ganglio afectado: radioterapia local … si esta asintomático valorar conducta expectante2) Tx: clorambucilo o CVP+fludarabina, CHOP

• Tratamiento para un MALTOMA?• Terapia para HELICOBACTER PYLORI

Page 34: ENARM  COMPENDIO

LEUCEMIA AGUDA

• Cáncer mas frecuente en niños? Leucemia siendo la aguda la mas frecuente (80%)

• El cáncer de los ganglios linfáticos se llama? Linfoma

• El cáncer de medula ósea? Leucemia

• Característica de la leucemia Linfoblastica Aguda? Tienen blastos (20% en MO y 90% en sangre periférica) y su caract.

Es la capacidad de infiltrar órganos (encías, pericardio, testículos, MO, SNC etc), 2da. Capacidad de producir Citopenias o Pancitopenias

Page 35: ENARM  COMPENDIO

• Cuadro Clínico de LLA? Por procesos de infiltración hay: Hemorragias (petequias, equimosis) Adenopatías,

Hepato-esplenomegalias, puede llegar a una CID

• Diferencia entre LLA y la hipoplasia medular y purpura trombocitopenica idiopática

En que puede haber trombocitopenia pero TANTO EN LA HIPOPLASIA COMO EN LA PURPURA NUNCA habrá ganglios linfáticos palpables o hepatoesplenomegalias

• Características de la CID? Tiempo de protrombina alargado, dímero D presente, trombocitopenia.

• Que tipo de leucemia mielocitica aguda tiene la característica de presentarse como una CID

La promielocitica y la monocitica aguda (habitualmente son de buen pronostico mientras que no se presente con CID)

Page 36: ENARM  COMPENDIO

• Citogeneticas desfavorables? Monosomia 5 y 7, Cromosoma Ph t(4;11) (Encontrar Cromosoma ph en LMA(2%) es

de muy mal pronostico habitualmente se observa en LMC (98%)

• Con respecto a marcadores de superficie: Determinación de una Enzima desoxinucleotidil transferasa terminal….. es una LLA (95%)

• Si es MIELOPEROXIDASA POSITIVA…. LMA Marcador CD19, CD10…. Es una LLA de células B (mas frecuente) Marcador CD2, CD5, CD7…. Es una LLA de células T (menos frecuente)

• Tratamiento de Leucemias Mielocitica Agudas? 1ra etapa: Inducción de la Remisión Daunorrubicina o Idarrubicina mas citarabina 2da etapa: Quimioterapia intensa de repetición mas transplante de MO autologa

• Tratamiento de la Leucemia Linfoblastica aguda? 1ra etapa: Inducción de la Remisión Vincristina, Prednisona, Daunorrubicina, L Asparaginasa

(4 medicamentos) Metrotexate (profilaxis del SNC) 2da etapa: Quimioterapia a dosis altas mas transplante de MO

Page 37: ENARM  COMPENDIO

• Leucocitosis con Desviación a la DERECHA: Cuando los LINFOCITOS son los que están elevados (arriba del 20%)

• Leucocitosis con Desviación a la IZQUIERDA: Cuando los NEUTRÓFILOS están elevados

• Característica Histológicas de LMA? Mieloperoxidasas positivas, cuerpos de AUER en citoplasma

• Características Histológicas de LLA? Antígeno CALLA (Antígeno común de LLA) y PAS Acido periódico de Shift

positivo

• Citogenética Favorables? En px con LMA t(8:21), t(15:17),inv (cr16), Inv (p13; q22) buen

pronostico

Page 38: ENARM  COMPENDIO

LEUCEMIA CRONICA• Característica de una LLC?1) Linfocitos mayores de 5000, linfocitos de apariencia normal (madura), con expresión de CD19, y CD52) Origen: malignidad de linfocitos B (95%) inmunoincopetentes

• Cuadro clínico de una LLC? Se presenta en px MAYORES DE 50 AÑOS (90%), linfocitosis impresionante, linfadenopatia y

Hepatoesplenomegalia

• Clasificación de la LLC (CLASIFICACION RAI) ETAPA O Linfocitosis mas de 5000 ETAPA 1 Linfocitosis MAS LINFADENOPATIA (revisar al px) ETAPA 2 Linfocitosis MAS ORGANOMEGALIA HEPATOESPLENOMEGALIA ETAPA 3 Linfocitosis mas anemia Etapa 4 Linfocitosis , lo anterior mas trombocitopenia (mal pronostico)

• Laboratorios de una LLC? LINFOCITOSIS AISLADA (mas de 5000), Leucos mayor de 20000, inmunogenotipo: CD19 (LLC de Celulas

B), CD5 (LLC de Celulas T)

Page 39: ENARM  COMPENDIO

• Tratamiento de LLC con sintomatología? CLORAMBUCILO alternativa fludarabina en px jóvenes

• Característica de LMC?1) Leucocitosis, Cromosoma Ph t(9:22) presentes en un 95%,2) Leucocitosis con desviación a la izquierda (aumento delos

neutrófilos)

• Tx de LMC? EL TX no es urgente Mesilato de Imatinib, alternativa Hidroxiurea

o interferon alfa

• Cuadro clínico de LMC? Crecimiento impresionante de bazo, mas leucocitosis con desviación

a la izq con serie mieloide, sensibilidad esternal, cromosoma Ph (95%)

Page 40: ENARM  COMPENDIO

SINDROME PARANEOPLASICO/ AC• Frecuentemente presentan pleocitosis linfocitaria en LCR

con aumento de proteínas y de IgG. Algunos pacientes presentan anticuerpos característicos en suero y LCR que proveen una fuerte evidencia de que el cuadro neurológico es paraneoplásico y que además sugieren el tipo de cáncer asociado, por ejemplo el anticuerpo anti-Yo se asocia con degeneración cerebelosa y cáncer ginecológico, el anticuerpo anti-Hu con encéfalomielitis paraneoplásica y cáncer pulmonar de células pequeñas y el anticuerpo anti-Ki con opsocionus y cáncer de mama

Page 41: ENARM  COMPENDIO

MICELANEAS• En un paciente con anemia ferropénica que

esperaría encontrar en su estudio de laboratorio?

• Ferritina, ferremia y saturación de transferrina bajas y transferrina alta.

• Por cuanto tiempo se realiza el tratamiento de la anemia ferropenica?

• R = 6 meses

Page 42: ENARM  COMPENDIO

CARDIOLOGIA

ENARM

Page 43: ENARM  COMPENDIO

EKG

Page 44: ENARM  COMPENDIO

EKG I N T E R V A L O S

PR: Se denomina así, al espacio que va del inicio de la Onda P al comienzo de la Onda R.

Valor normal: 120 a 200 milisegundos ( 0.12 – 0.20 segundos).

COMPLEJO QRS: Corresponde a la activación del miocardio ventricular.

Valor Normal: 60 a 100 milisegundos ( 0.06 – 0.10 segundos).

QT: Se mide desde el inicio del Complejo QRS hasta el final de la Onda T y corresponde a la duración total de la Sístole

Ventricular.

Valor normal: 240 a 480 milisegundos ( 0.24 – 0.48 segundos ).( Varia de acuerdo a la frecuencia cardiaca )

S E G M E N T O ST: Es el intervalo normalmente iso-electrico entre el final del Complejo QRS y el inicio de la Onda T.

Valor normal: 60 a 160 milisegundos ( 0.06 a 0.16 segundos )

Page 45: ENARM  COMPENDIO

EKG• I N T E R V A L O Q T

P r o l o n g a d o: C a r d i o p a t í a I s q u é m i c a. I. C. C. M i o c a r d i t i s. Drogas: Quinidina, Amiodarona, Antidepresivos triciclicos...

H i p o m a g n e s e m i a. H i p o c a l c e m i a. H i p o k a l e m i a ?.

A c o r t a d o: R e p o l a r i z a c i ó n p r e c o z. D r o g a s: Digital...

H i p e r c a l c e m i a. H i p e r k a l e m i a.

Page 46: ENARM  COMPENDIO

HIPOKALEMIA

• < 3 mEq/L: onda T plana, depresión ST, ondas U

• < 2,5 mEq/L: onda U prominente, inversión onda T, PR y QT prolongado, QRS ensanchado

Page 47: ENARM  COMPENDIO

HIPERKALEMIA : POTASIO SÉRICO > 5 mEq/L

LEVE 5.0 - 5.5 mEq/L

MODERADA 5.5 - 6.0 mEq/L

SEVERA > 6.0 mEq/L

Page 48: ENARM  COMPENDIO

K+ 5.0 - 6.5 mEq/L Cambios mínimos

K+ 6.5 - 8.0 mEq/L Onda T picudaAplanamiento y desaparición de la onda

PDepresión del segmento ST

K+ 8.0 a más Ensanchamiento del QRSBAVArritmias cardíacasRitmo idioventricularParo cardíaco en asistolia.

HIPERPOTASEMIA Y EKG

Page 49: ENARM  COMPENDIO

EJES ELECTRICOS EKG

Page 50: ENARM  COMPENDIO

PULSO VENOSO YUGULARAXVAYC

• Se distinguen fundamentalmente dos ondas, la "a" y la "v". La primera, la onda "a", ocurre justo antes del sístole, y se debe a la CONTRACCIÓN DE LA AURÍCULA DERECHA (al final del diástole, cuando se termina de vaciar al ventrículo derecho). El colapso de la vena

después de la onda "a", es el descenso "x" y se debe a la RELAJACIÓN DE LA

AURÍCULA. AX

• La onda "v" se debe al LLENE PASIVO DE LA AURÍCULA DERECHA debido al retorno venoso normal, mientras la VÁLVULA TRICÚSPIDE PERMANECE CERRADA durante el sístole. Por lo tanto, es una onda que ocurre al mismo tiempo del sístole y que se vería sobre el vena yugular. El colapso que se observa después de la onda "v", se denomina

el descenso "y", que corresponde al PASO DE LA SANGRE DE LA AURÍCULA AL

VENTRÍCULO DURANTE EL DIÁSTOLE, después que se abre la válvula tricúspide. VAY• Con registros muy finos, se describe una pequeña muesca ubicada en el descenso de la

onda "a", que se ha llamado la onda "c", ATRIBUIDA AL CIERRE DE LA VÁLVULA TRICÚSPIDE, después que se ha terminado de contraer la aurícula derecha y está comenzando el sístole, pero no es posible de ver a simple vista.

Page 51: ENARM  COMPENDIO
Page 52: ENARM  COMPENDIO
Page 53: ENARM  COMPENDIO

FARMACOLOGIA

Page 54: ENARM  COMPENDIO
Page 55: ENARM  COMPENDIO

ANGINA DE PECHO

Page 56: ENARM  COMPENDIO

ANGINA ESTABLE

• Es la más frecuente, aparece con el esfuerzo y remite espontáneamente con el reposo y/o la medicación. Posee una duración de pocos minutos y presenta un patrón regular, por lo que el paciente puede ser capaz de identificarla e incluso predecir su aparición. Su origen se halla primordialmente en una arteriopatía aterosclerótica que causa la progresiva reducción de la luz vascular, de uno o varios vasos coronarios, en porcentajes del orden del 70% o superior.

Page 57: ENARM  COMPENDIO
Page 58: ENARM  COMPENDIO

ANGINA INESTABLE• La angina inestable no se relaciona con un mayor trabajo cardíaco, es

decir no deriva de un mayor consumo miocárdico de oxígeno. Su causa debe buscarse en una disminución aguda del flujo cardíaco coronario, que puede deberse a la complicación de una placa aterosclerótica coronaria por erosión, fisura o rotura y trombosis sobreañadida que cause una interrupción súbita del flujo coronario o por causas extrínsecas al árbol coronario que produzcan inestabilización. Su sintomatología clínica es muy similar a la que registra el infarto agudo de miocardio, sin embargo, en la angina inestable no se produce necrosis miocárdica.

• La angina inestable incluye diversos tipos de anginas caracterizadas por su evolución imprevisible, aunque no siempre fatal y que se apartan claramente del patrón típico de angina estable: angina de reciente comienzo, angina progresiva, angina de reposo, angina prolongada, angina vasoespástica o de Prinzmetal y angina postinfarto, todas ellas consideradas urgencias médicas.

Page 59: ENARM  COMPENDIO

IAM

Page 60: ENARM  COMPENDIO

IAM

Page 61: ENARM  COMPENDIO

IAM

Page 62: ENARM  COMPENDIO

SICA

Page 63: ENARM  COMPENDIO

CORONARIAS INVOLUCRADAS EN IAMDERIVACION CON ELEVACION DEL ST TIPO DE INFARTO

ARTERIA CORONARIA

RESPONSIBLECOMPLICACIONES

V1-V2 SeptalDescendente

anterior (ramos septales)

Bloqueos de rama

V3-V4 Pared anteriorDescendente

anterior (ramos diagonales)

Disfuncion VI, IC, Bloqueo de rama

V5-V6 Lateral alto CircunflejaHipotension (evitar

nitroglicerina o morfina)

DII, DIII y aVFPosteroinferior con

extension al ventriculo derecho

Derecha (ramos proximales) Hipotension

V1-V4 PosteriorCircunfleja o descendente

posteriorDisfuncion VI

Page 64: ENARM  COMPENDIO
Page 65: ENARM  COMPENDIO

PREGUNTAS

Cual es la triada clásica del infarto del ventrículo derecho?

Hipotensión, campos pulmonares limpios y elevación de la PVY

Cual es el tratamiento de la taquicardia auricular multifocal?

Suele asociarse a enfermedad pulmonar grave, mejora con la ventilación mecánica y la oxigenación

Page 66: ENARM  COMPENDIO

TIPOS DE SHOCK

Page 67: ENARM  COMPENDIO

VALVULOPATIAS

Page 68: ENARM  COMPENDIO
Page 69: ENARM  COMPENDIO

RESUMEN SOPLOS• ESTENOSIS MITRAL; SINTOMAS Y SOPLO DISNEA de esfuerzo, ORTOPNEA, EAP, disfonía por compresión del NLR.

Imagen de 4 arcos en Rx por congestion venocapilar. Soplo diastólico• INSUFICIENCIA MITRAL; SINTOMAS Y SOPLO DISNEA de esfuerzo y fatiga, ORTOPNEA, DISNEA PAROXÍSTICA

NOCTURNA. Por prolapso valvular dolor torácico (signo de Barlow) dx dif con SICA.

Soplo sistólico• ESTENOSIS AORTICA; SINTOMAS Y SOPLO Triada clásica, ANGINA DE PECHO - IC – SINCOPE. Palpitaciones, visión

borrosa. HV. Soplo sistólico • INSUFICIENCIA AORTICA; SINTOMAS Y SOPLO Disnea que va de esfuerzo a DISNEA PAROXÍSTICA NOCTURNA, estertores. Soplo diastólico.

Page 70: ENARM  COMPENDIO

BRI

Page 71: ENARM  COMPENDIO

BRD

Page 72: ENARM  COMPENDIO

B AV I

• Prolongacion del intervalo PR en forma continua

• >20” adultos• >16” en infantes

Page 73: ENARM  COMPENDIO

B AV II/ M1

Page 74: ENARM  COMPENDIO

B AV II/M2

Page 75: ENARM  COMPENDIO

B AV C

Page 76: ENARM  COMPENDIO

RESUMEN BLOQUEO AV I-III

Page 77: ENARM  COMPENDIO

BLOQUEOS AV MANEJO

Page 78: ENARM  COMPENDIO

FA

Page 79: ENARM  COMPENDIO
Page 80: ENARM  COMPENDIO

QT LARGO

Page 81: ENARM  COMPENDIO

TRAZOS EKG PATOLOGICOS

Page 82: ENARM  COMPENDIO

SX WPW

Page 83: ENARM  COMPENDIO

SX BRUGADA

Page 84: ENARM  COMPENDIO

INSUFICIENCIA CARDIACA

Page 85: ENARM  COMPENDIO

INSUFICIENCIA CARDIACA

Page 86: ENARM  COMPENDIO
Page 87: ENARM  COMPENDIO

ENDOCARDITIS INFECCIOSA

Page 88: ENARM  COMPENDIO
Page 89: ENARM  COMPENDIO
Page 90: ENARM  COMPENDIO

Endocarditis infecciosa

A. Amoxicilina 2gr 1 hora antes

B. Ampicilina 2gr IM 30 minutos antes (en intolerancia oral)

Alergia a penicilina

1) Claritromicina 500 mg 1 hora antes

2) Clindamicina 600 mg 1 hora antes

3) Cefalexina 2gr 1 hora antes

Profilaxis antibiótica en procedimientos dentales, cavidad oral, respiratorio y esófago

Page 91: ENARM  COMPENDIO

Endocarditis infecciosa

• Ampicilina 2 gr IM o IV + Gentamicina 1,5mg/Kg/ 30 minutos antes, 6 h después Ampicilina 1gr IM/IV o Amoxicilina oral 1gr

Alérgicos a penicilina

• Vancomicina 1gr IV en 1-2 h + Gentamicina 1,5 mg/Kg IV/IM terminando la perfusión 30 minutos después del procedimiento

Profilaxis antibiótica en procedimientos genitourinarios y gastrointestinales

Page 92: ENARM  COMPENDIO

LESIONES DE JANEWAY

Page 93: ENARM  COMPENDIO

MANCHAS DE ROTH

Page 94: ENARM  COMPENDIO

NODULOS DE OSLER

Page 95: ENARM  COMPENDIO

TAMPONADE

Page 96: ENARM  COMPENDIO

JOVEN + DOLOR PRECORDIAL, FIEBRE Y ST (LESIÓN) EXTENSA = PERICARDITIS AGUDA. LO EXACERBA LA RESPIRACIÓN.

ES FRECUENTE ANTECEDENTE DE IRAS VIRALESDE NO CONOCERSE LA ETIOLOGÍA SE DAN AINES (AAS)

Page 97: ENARM  COMPENDIO

PERICARDITIS• El supradesnivel del ST en la pericarditis se diferencia del infarto

agudo del miocardio porque en éste el supradesnivel es convexo y más localizado, pueden coexistir ondas T negativas al mismo tiempo del supradesnivel, presencia de ondas Q cuando es un infarto Q y el EKG no normaliza antes de hacerse negativa la onda T.

• De la angina de Prinzmetal se distingue porque el supradesnivel es transitorio y sólo durante el dolor; además, compromete derivaciones más localizadas.

• La imagen de la repolarización precoz es parecida pero nunca hay depresión del PR, no evoluciona y no hay síntomas.

Page 98: ENARM  COMPENDIO

MIOCARDIOPATIAS

Page 99: ENARM  COMPENDIO

Adulto + anasarca + complejo bajos = pericarditis constrictiva

Page 100: ENARM  COMPENDIO

Rx de mujer , imagen de hiperflujo pulmonar, ECG con BRD y eje a la derecha= Comunicación Interauricular (OS)

Page 101: ENARM  COMPENDIO

CARDIOPATIAS CIANOGENAS Y ACIANOGENAS

Page 102: ENARM  COMPENDIO

CIV

Page 103: ENARM  COMPENDIO

CIA

Page 104: ENARM  COMPENDIO

COARTACION AORTICA

Page 105: ENARM  COMPENDIO

ANEURISMAS

Page 106: ENARM  COMPENDIO

CARDIOPATIAS CONGENITAS

Page 107: ENARM  COMPENDIO

DISECCION AORTICA

Page 108: ENARM  COMPENDIO

EMBOLIA VS TROMBOSIS

Page 109: ENARM  COMPENDIO

OCLUSION ARTERIAL CRONICA

Page 110: ENARM  COMPENDIO

HAS

Page 111: ENARM  COMPENDIO

HAS

Page 112: ENARM  COMPENDIO

HAS

Page 113: ENARM  COMPENDIO

HAS

Page 114: ENARM  COMPENDIO
Page 115: ENARM  COMPENDIO

• Cual es el signo electrocardiografico mas común en tromboembolismo pulmonar?• R = Taquicardia sinusal.

• -A 42-year-old man develops shortness of breath (SOB) and chest pain 7 days after an open cholecystectomy. His blood pressure is 145/86 mm Hg, pulse is 120/min, respirations 24/min, and oxygen saturation of 97%. Pulmonary embolism is clinically suspected. Which of the following is the most common ECG finding of pulmonary embolism?

• (A) a deep S wave in lead I• (B) depressed ST segments in leads I and II• (C) prominent Q wave in lead I, and inversion of T wave in lead III• (D) sinus tachycardia• (E) clockwise rotation in the precordial leads• Correcta D

• Cuales son los síntomas de estenosis mitral?• R = DISNEA, ortopnea y disnea paroxística nocturna

• Como es el soplo en insuficiencia mitral?• R = PanSISTOLICO en vértice e irradiado a axila.

• -A 25-year-old woman is found to have a midsystolic murmur on routine evaluation. The murmur does not radiate but it does increase with standing. She otherwise feels well and the rest of the examination is normal.

• R = Mitral valve prolapsed.

• -A 65-year-old man with a previous history of an anterior MI comes for follow-up. On examination, he has a systolic murmur heard best at the apex and radiating to the axilla. Transient external compression of both arms with blood pressure cuffs 20 mm Hg over peak systolic pressure increases the murmur.

• R = Regurgitación mitral

Page 116: ENARM  COMPENDIO

• Que caracteriza al signo de MUSSET y en que patología se presenta?• R = Hay movimientos de la cabeza en cada pulsación debido a INSUFICIENCIA DE LA VÁLVULA AORTICA.

• -A 75-year-old man is bought to the hospital because of a syncopal episode. There was no incontinence or post-event confusion. On examination, his blood pressure is 140/80 mm Hg, pulse 72/min with no postural changes. His second heart sound is diminished and there is a systolic ejection murmur that radiates to the carotids. With the Valsalva maneuver, the murmur decreases in length and intensity.

• R = Aortic stenosis.

• En que patología se escucha el soplo de AUSTIN FLINT?• Insuficiencia aortica

• Que sucede cuando hay regurgitación aortica en forma aguda?• Insuficiencia ventricular izquierda manifestada como EDEMA AGUDO PULMONAR.

• -A 32-year-old asymptomatic woman has a rapidly rising, forceful pulse that collapses quickly. Which of the following is the most likely diagnosis?• (A) mitral stenosis• (B) mitral regurgitation• (C) aortic stenosis• (D) aortic regurgitation• Respuesta correcta D

• Que medicamento es de elección en el síndrome de Marfan con regurgitación aortica?• R = BETABLOQUEADORES

• Que medicamentos pueden disminuir la dosis de homocisteina?• R = Acido fólico, B6 y B12.

• Cuál es el fenómeno de hibernación y aturdimiento miocardico?• R = Son las áreas del miocardio que se encuentran subperfundidas que se adaptan para ser viables con disfunción contráctil sostenida.

• Que es el síndrome de TIETZE?• R = Inflamación de uniones condrocostales tumefactas, rojas que causa dolor retroesternal

• Que antianginosos han demostrado prolongar la vida?• R = Los BETABLOQUEADORES a excepción de el pindolol que exacerba la angina

Page 117: ENARM  COMPENDIO

RESUMEN SOPLOS• ESTENOSIS MITRAL; SINTOMAS Y SOPLO Disnea de esfuerzo, ortopnea, EAP, disfonía por compresión del NLR.

Imagen de 4 arcos en Rx por congestión venocapilar. Soplo diastólico• INSUFICIENCIA MITRAL; SINTOMAS Y SOPLO Disnea de esfuerzo y fatiga, ortopnea, disnea paroxística nocturna. Por

prolapso valvular dolor torácico (signo de Barlow) dx dif con SICA. Soplo sistólico• ESTENOSIS AORTICA; SINTOMAS Y SOPLO Triada clásica, angina de pecho - IC – Sincope. Palpitaciones, visión

borrosa. HV. Signo de Musset. Soplo sistólico • INSUFICIENCIA AORTICA; SINTOMAS Y SOPLO• Disnea que va de esfuerzo a disnea paroxística nocturna, estertores.

PULSO DE CORRIGAN , QUINQUE Y DUROZEIZ . Soplo de Austin Flint. Soplo diastólico.

Page 118: ENARM  COMPENDIO

• En cuanto tiempo tiene que estar el TTP cuando se da tratamiento anticoagulante?• R = 50-70 segundos.

• En que pacientes se utiliza la profilaxis antiarrítmica?• R = LATIDOS ECTÓPICOS FRECUENTES y taquicardia ventricular.

• Como se trata una fibrilación ventricular que no responde a una cardioversión?• R = Amiodarona y RCP

• Como se dx infarto del ventrículo derecho?• R = Con la ELEVACIÓN DEL ST DEL HEMITORAX DERECHO

• -A 69-year-old woman complains of some atypical chest pain 2 days prior to presentation. On examination, the JVP is at 8 cm, positive Kussmaul’s sign, and normal heart sounds. The lungs are clear. The ECG is abnormal, and the CXR shows a normal cardiac silhouette.

• Que tan frecuente se presenta un AUNERISMA VENTRICULAR IZQUIERDO POST IAM?• R = 20% y se da el dx por medio de la ELEVACIÓN PERSISTENTE DEL ST DURANTE 4-8 SEMANAS.

• -Three months after an anterior MI, a 73-yearold man has a follow-up ECG. He is clinically feeling well with no further angina symptoms. His ECG shows Q waves in the anterior leads with persistant ST-segment elevation. The current ECG is most compatible with which of the following diagnosis?

• (A) ventricular aneurysm• (B) hibernating myocardium• (C) acute infarction• (D) silent infarction• (E) early repolarization

Page 119: ENARM  COMPENDIO

• Que caracteriza al SÍNDROME DE DRESSLER?• R = SE PRESENTA POST IAM, debido a un FENÓMENO AUTO INMUNITARIO CON PERICARDITIS, FIEBRE, LEUCOCITOSIS y

ocasionalmente derrame pericardico o pleural.

• -A 67-year-old man presents with an anterior myocardial infarction (MI) and receives thrombolytic therapy. Three days later, he develops chest pain that is exacerbated by lying down, and his physical findings are normal except for a friction rub. His ECG shows evolving changes from the anterior infarction but new PR-segment depression and 1-mm ST-segment elevation in all the limb leads. Which of the following is the most likely diagnosis?

• (A) reinfarction• (B) pulmonary embolus• (C) viral infection• (D) post-MI pericariditis• (E) dissecting aneurysm• Respuesta correcta D

• Cual es el manejo del infarto agudo del VD?• R = CARGA LIQUIDA para mejorar el llenado ventricular derecho e INOTRÓPICOS.

• Cual es el mecanismo de acción de los antiarrítmicos clase Ia y cuales son?• R = BLOQUEADORES DE LOS CANALES DE SODIO. Procainamida, quinidina y disopiramida.

• Cual es el mecanismo de acción de los antiarrítmicos clase Ib y cuales son?• R = BLOQUEADORES DE LOS CANALES DE SODIO. Lidocaina y difenilhidantoina.

• Cual es el mecanismo de acción de los antiarrítmicos clase II y cuales son?• R = BETABLOQUEADORES, retardando la conducción AV. Propanolol, metoprolol y esmolol.

Page 120: ENARM  COMPENDIO

• Cual es el mecanismo de acción de los antiarrítmicos clase III y cuales son?• R = PROLONGAN EL POTENCIAL DE ACCIÓN. Amiodarona, sotalol, dofelitida.

• Cual es el mecanismo de acción de los antiarrítmicos clase IV y cuales son?• R = BLOQUEADORES DE LOS CANALES LENTOS DE CALCIO. Verapamil, diltiazem, digoxina y adenosina.

• Con que fenómenos se relaciona la taquicardia paroxística supraventricular, la cual suele cursar asintomática.• R = Intoxicación por digitalicos, bloqueos AV y fenómeno de reentrada.

• Que tratamiento se utiliza en taquicardia paroxística supraventricular en caso de estar contraindicados los antiarrítmicos clase IV y que ha demostrado 100% de éxito?

• R = Cardioversion.

• Cual es el fármaco de elección en caso de prevención de ataques de taquicardia paroxística supraventricular?• R = DIGOXINA

• Que caracteriza al síndrome de LOWN-GANONG-LEVINE?• R = INTERVALO PR CORTO Y MORFOLOGÍA NORMAL DE QRS

• En que casos no necesita tratamiento las taquicardias supraventriculares causadas por vías accesorias o síndrome de preexitacion?

• R = En casos de no presentar palpitaciones, mareos o sincope.

• Cual es el procedimiento de elección en pacientes con síndrome de preexitacion?• R = ABLACIÓN POR RADIOFRECUENCIA

• Que agentes farmacológicos deben evitarse en el síndrome de preexitacion?• R = Digoxina, betabloqueador y bloqueador de los canales de calcio.

• Cual es el tratamiento a largo plazo de el síndrome de preexitacion?• R = Procainamida, verapamil y digoxina.

Page 121: ENARM  COMPENDIO

• Cuales son los fármacos de elección en extrasístoles ventriculares sintomáticas?• R = BETABLOQUEADORES.

• Que medida se utiliza cuando la taquicardia ventricular es recurrente?• R = Marcapasos

• - A22-year-old woman complains of palpitations and has a regular heartbeat at a rate of 170/min, with a blood pressure of 110/70 mm Hg. The rate abruptly changes to 75/min after applying carotid sinus pressure. Which of the following is the most likely diagnosis?

• (A) sinus tachycardia• (B) paroxysmal atrial fibrillation• (C) paroxysmal atrial flutter• (D) paroxysmal supraventricular tachycardia (PSVT)• Respuesta correcta D

• Cual es el mecanismo de elección para la prevención de muerte súbita en un paciente con factores de riesgo?• R = Desfibrilador implantado

• Que caracteriza al síndrome QT PROLONGADO? • R = SINCOPE RECURRENTE, arritmias ventriculares y muerte súbita. Es una ANOMALÍA CONGÉNITA hereditaria que afecta los CANALES DE Na y K.

• Cual es la terapéutica mas eficaz en el síndrome de QT prolongado?• R = BETABLOQUEADORES Y DESFIBRILADOR IMPLANTADO.

• Que hallazgo EKG caracteriza al bloqueo AV tipo I?• R = PROLONGACIÓN DEL PR > .20” EN ADULTOS Y > .16”. LA ALTERACIÓN ES NODAL.

• Que hallazgo EKG caracteriza al bloqueo AV tipo II?1) - 1er grado o fenómeno de WENCKEBACH, MOBITZ I: prolongación progresiva del PR hasta que 1 onda P no conduce. 2) - 2do grado o MOBITZ II: intervalo PR, sin embrago hay onda P que no conduce con un patrón 2:1 o 3:1

• En que casos se diagnostica el síndrome de SENO ENFERMO?• R = PARO SINUSAL, bloqueo sinoauricular, BRADICARDIA SINUSAL PERSISTENTE o bradiarritmias.

Page 122: ENARM  COMPENDIO

• Cual es el manejo del síndrome de seno enfermo?• R = TEOFILINA ORAL Y MARCAPASOS.

• Cual es la indicación terapéutica del Mobitz III?• R = MARCAPASOS

• Cuales son los síntomas inadvertidos de ICC?• R = Tos crónica, nicturia

• Que medicamento se utiliza en caso de intoxicación por digoxina?• R = Anticuerpos FAP

• Cual es el cuadro clínico característico de alguien con ICC con edema agudo pulmonar?• R = DISNEA, tos, ESPUTO ROSADO, diaforesis, cianosis.

• En que consiste el fenómeno de reentrada?• R = Involucra un circuito que conduce de forma anterograda al ventrículo y de forma retrograda a la aurícula

utilizando vías accesorias.

• Que tratamiento se utiliza en ICC después de que los diuréticos y nitratos no mejoran la sintomatología?• R = NISERITIDA, forma recombinante de péptido atrial natriuretico del cerebro humano.

• Que agente antihipertensivo se debe evitar en ICC?• R = CALCIOANTAGONISTAS

• Como es típicamente la Rx de torax en edema agudo pulmonar con ICC?• R = PATRÓN EN MARIPOSA en la distribución del edema alveolar.

Page 123: ENARM  COMPENDIO

• En el EAP la presión capilar en cuna se encuentra siempre elevada, a que cantidad aproximadamente? • R = > 25 mm Hg

• La morfina es eficaz en el manejo de derrame pleural cardiogenico, debido a que mecanismo?• R = AUMENTA LA CAPACITANCIA VENOSA, disminuye la presión auricular izquierda y disminuye la ansiedad.

• Cuál es la diferencia entre cardioversión y desfibrilación?• R = La primera se encuentra sincronizada con complejo QRS y la 2da no hay sincronización.

• Cuáles son los agentes más comunes para miocarditis aguda?• R = VIRAL (COXACKIE) bacteriano, riketsias, espiroquetas, micoticos y parasitarios.

• -A 23-year-old man develops sharp left-sided chest pain, fever, and a friction rub heard at the lower left sternal border, unaffected by respiration. The pain is also aggevated by lying down and relieved by sitting up. He is otherwise well with no other symptoms and the remaining physical examination is normal. Which of the following is the most likely cause for his symptoms?

• (A) rheumatic fever• (B) tuberculosis (TB)• (C) herpes simplex virus• (D) MI• (E) coxsackievirus• Respuesta correcta E

• La miocarditis Riketsial , por que enfermedad es originada? • R = TIFO, fiebre de las montanas rocallosas y fiebre Q.

Page 124: ENARM  COMPENDIO

• Cual es el agente causal y cual es el periodo de incubación de la ENFERMEDAD DE CHAGAS?• R = Es causado por el TROFOSZOITO TRIPANOSOMA CRUZI y su principal manifestación clínica aparece

en 10 años.

• Que afección sistémica es la regla para un individuo que padece Chagas?• R = MEGAESOFAGO O MEGACOLON.

• Entre las afecciones parasitarias cual es la mas frecuente de afección cardiaca?• R = La triquinosis.

• Cual es la fisiología de la maniobra de valsalva?• R = Se produce un aumento del tono parasimpático por liberación de acetilcolina, lo que genera un

retardo en la conducción AV causando un bloqueo AV transitorio.

• Que dato puede ser único o inicial en el EKG en MIOCARDITIS INFECCIOSA?• R = Ectopia ventricular

• Que medicamento esta indicado en el espasmo coronario inducido por la COCAÍNA?• R = BETABLOQUEADORES Y CALCIOANTAGONISTAS

• Cuales son los síntomas de MIOCARDIOPATIA DILATADA?• R = Disnea, dolor torácico .

• Cual es el manejo del paciente con CARDIOPATÍA HIPERTRÓFICA?• R = BETABLOQUEADORES, CALCIOANTAGONISTAS Y MARCAPASOS.

Page 125: ENARM  COMPENDIO

• -A 45-year-old woman has developed increasing SOB on exertion and fatigue. She has a loud systolic ejection murmur heard best at the left sternal border, and the murmur increases with standing. A double apical impulse is also felt.

• R = Cardiomiopatia hipertrofica.

• 66. Cuales son las causas principales de MIOCARDIOPATIA RESTRICTIVA?• R = AMILOIDOSIS, RADIACIÓN, fibrosis por cirugía, sarcoidiosis, hemocromatosis y síndrome carcinoide.

• -57 A 63-year-old man develops edema, and dyspnea on exertion. He has no prior cardiac or renal conditions, and his examination is significant for macroglossia, elevated jugular venous pressure (JVP), hepatomegaly, and 3+ pedal edema. His investigations reveal 3.5 g/d of protein in the urine, anemia, normal fasting glucose, and serum immunoelectrophoresis is positive for a monoclonal immunoglobulin. Which of the following is the most characteristic neurologic finding associated with this condition?

• -Peripheral motor and sensory neuropathy: In addition to peripheral motor and sensory neuropathy, cardiac involvement, tongue enlargement, gastrointestinal (GI) manifestations, and carpal tunnel syndrome are also seen in amyloidosis. The specific diagnosis requires tissue biopsy with presence of amyloid with specific stains. In primary amyloidosis and myeloma, the amyloid protein is of the ALtype. In reactive amyloidosis, the protein is of the amyloid Aprotein (AA) type.

• Cuales son los criterios mayores de Jones de CARDIOPATÍA REUMÁTICA?1) CARDITIS: pericarditis, ICC, cardiomegalia y soplos.2) ERITEMA MARGINADO 3) NÓDULOS SUBCUTÁNEOS: los nódulos subcutáneos SON INDOLOROS y menores de 2 cm4) COREA DE SHYDENHAM: Movimientos coreoatetoides5) ARTRITIS: dura de 1-5 semanas y responde a AINES

• Cuales son los criterios menores de cardiopatía reumática?• R = Fiebre, VSG elevada, prolongación del PR.

• Cual es el manejo de cardiopatía reumática?• R = PENICILINA BENZATINICA 1.2 millones DU o PEN-PRO 600, 000 x 10 dias.

• Como se previene la fiebre reumática?• R = PEN-BEN 1.2 cada 4 semanas

• En los pacientes con Wolf-Parkinson-Withe. Que medicamentos están contraindicados?• R = ADENOSINA, DIGOXINA, CALCIOANTAGONISTAS Y BETABLOQUEADORES YA QUE BLOQUEAN EL NODO AV.

Page 126: ENARM  COMPENDIO

• En los pacientes con Wolf-Parkinson-Withe. Cual es el manejo indicado?• R = Cardioversión eléctrica y ablación por radiofrecuencia de vías accesorias.

• Cuales son los síndromes de preexitacion?• R = Síndrome de WPW y Lown-Ganong-Levine

• Cual es el microorganismo causal de la ENFERMEDAD DE LYME?• R = Borrelia Burgdorjeri

• Que características tiene la pericarditis secundaria a síndrome urémico?• R = Pericardio afelpado, derrame hemorrágico y exudativo.

• Que caracteriza al síndrome de Dressler?• R = Pericarditis post IAM o cirugía del corazón abierto y de etiología autoinmune.

• Como se define la hipertensión pulmonar primaria?• R = Como aumento de la resistencia vascular pulmonar sin enfermedad subyacente.

• -Auscultation of the heart of a 17-year-old boy reveals an increased intensity of the pulmonary component of the second heart sound. He complains of dyspnea on exertion but no other cardiac or pulmonary symptoms. Which of the explanations is the most likely cause of his dyspnea?

• (A) pulmonary stenosis• (B) aortic stenosis• (C) MI• (D) pulmonary hypertension• (E) systemic hypertension• Respuesta correcta D

• Cual es el manejo de hipertensión pulmonar primaria?• R = CALCIOANTAGONISTAS, anticoagulantes, prostaciclina (potente vasodilatador pulmonar)

Page 127: ENARM  COMPENDIO

• Cual es el tratamiento de elección de la hipertensión primaria?• R = Endotelina

• A que hace alusión el termino Cor-pulmonale?• R = Hipertrofia ventricular derecha e insuficiencia por enfermedad pulmonar, mas frecuente en EPOC

• Cual es el dato de laboratorio mas frecuente en el Cor-pulmonale?• R = Policitemia

• Cuales son los tumores que afectan al corazón?• R = Carcinoma broncogenico, carcinoma mamario, melanoma maligno, linfomas, carcinoma de células renales y sarcoma de kaposi.

• Cuales son los tumores primarios del corazón mas frecuentes?• R = MIXOMA AURICULAR el cual es un tumor benigno que puede embolizar sistémicamente y se ubica comúnmente en la aurícula

derecha.

• -A 47-year-old woman has new-onset transient right arm weakness and word finding difficulty symptoms lasting 3 hours. She is also experiencing exertional dyspnea, and had a syncopal event 1 month ago. Her echocardiogram reveals a cardiac tumor in the left atrium, it is pendunculated and attached to the endocardium. Which of the following is the most likely cause of this lesion?

• (A) myxoma• (B) sarcoma• (C) rhabdomyoma• (D) fibroma• (E) lipoma• Respuesta correcta A

• En que consiste la ENFERMEDAD DE BURGUER?• R = Es un PROCESO INFLAMATORIO Y TROMBOTICO de las arterias y venas periféricas PRODUCIENDO CLAUDICACIÓN, DOLOR Y

NECROSIS.

• Cuales son los datos clínicos mas frecuentes de la enfermedad de Takayasu?• R = Soplos vasculares, PULSOS PERIFÉRICOS DISMINUIDOS y ASIMETRÍA DE LA PA DE LAS EXTREMIDADES.

• Cual es el tratamiento de la enfermedad de Takayasu?• R = ESTEROIDES para la inflamación y después COLOCAR STENT O PUENTES.

Page 128: ENARM  COMPENDIO

• Cual es el tratamiento para pacientes con displasia fibromuscular de la intima?• R = ASA

• Que fenómenos deben descartarse en una persona que haya cursado con enfermedad de Raynaud?• R = 80% ESCLERODERMIA, MIOSITIS, LES, AR.

• Cual es el síndrome doloroso regional o complejo tipo 1 distrofia simpática refleja?• R = DOLOR ARDOROSO O QUEMANTE de mas duración de lo esperado POR TRAUMATISMO EN EXTREMIDAD

secundario a aplastamiento o quemadura.

• Cuales son los datos clínicos de la distrofia simpática refleja?• R = DOLOR LOCAL, hiperestesia, calor, RIGIDEZ MUSCULAR, RIGIDEZ ARTICULAR, con consiguiente DESUSO Y

OSTEOPENIA RADIOLÓGICA

• Que tratamientos se utilizan en distrofia simpática refleja?• R = OPIOIDES Y GABAPENTINA

• Cual es el tipo mas común de síndrome de salida torácica?• R = NEUROGENICO 90%, venoso y arterial.

• En el síndrome de salida torácica , que es el síndrome de PAGET-SCHROETTER O TROMBOSIS DE ESFUERZO?• R = Se presenta como EDEMA UNILATERAL AGUDO DEL BRAZO, pesantez de axila, CIANOSIS DE LA MANO E

INGURGITACIÓN DE LAS VENAS DEL HOMBRO Y TÓRAX.

• Qué medidas se toman en un sujeto con choque que no responde a estímulos?• R = DEXTROSA AL 50%, naloxona 2 ml iv. Manteniendo la diuresis horaria mayor de .5

• Que te indica la PVC?• R = MENOR DE 5 mm hg indica HIPOVOLEMIA, MAYOR DE 18 SOBRECARGA VOLUMEN.

Page 129: ENARM  COMPENDIO

• Cuáles son los datos clínicos de taponamiento cardiaco?• R = Taquicardia, HIPOTENSIÓN, pulso paradójico, AUMENTO DE LA PVY. Excepto en pacientes urémicos o con

hipotiroidismo.

• -A 25-year-old man complains of left precordial chest pain that radiates to the left shoulder but not down the left arm. The pain is accentuated by inspiration and relieved by sitting up. The pain is accompanied by fever and chills. His blood pressure is 105/75 mm Hg, pulse 110/min and regular, and temperature 37.5°C. Aside from the tachycardia, there are no abnormal physical findings in the heart or lungs. The ECG shows STsegment elevation in all leads except aVR and VI. On the third hospital day, the patient’s blood pressure falls, JVP rises, and he goes into CHF. Which of the following is the most likely diagnosis?

• (A) a second pulmonary embolus• (B) extension of a myocardial infarct• (C) cardiac tamponade• (D) secondary bacterial infection• (E) rupture of a chordae tendineae• Respuesta correcta C

• -A 56-year-old man presents with SOB, fatigue, and edema. He has also noticed weight gain, abdominal discomfort, and distension. He has a prior history of lung cancer treated with radiotherapy to the chest. There is no history of liver or cardiac disease in the past. On examination, he has an elvated JVP, prominent y descent of neck veins, and positive Kussmaul’s sign. The heart sounds are normal. The CXR shows a normal cardiac silhouette and the ECG has low voltages.

• -A 55-year-old woman with metastatic lung cancer presents with dyspnea and pedal edema. On examination, the JVP is at 10 cm, with a negative Kussmaul’s sign. The heart sounds are diminished and the lungs have bibasilar crackles. The ECG shows QRS complexes of variable height

• Como afecta el cliostazol en la enfermedad arterial periférica?• R = Es un INHIBIDOR DE LA FOSFODIESTERASA 3 que disminuye la agregación plaquetaria, activa la lipoproteína

lipasa y causa vasodilatación.

Page 130: ENARM  COMPENDIO

• Cuál es el objetivo del tratamiento en un paciente con choque?• R = PVC 8-12, PA ½ 65-90, índice cardiaco 2-4 lts/min, y O2 venoso > 70%.

• Cuáles son los efectos de la dopamina?• R = 2-3 ug/kg/min estimula RECEPTORES DOPA y B AGONISTAS con AUMENTO DEL FG, FC Y IONOTROPISMO. >5 ug/kg/min estimula

receptores ALFA adrenergicos produciendo VASOCONSTRICCIÓN PERIFÉRICA.

• Cual es el medicamento de elección para choque cardiogenico?• R = DOBUTAMINA con aumento del ionotropismo y disminución de la poscarga

• Que efecto tiene la desmopresina utilizada en choque distributivo?1) Vasoconstricción periférica, disminución de la FC, vasodilatación coronaria, pulmonar y cerebral. 2) La vasoconstricción es mediada por oxido nítrico.

• Cual es la causa de la miocardiopatía hipertrófica?• R = DEFECTO CONGÉNITO en los sarcomeros.

• Cual es la causa de miocardiopatía dilatada?• R = Isquemia, cardiotoxicos. EMBARAZO.

• Cual es la causa de la miocardiopatía restrictiva?• R = Primaria: fiebre endomiocardica, endocarditis de coffler o ideopatica, Secundaria. RADIACION.

• -A 55-year-old woman is recently diagnosed with amyloidosis. She is now noticing increasing SOB, fatigue, and edema. On examination, the JVP is at 10 cm with a negative Kussmaul’s sign but prominent x and y descent. The blood pressure is 90/70 mm Hg, no pulsus paradoxus, pulse 100/min with low volume, and normal heart sounds.

• -A 60-year-old man presents with SOB, increasing abdominal distention, and lower leg edema. He has no prior history of cardiac, renal, or liver disease. On examination, the JVP is at 8 cm with a negative Kussmaul’s sign but prominent x and y descent. The blood pressure is 95/75 mm Hg, no pulsus paradoxus, pulse 100/min with low volume, and normal heart sounds. There is shifting dullness of the abdomen and pedal edema. His blood glucose and hemoglobin A1C are elevated.

Page 131: ENARM  COMPENDIO

• Que medicamentos están contraindicados en miocardiopatía restrictiva?• Digitalicos y agonistas B adrenérgicos.

• Cuales son los criterios para ENDOCARDITIS BACTERIANA DE DUKE.

• CRITERIOS MAYORES: • 1. Hemocultivos positivos para EI

1.1. Microorganismos típicos de EI en dos hemocultivos separados 1.1.1 ESTREPTOCOCO VIRIDANS S. BOVIS HACEK 1.1.2. S. Aureus o Enterococus adquiridos en la comunidad en ausencia de foco primario 1.2. Hemocultivos persistentes positivos 1.2.1. Hemocultivos extraidos con más de 12 horas de separación 1.2.2. La totalidad de tres, o la mayoría de cuatro o más hemocultivos separados siempre que entre el primero y el último haya al menos una hora

• 2. Evidencia de afectación miocárdica 2.1. Ecocardiograma positivo 2.1.1. VEGETACIÓN EN VÁLVULA O ESTRUCTURAS ADYACENTES o en el choque del jet, o sobre dispositivos protésicos en ausencia de otra explicación anatómica 2.1.2. Absceso 2.1.3. Nueva dehiscencia parcial de una válvula protésica 2.2. Nueva regurgitación valvular (incremento o cambio en un soplo preexistente no es suficiente)

Page 132: ENARM  COMPENDIO

• CRITERIOS MENORES 1. Predisposición. Una cardiopatía predisponente o ser ADVP. 2. FIEBRE > 38ºC 3. Fenómenos vasculares: émbolos en arterias mayores, infartos pulmonares, sépticos, aneurismas micóticos, hemorragia intracraneal, hemorragia conjuntival y LESIONES DE JANEWAY 4. Fenómenos inmunológicos (glomerulonefritis, NÓDULOS DE OSLER, MANCHAS DE ROTH Y factor reumatoide). 5. Ecocardiograma: sugestivos de eI sin alcanzar los criterios mayores antes mencionados. 6. Evidencia microbiológica (hemocultivos positivos que no cumplen los criterios mayores) o evidencia serológica de infección activa con un microorganismo que produce EI.

• -A 28-year-old man develops viridans group streptococci septicemia. Which of the following cardiac lesions has the highest risk of developing endocarditis?

• (A) ventricular septal defect• (B) atrial septal defect, secundum type• (C) mitral valve prolapse with regurgitation• (D) pure mitral stenosis• (E) asymmetric septal hypertrophy• Respuesta correcta A

• En que consiste la PERICARDITIS CONTSTRICTIVA?• R = Constrictive pericarditis is characterized by a prominent y descent of the neck veins and LOW VOLTAGE ON ECG. THE

PRESENCE OF A POSITIVE KUSSMAUL’S SIGN HELPS DIFFERENTIATE THE SYNDROME FROM COR PULMONALE AND RESTRICTIVE CARDIOMYOPATHIES. Apericardial knock is characteristic of constrictive pericarditis. It is in fact an early S3, occurring 0.06–0.12 seconds after aortic closure. S1 and S2 are frequently distant.

• -A 56-year-old man presents with SOB, fatigue, and edema. He has also noticed weight gain, abdominal discomfort, and distension. He has a prior history of lung cancer treated with radiotherapy to the chest. There is no history of liver or cardiac disease in the past. On examination, he has an elvated JVP, prominent y descent of neck veins, and positive Kussmaul’s sign. The heart sounds are normal. The CXR shows a normal cardiac silhouette and the ECG has low voltages.

Page 133: ENARM  COMPENDIO

• -A 64-year-old presents with dyspnea and edema. He had previous coronary bypass surgery 5 years ago, which was uncomplicated. Since then he has had no further chest pain. On examination, his JVP is at 8 cm, with prominent Kussmaul’s sign. The heart sounds are easily heard but there is an early diastolic filling sound (pericardial knock).

• Cuál es el diagnostico diferencial entre miocardiopatia constrictiva y pericarditis restrictiva?

1) PERICARDITIS RESTRICTIVA: Puede ser causada por tuberculosis, Rx del tórax y cirugía cardiaca.

2) PERICARDITIS CONSTRICTIVA: Causada por un tamponade.3) MIOCARDIOPATIA CONSTRICTIVA: Lo causa la amiloidosis, hemocromatosis,

sarcoidiosis, esclerodermia, carcinoide.

Page 134: ENARM  COMPENDIO

REUMATOLOGIA

ENARM

Page 135: ENARM  COMPENDIO
Page 136: ENARM  COMPENDIO

ARTRITIS CRONICA JUVENIL

Page 137: ENARM  COMPENDIO
Page 138: ENARM  COMPENDIO
Page 139: ENARM  COMPENDIO
Page 140: ENARM  COMPENDIO
Page 141: ENARM  COMPENDIO

ESPONDILITIS ANQUILOSANTE• Que espondiloartropatias se asocian al HLA-B27? R = ESPONDILITIS ALQUILOSANTE, ARTRITIS REACTIVA, ESPONDILOARTROSIS PSORIASICA,

uveítis anterior aguda.

• Cual es el síntoma inicial de la espondilitis alquilosante (EA) y que hallazgos son frecuentes en la ex fis? R = DOLOR en regiones SACROILIACAS Y LUMBARES. En la ex fis HAY DISMINUCIÓN DE LA MOVILIDAD DE

LA COLUMNA LUMBAR, dolor al presionar las regiones sacroiliacas y disminución de la expansión del tórax.

• Cual es la manifestación extrarticular mas frecuente de la EA? R = UVEITIS ANTERIOR, 2/3 de px tiene CAMBIOS EN LA MUCOSA Y SUBMUCOSA DEL COLON E ÍLEON

TERMINAL

• Cuales son los hallazgos más comunes en imagen obtenidas en la espondilitis anquilosante? R = Erosiones y esclerosis sacroiliacas, IMAGEN EN COLUMNA DE BAMBU

• Cual es el manejo de la EA? R = AINES, COX-2, Infliximab, Etanercept mas terapia física. Indometacina, INFLIXIMAB que disminuye el FNTa.

Page 142: ENARM  COMPENDIO
Page 143: ENARM  COMPENDIO

ENFERMEDAD DE REITER• Cuales son los datos clinicos más comunes de ARTRITIS REACTIVA o

enfermedad de Reiter y con que HLA se relaciona? R = Ligado al HLA B27, oligoartritis, CONJUNTIVITIS, URETRITIS,

CERVICITIS, ULCERAS ORALES x un CUADRO INFECCIOSO PREVIO.

• Que situación precede a una manifestación de ARTRITIS REACTIVA o síndrome de Reiter y cual es el patógeno > común?

R = Alguna INFECCIÓN por SALMONELLA, SHIGELLA, YERSINIA,

CLAMYDIA TRACHOMATIS +++ o CAMPYLOBACTER que PRECEDE a la MANIFESTACIÓN ARTICULAR 3-4 semanas después.

• Cual es el estudio de laboratorio o gabinete de elección del síndrome

de Reiter? R = CULTIVO CON ANTIBIOGRAMA, se puede encontrar también Ac anti

bacteriano en el suero o liquido sinovial o DNA bacteriano.

Page 144: ENARM  COMPENDIO

ENFERMEDAD DE STILL• Que caracteriza clínicamente a la enfermedad de Still y que la precede?

R = VARIANTE DE LA ARTRITIS REUMATOIDE en la cual hay FIEBRE DE MAS DE 40

y con precipitación posterior varios grados debajo, ODINOFAGIA Y POLIADENOPATIAS. LO PRECEDE FARINGITIS.

• Cual es una manifestación clínica característica de la enfermedad de Still?

R = EXANTEMA NO PRURIGINOSO, maculopapular de COLOR SALMÓN en TÓRAX Y ABDOMEN. (EL CUAL NO HAY EN REITER)

• Cuales son los laboratorios distintivos de la enfermedad de Still, que inmunoglobulina se eleva y como afecta al FR?

R = LEUCOCITOSIS > 40, VSG AUMENTADA, hipergamaglobulinemia IgG y anemia. NO se detectan AUTOANTICUERPOS NI FR.

• Cual es el manejo de la enfermedad de Still? R = AINES

Page 145: ENARM  COMPENDIO

ARTRITIS SORIASICA• Que es la artritis soriasica? R = Ligado al HLA B27. Se define como una ARTRITIS usualmente

SERONEGATIVA que se ASOCIA A SORIASIS.

• Cual es el cuadro clínico de la artritis soriasica? R = PRECEDE PSORIASIS A ARTRITIS, ARTRITIS ASIMÉTRICA con DEDOS EN

APARIENCIA DE SALCHICHA (DACTILITIS) de los dedos de manos y pies.

• Como diagnosticas artritis soriasica? R = HC, y RX con evidencia de SACROILEITIS ASIMÉTRICA, alteraciones de la

conformación de hueso nuevo con IMAGEN DE PUNTA DE LÁPIZ EN UNA COPA.

• Cual es el manejo de la artritis soriasica? R = AINES E INFLIXIMAB. Tratamiento de soriasis.

Page 146: ENARM  COMPENDIO

RESUMEN ESPONDILOARTROPATIAS SERONEGATIVAS

Page 147: ENARM  COMPENDIO

LUPUS ERITEMATOSO SISTEMICO• Que HLA tienen relación con el LES?

1) HLA B8, HLA DR2 y DR3.

2) Los alelos de HLA están en el brazo corto del CROMOSOMA 6.

• Que fármacos inducen la producción de anticuerpos antinucleares y LES? R = HIDRALAZINA, procainamida, clorpromazina, ISONIAZIDA y diversos anticonvulsivantes.

• Cual es el cc del LES en cada aparato – sistema y con que padecimientos se relaciona? 1. Se relaciona con: PTI, ANEMIA HEMOLITICA Y TIROIDITIS. 2. Constitucionales: la FATIGA es lo mas común 3. Mucocutaneas: 80%, LESIONES DISCOIDES observándose en cara, pabellones auriculares y el AREA DE

V DEL ESCOTE. Eritema malar o en ALAS DE MARIPOSA QUE ES FOTOSENSIBLE. EL neonatal QUE SE PRESENTA EN LOS HIJOS DE PX CON LES POR EL PASO DE IgG CARACTERIZÁNDOSE POR LESIONES CUTÁNEAS Y BLOQUEO CARDIACO CONGÉNITO EN LOS PRIMEROS 6 MESES DE EDAD. ALOPECIA.

4. Musculoesqueletico: ARTRALGIAS con artritis. Osteonecrosis en la cabeza femoral, humeral, cóndilos femorales, carpo, tarso manifestándose clínicamente con dolor que mejora con el reposo.

5. Renal: nefritis lupica siendo la mas común la TIPO IV ¨GLOMERULONEFRITIS PROLIFERATIVA DIFUSA¨. 6. SNC: Meningitis aséptica, EVC, síndromes desmielinizantes y cefaleas. 7. Cardiopulmonar: En corazón con VEGETACIONES ENDOCARDICAS de LIBMAN-SACKS, en pleura lo

mas común es pleuritis con o sin derrame pleural con niveles altos de proteínas DHL, complejos inmunes, ANA y células LE. LA VALVULA MAS AFECTADA ES LA MITRAL.

8. Gastrointestinal: Se puede presentar con SEROSITIS, disfagia y dispepsia muy a menudo. 9. Hematológico: PANCITOPENIA Y >TTP. Anemia por deficiencia de hierro MICROCITICA-HIPOCROMICA.

Page 148: ENARM  COMPENDIO
Page 149: ENARM  COMPENDIO

ESCLERODERMIA

Page 150: ENARM  COMPENDIO
Page 151: ENARM  COMPENDIO
Page 152: ENARM  COMPENDIO

SX SJOGREN

Page 153: ENARM  COMPENDIO

POLIARTERITIS NODOSA• Cuales son los signos y síntomas MAS COMUNES de poliarteritis nodosa? R = DOLOR EN EXTREMIDAD POR ARTRALGIA, MIALGIA QUE AFECTA PANTORRILLAS, puede

afectar riñón causando HAS, IRA O HEMORRAGIA POR MICROANEURISMAS. MONONEURITIS MÚLTIPLE (PIE CAÍDO), corazón con IAM. Livides reticular, nódulos subcutáneos, y ulceras en piel. Son necesarios 3 de 10 criterios para su diagnostico.

• Cuales son las pruebas de laboratorio en la poliarteritis nodosa y con que otro virus se relaciona habitualmente?

R = DESCARTAR VHB QUE SE ASOCIA EN 20-30%. ANCA (-) Y ANA (-) .Anemia, leucocitosis y trombocitosis. Se confirma diagnostico con toma de BIOPSIA DE LA LESIÓN.

• Que datos se obtienen al tomar la biopsia en la poliarteritis nodosa? R = BIOPSIA DE ARTERIA de mediano o pequeño calibre con PRESENCIA DE INFILTRADO

GRANULOCITICO, PMN Y CÉLULAS MONONUCLEARES EN LA PARED DEL VASO.

• Cual es el manejo de la poliarteritis nodosa? R = ESTEROIDES, pero SI SE ASOCIA CON VHB SE AGREGAN RETROVIRALES.

Page 154: ENARM  COMPENDIO

ARTERITIS DE LA TEMPORAL

Page 155: ENARM  COMPENDIO
Page 156: ENARM  COMPENDIO
Page 157: ENARM  COMPENDIO

VASCULITIS

Page 158: ENARM  COMPENDIO

ARTRITIS SEPTICA PIOGENA• Cuales son los factores de riesgo para la artritis séptica piógena (gonocócica o no)? R = Abuso de drogas IV, jóvenes sexualmente activos, enfermedades concomitante como DM, AR o lupus,

antecedentes de cirugía o prótesis.

• Cual es el cc de la artritis séptica piógena gonocócica o no? R = Es una entidad de inicio agudo y progresivo caracterizado por dolor e inflamación local (salvo en el primer año de

vida donde es similar a la sepsis). El dolor es moderado a intenso con limitación funcional, aumento de la temperatura local y rubor es monoarticular, cuando es poliarticular es en paciente inmunocomprometido siendo la rodilla la mas afectada.

• Cuales son los hallazgos radiológicos en la artritis séptica piógena? R = Edema de tejidos blandos y derrame articular. EN AFECCIÓN POR ANAEROBIOS SE VE GAS EN TEJIDOS BLANDOS.

El gamagrama óseo con tecnecio (valora flujo) o con galio (valora actividad inflamatoria) pueden utilizarse de manera temprana.

• Cual es el manejo de la artritis séptica piógena?1) Consiste en la administración de antibióticos y drenaje de la articulación afectada. En caso de ARTRITIS PIÓGENA

debe administrarse de 3 A 4 SEMANAS y en caso de ARTRITIS GONOCÓCICA DE 7 A 14 DÍAS.2) En menores de 2M: DICLOXACILINA mas CEFOTAXIMA3) En LACTANTES: DICLOXACILINA mas CEFOTAXIMA4) En niños MAYORES DE 3ª: DICLOXACILINA5) Artritis séptica GONOCÓCICA: 1g por via IM o IV cada 24 hrs.6) Artritis séptica en DROGADICTOS por via IV: DICLOXACILINA mas GENTAMICINA7) Artritis séptica en INMUNOCOMPROMETIDOS: CEFTAZIDIMA mas AMIKACINA8) Artritis séptica POSTOPERATORIA: VANCOMICINA mas AMIKACINA9) Artitis séptica en FRACTURA ABIERTA: AMOXICILINA con ACIDO CLAVULANICA

Page 159: ENARM  COMPENDIO

ARTRITIS SEPTICA PIOGENA

Page 160: ENARM  COMPENDIO

RESUMEN DE LAS ENFERMEDADES METABOLICAS OSEAS

Page 161: ENARM  COMPENDIO

CPK

• Las siguientes patologías cursan con aumento de la CPK

1) HIPOTIROIDISMO2) RAZA NEGRA3) POLIMIOSITIS Y4) DERMATOMIOSITIS

Page 162: ENARM  COMPENDIO

GASTROENTEROLOGIA

ENARM

Page 163: ENARM  COMPENDIO
Page 164: ENARM  COMPENDIO

ACALASIA• Que es la acalasia? R = Es un trastorno por motilidad inadecuada que se caracteriza por una RELAJACIÓN INCOMPLETA y AUMENTO

DE LA PRESIÓN BASAL DEL EEI, además de presentar APERISTALSIS DEL CUERPO.

• Que causa la acalasia? R = Es IDIOPÁTICO, aunque una de las hipótesis indica que es de origen central con DESTRUCCIÓN PARCIAL DEL

NÚCLEO DORSAL DEL VAGO o sus fibras nerviosas a nivel del esófago con la siguiente alteración de la función.

• Cuales son los síntomas más comunes de la acalasia? R = DISFAGIA EN EL 90% DE LOS CASOS, INICIALMENTE A SÓLIDOS Y DESPUÉS A LÍQUIDOS. El segundo

síntoma mas frecuente es la REGURGITACIÓN EN 75% mas acentuada en decúbito, EL DOLOR 20-30% ES MENOS COMÚN, DE LOCALIZACIÓN EPIGÁSTRICA, RETROESTERNAL, E IRRADIADO HACIA CUELLO, DORSO Y EXTREMIDADES SUPERIORES.

• Cuales son los estudios de gabinete necesarios para el diagnostico de acalasia? R = La MANOMETRIA ES DE ELECCIÓN la cual nos muestra una PRESIÓN DEL EEI AUMENTADA (>69 MM/HG),

SIN PRESENCIA DE ONDAS PERISTÁLTICAS A LA DEGLUCIÓN. La ESOFAGOGRAFIA CON BARIO MUESTRA ALTERACIONES RELACIONADAS CON LAS ONDAS PERISTÁLTICAS Y LA AUSENCIA DE RELAJACIÓN DEL EEI DANDO UNA IMAGEN EN PICO DE PÁJARO O EN PUNTA DE LÁPIZ característica.

• Cual es el manejo de la acalasia?1) MEDICO: A base de DILATACIONES HIDROSTÁTICAS O NEUMÁTICA, además de LA INYECCIÓN CON TOXINA

BOTULINICA.

2) Quirúrgico: MIOTOMIA DE HELLER con una tasa de curación del 95%.

Page 165: ENARM  COMPENDIO

ESPASMO ESOFAGICO DIFUSO• Que es el espasmo esofágico difuso? R = Se trata de un trastorno motor esofágico que afecta fundamentalmente a la musculatura lisa y que se caracteriza por LA

APARICIÓN DE ACTIVIDAD CONTRÁCTIL NO PROPULSIVA que alterna con episodios de peristalsis normal.

• Cual es el cuadro clínico del espasmo esofágico difuso? R = DOLOR TORAXICO CON LOCALIZACIÓN fundamentalmente RETROESTERNAL, aunque a veces SE IRRADIA A CUELLO Y A

LOS HOMBROS SIMULANDO UNA ENFERMEDAD CARDIACA aumentando cuando están bajo estrés sicológico o emocional. La

DISFAGIA es tan frecuente o mas que el dolor torácico. También es intermitente SE DESENCADENA CON FRECUENCIA CON LA TOMA DE SÓLIDOS O BEBIDAS FRÍAS.

• Cuales son las alteraciones histológicas que esperas encontrar en el espasmo esofágico difuso? R = HIPERPLASIA DE LAS CÉLULAS musculares y ocasionalmente mínimas ALTERACIONES a nivel de las fibras nerviosas terminales

del PLEXO DE AUERBACH, no existiendo alteraciones significativas en las células ganglionares y nervios o en todo caso estas son mínimas.

• Cuales son los hallazgos al gabinete en el espasmo esofágico difuso? R = ESOFAGOGRAMA: Que nos muestra hallazgos característicos con APARICIÓN DE FRECUENTES ONDAS NO PROPULSIVAS en el

musculo liso esofágico QUE INDENTAN LA COLUMNA DE BARIO y retrasan su evacuación, lo cual se describe como ESÓFAGO EN SACACORCHOS. MANOMETRIA: Alteraciones segmentarias, apareciendo generalmente en los DOS TERCIOS INFERIORES DEL CUERPO ESOFÁGICO. PHmetria obligatoria para descartar ERGE como causa del espasmo.

• Cual es el manejo MEDICO Y QUIRURGICO del espasmo esofágico difuso?

• MEDICO: NIFEDIPINO, ANTIDEPRESIVOS Y ANSIOLÍTICOS. • QUIRURGICO: La ESOFAGOMIOTOMIA AMPLIADA hasta el cayado aórtico puede ser una alternativa muy acertada EN CASOS

GRAVES REFRACTARIOS a cualquier otro tratamiento.

Page 166: ENARM  COMPENDIO
Page 167: ENARM  COMPENDIO

PERISTALSIS ESOFÁGICA SINTOMÁTICA O ESÓFAGO EN CASCANUECES• Que es la peristalsis esofágica sintomática o esófago en cascanueces y con que se relaciona? R = Trastorno de la motilidad esofágica que se caracteriza por la PRESENCIA DE CONTRACCIONES DE

GRAN AMPLITUD, SIN ALTERACIONES EN EL PERISTALTISMO del cuerpo esofágico y relajación del esfínter esofágico inferior. Se RELACIONA CON ERGE y a la histología no presenta alteraciones.

• Cual es el cuadro clínico de la peristalsis esofágica sintomática o esófago en cascanueces? R = DOLOR TORÁCICO RECURRENTE y con menor frecuencia disfagia.

• Cuales son los hallazgos de gabinete en la peristalsis esofágica sintomática o esófago en cascanueces en la manometria o en el esofagograma?

R = El diagnostico se establece por la MANOMETRIA y se caracteriza por la aparición de ONDAS DE GRAN AMPLITUD (180 MM/HG), QUE OCASIONALMENTE TAMBIÉN SON DE LARGA DURACIÓN (6SEG), siendo condición indispensable que EL PERISTALTISMO ESOFÁGICO SEA NORMAL. Siempre se debe descartar ERGE realizando una PHmetria. NO HALLAZGOS CON ESOFAGOGRAMA.

• Cual es el manejo de la peristalsis esofágica sintomática o esófago en cascanueces?1) MEDICO: Si existe ERGE se recomiendo uso de OMEPRAZOL y evitar MIORRELAJANTES (nifedipino,

diltiazem o nitratos) POR QUE EMPEORARÍAN EL ERGE. BENZODIAZEPINAS Y SOBRE TODO LOS ANTIDEPRESIVOS TRICICLICOS parecen útiles.

2) QUIRURGICO: ESOFAGOMIOTOMIA AMPLIADA HASTA EL CAYADO AÓRTICO puede ser una alternativa en pacientes intratables. La inyección de toxina botulinica tiene resultados muy pobres.

Page 168: ENARM  COMPENDIO

EEI HIPERTENSO• Como se define el EEI hipertenso? R = Como la existencia DE UN TONO O PRESIÓN media máxima basal DEL EEI

IGUAL O SUPERIOR A DOS DEVIACIONES ESTÁNDAR, mas de 40-45 mm/Hg, siendo además condicionantes indispensables que existan RELAJACIONES COMPLETAS DEL EEI y que EL PERISTALTISMO ESOFÁGICO SEA NORMAL.

• Cual es el cuadro clínico del EEI hipertenso? R = DOLOR TORÁCICO O DISFAGIA. Mejorando generalmente el dolor tras el

tratamiento, siendo la disfagia la mas difícil de tratar.

• Cual es el manejo del EEI hipertenso? R = RELAJANTES DE LA MUSCULATURA LISA

Page 169: ENARM  COMPENDIO
Page 170: ENARM  COMPENDIO
Page 171: ENARM  COMPENDIO

SX DUMPING

Page 172: ENARM  COMPENDIO

CÁNCER GÁSTRICO• Cual es la estirpe etiológica mas común del cáncer gástrico? R = ADENOCARCINOMA.

• Cual es el cuadro clínico de un paciente con cáncer gástrico? R = PERDIDA DE PESO, dolor abdominal, SACIEDAD TEMPRANA. Generalmente se presentan con

palidez. Entre las manifestaciones dermatológicas mas frecuentes se encuentran la ACANTOSIS NIGRICANS y SIGNO DE LESER-TRELAT (queratosis seborreica de aparición súbita)

• Como se manifiesta la metastasis comúnmente el cáncer gástrico? R = GANGLIOS EN SUPRACLAVICULAR (Virchow), GANGLIOS PERIUMBILICALES (Hermana Maria

Jose), estructuras rectales (signo del anaquel de blúmer), OVARIOS (tumor de Krukenberg).

• Cual es el estándar de oro para el diagnostico del cáncer gástrico?

R = PANENDOSCOPIA con toma de 7 BIOPSIAS de la lesión y después TAC PARA ESTADIFICACION en caso de metástasis.

• Cual es el manejo del cáncer gástrico? R = Quirúrgico. La gastrectomía total se recomienda en caso de lesiones en el estomago proximal.

La QUIMIOTERAPIA no mejora la sobrevida sobre la cirugía, sin embargo COMBINADA CON LA RADIOTERAPIA si la mejora.

Page 173: ENARM  COMPENDIO

LINFOMA GASTRICO• 60. Cual es la estirpe etiológica mas común en el linfoma gástrico? R = No HODKING 95%

• 61. Cual es el cuadro clínico del linfoma gástrico? R = Sintomatología no especifica como DOLOR ABDOMINAL, nausea,

vomito, PERDIDA DE PESO, SUDORES NOCTURNOS y diarrea.

• 62. Cual es la apariencia del linfoma gástrico a la endoscopia? R = Aparecen como LESIONES FUNGOIDES, PÓLIPOS, ENGROSAMIENTO DE

PLIEGUES.

• 63. Cual es el manejo del linfoma gástrico? R = Quirúrgico

Page 174: ENARM  COMPENDIO
Page 175: ENARM  COMPENDIO
Page 176: ENARM  COMPENDIO

MALABSORCION• Cuales son los datos clínicos de malabsorción? R = Diarrea, DISMINUCIÓN DE PESO y desnutrición, acompañados de

DISTENCIÓN ABDOMINAL, síntomas y signos secundaros a los déficit específicos, ESTEATORREA.

• Que datos de laboratorio se encuentran en un paciente con malabsorción? R = Hipocolesterolemia, HIPOALBUMINEMIA, alteraciones electrolíticas y PH.

• Cual es la posología de la prueba con D-Xilosa en malabsorción? R = Se da EN AYUNAS midiendo su nivel en suero 1hr después y la excreción en

orina en 6hr, este azúcar SE ABSORBE EN FORMA PASIVA SI LA MUCOSA INTESTINAL ESTA INTEGRA.

• Cuales son las principales causas de malabsorción? R = DEFICIENCIA DE LACTASA, ESPRUE celiaco, ESPRUE TROPICAL, ENFERMEDAD

DE WIPPLE, sobre crecimiento bacteriano y síndrome de intestino corto.

Page 177: ENARM  COMPENDIO

ESPRUE CELIACO• Que es el esprue celiaco? R = Causa malabsorción debido a lesión ocasionada por el GLUTEN a nivel intestinal, el cual es

contenido en el trigo, CEBADA, CENTENO Y AVENA.

• Cual es el cuadro clínico del esprue celiaco? R = Hay malabsorción de CHBTS, PROTEÍNAS Y GRASAS. Perdida progresiva de peso, distensión

abdominal, flatulencia y diarrea, evacuaciones voluminosas y de mal olor.

• Cuales estudios de diagnostico hay para el esprue celiaco y cuales son los anticuerpos especificos?

1) La biopsia en LA UNIÓN DUODENOYUYENAL que suele REPORTAR ATROFIA O ACORTAMIENTO DE LAS VELLOSIDADES, HIPERPLASIA DE LAS CRIPTAS y aumento en la mitosis CON INCREMENTO DE LINFOCITOS, NEUTROFILOS, MASTOCITOS Y EOSINOFILOS EN LAMINA PROPIA.

2) Cuantificación de grasa en heces de 24 hrs mayor a 7g/dia , la ABSORCIÓN DE D-XILOSA SUELE SER ANORMAL, los ANTICUERPOS SÉRICOS ANTIGLIADINA SON POSITIVOS EN 90%, el anticuerpo IgA es el mas especifico.

3) Los anticuerpos IgA ANTIENDOMISIO Y ANTIRRETICULINA son los MAS ESPECÍFICOS.

• Cual es el manejo del esprue celiaco? R = Dieta libre de gluten

Page 178: ENARM  COMPENDIO

ESPRUE TROPICAL• Cual es la etiología del esprue tropical? R = Es producido por la COLONIZACIÓN PERSISTENTE del intestino delgado por KLEBSIELLA

PNEUMONIAE, ENTEROBACTER CLOACAE o E. COLI.

• Cual es el cuadro clínico del esprue tropical?1) CRISIS DE DIARREA ACUOSA, acompañada de DOLOR ABDOMINAL y FLATULENCIA, los cuales

disminuyen de intensidad a la semana de evolución y posteriormente se hacen crónicos.2) ANEMIA MEGALOBLASTICA.

• Cuales son los estudios diagnósticos del esprue tropical?1) La biopsia de intestino delgado reporta ALARGAMIENTO DE LAS CRIPTAS, ensanchamiento y

acortamiento de las vellosidades e INFILTRACIÓN DE LA LAMINA PROPIA POR CÉLULAS INFLAMATORIAS.

2) Se encuentra esteatorrea, ABSORCIÓN DE D-XILOSA ANORMAL.

• Cual es el manejo del esprue tropical?

R = TETRACICLINA y ACIDO FÓLICO para mejorar ANEMIA MEGALOBLASTICA.

Page 179: ENARM  COMPENDIO

ENFERMEDAD DE WHIPPLE• Cual es el agente causal de la enfermedad de Whipple y con que HLA se relaciona? R = Es secundaria a la infección por TROPHERYMA WHIPELLII y se relaciona con HLA B27 + en 25%.

• Cual es el cuadro clínico de la enfermedad de Whipple?

R = Perdida de peso, diarrea o esteatorrea, artralgias, FIEBRE, dolor abdominal y HEMORRAGIA INTESTINAL, LAS ARTRALGIAS SON MIGRATORIAS, no destructivas y afectan las grandes articulaciones,

se presenta SACROILEITIS EN 25%. Un signo característico es la MIORRITMIA OCULOMASTICATORIA, la cual consiste en la convergencia rítmica de los ojos asociada a contracciones

sincrónicas de los músculos de la masticación. A nivel ocular se manifiesta como UVEÍTIS, vitreitis, retinitis y

papiledema. A la ex fis se encuentra LINFADENOPATIA periférica.

• Cuales son los hallazgos de laboratorio en la enfermedad de Whipple?

R = La biopsia de intestino delgado muestra MACRÓFAGOS POSITIVOS CON LA TINCIÓN PAS. Sin embargo se debe demostrar el AGENTE CAUSAL EN EL LCR, tejido cerebral o GANGLIOS LINFÁTICOS. Anemia secundaria a enfermedad crónica, deficiencia de hierro, folato o VB12, asociada a neutrofilia. Prueba de D-Xilosa anormal,

LINFADENOPATIA ABDOMINAL detectada por ultrasonido. Radiografía de TORAX CON INFILTRADOS, FIBROSIS O DERRAME PLEURAL.

• Cual es el manejo de la enfermedad de Whipple y por cuanto tiempo se realiza? R = TMP/SMZ durante 1ª, si hay ALERGIA A SULFAMIDAS esta indicada la PENICILINA por vía oral durante el

mismo periodo y se deben tratar los trastornos nutricios.

Page 180: ENARM  COMPENDIO
Page 181: ENARM  COMPENDIO
Page 182: ENARM  COMPENDIO

SX COLON IRRITABLE• Cuales son los criterios de ROMA II para integrar el síndrome de colon irritable? R = Que los síntomas hallan durado POR LO MENOS 12 SEMANAS (no necesariamente continuas)

durante los 12m previos de malestar o dolor abdominal que tiene 2 de las siguientes características. Se dice que el síndrome de intestino irritable se diagnostica cuando cumple al menos con 1 o mas de los criterios 2, 4 y 6 o bien 1, 3 y 5 DEPENDIENDO DE SI ES DIARREA LA QUE PREDOMINA O CONSTIPACIÓN. Se exacerba con el estrés y es MAS FRECUENTE EN MUJERES.

1. Menos de 3 evacuaciones por semana2. Mas de 3 evacuaciones al día3. Evacuaciones de consistencia dura4. Evacuaciones sueltas o duras5. Pujo al evacuar6. Urgencia al evacuar7. Sensación de evacuación completa8. Moco durante la evacuación9. Distención abdominal

• Cual es el manejo del síndrome de colon irritable? R = Debe ser INDIVIDUALIZADO a los síntomas. PInaverio con dimeticona.

Page 183: ENARM  COMPENDIO

POLIPOS DEL COLON• Cual es la clasificación de los pólipos neoplásicos de colon?

R = A) TUBULARES 75% que corresponde a ADENOMAS. B) VELLOSOS 10% de los adenomas, mas relacionado con cáncer colonico y C) TUBULOVELLOSOS 15% de los adenomas. Requiriéndose un periodo mínimo de 10ª para que un adenoma se convierta a cáncer.

• Cuales son las alteraciones genéticas implicadas en los adenomas y el cáncer de colon? R = Mutación del GEN RAS y lesiones de los cromosomas 5, 17 y 18.

• Cual es el cuadro clínico de un paciente con SÍNDROME DE PEUTZ-JEGHERS? R = Es un TRASTORNO AD caracterizado POR MANCHAS HIPERPIGMENTADAS en los labios, mucosa bucal, cara y dedos,

PRESENTANDO HAMARTROMAS EN TODO EL TUBO DIGESTIVO e INTUSUCEPCION CON STDB.

• Cual es el manejo del Síndrome de Peutz-Jeghers? R = Resección de pólipo sintomático por endoscopia

• Cual es el cuadro clínico de la poliposis juvenil difusa? R = Es un TRASTORNO AD caracterizado por una población homogénea de pólipos, tanto hamartromas como adenomas,

los pólipos pueden ocasionar hemorragia e intususcepción en este síndrome. 10% RIESGO DE DESARROLLO DE CA DE COLON.

• Cual es el manejo de la poliposis juvenil difusa? R = RESECCIÓN del colon con y anastomosis ileorectal. SEGUIMIENTO CON PROCTOSCOPIA CADA 6M y extirpación de

cualquier pólipo nuevo.

Page 184: ENARM  COMPENDIO

PEUTZ-JEGHERS

Page 185: ENARM  COMPENDIO

• Cual es el cuadro clínico del SÍNDROME DE COWDEN?

1) Enfermedad AD caracterizada por HAMARTROMAS en todo el tubo digestivo, PAPULAS FACIALES Y BUCALES, CRECIMIENTOS QUERATOSICOS EN MANOS Y PIES.

2) Estos pacientes pueden presentar cáncer de mama, tiroides o de útero.

• Cual es el cuadro clínico del SÍNDROME DE CRONKHITE-CANADA? R = NO ES HEREDITARIO y se caracteriza por HAMARTROMAS INTESTINALES generalizados, ALOPECIA,

PIGMENTACIÓN CUTÁNEA y ATROFIA DE NAILS. Los SÍNTOMAS INCLUYEN VOMITO, DIARREA, MALABSORCIÓN y enteropatía con perdida de proteínas, LA MAYORÍA DE LOS PACIENTES MUERE POCO DESPUÉS DEL DIAGNOSTICO.

• Cual es el cuadro clínico de la poliposis adenomatosa familiar? R = TRASTORNO AD y su defecto se localiza en el CROMOSOMA 5. Se caracteriza por la presencia de mas de 100

POLIPOS en todo el colon y recto, 100% DESARROLLA CÁNCER.

• Cual es el manejo de la poliposis adenomatosa familiar? R = Resección con RECTOCOLECTOMIA total con ileostomía.

• Cual es el cuadro clínico de la ENFERMEDAD DE GARDNER? R = POLIPOSIS ADENOMATOSA familiar que se asocia a OSTEOMIELITIS, QUISTES EPIDERMOIDES Y FIBROMAS DE

LA PIEL.

• Que es el SÍNDROME DE TURCOT? R = POLIPOSIS ADENOMATOSA FAMILIAR que se asocia con NEOPLASIAS DEL SNC.

• Como se manifiesta un Adenocarcinoma de colon ubicado en el lado derecho? R = Perdida de peso, HEMATOQUZIA, anemia por déficit de hierro y UNA MASA ABDOMINAL EN EL CID.

• Como se manifiesta un Adenocarcinoma de colon ubicado en el lado izquierdo? R = Perdida de peso, cambios en hábitos intestinales, RECTORRAGIA y dolor abdominal tipo cólico. Se debe realizar

una colonoscopia con biopsia.

Page 186: ENARM  COMPENDIO

COWDEN

Page 187: ENARM  COMPENDIO

Cronkhite-Canada

Page 188: ENARM  COMPENDIO

GARDNER

Page 189: ENARM  COMPENDIO
Page 190: ENARM  COMPENDIO
Page 191: ENARM  COMPENDIO
Page 192: ENARM  COMPENDIO

TROMBOSIS MESENTERICA• Que pacientes tienen el riesgo de desarrollo de trombosis venosa

mesentérica? R = Pacientes traumatizados, hipertensión portal, peritonitis o con

hipercoagubilidad

• Cual es el cuadro clínico de la trombosis venosa mesentérica? R = DOLOR ABDOMINAL DIFUSO, DISTENCIÓN ABDOMINAL, nauseas,

vómitos y signos de deshidratación

• Cual es el estándar de oro para el dx de la trombosis venosa mesentérica? R = TAC EN ESPIRAL con medio de contraste oral e IV

• Cual es el manejo de la TVM? R = ANTI COAGULACIÓN E HIDRATACIÓN.

Page 193: ENARM  COMPENDIO

STDB

Page 194: ENARM  COMPENDIO

TIPS STDA Y STDB

o Los pacientes que requieran mas de 10u de paquetes globulares deberán recibir también PLASMA FRESCO CONGELADO, PLAQUETAS O AMBOS.

o La colocación de BANDAS ELÁSTICAS es el tratamiento de elección en la rotura de VARICES ESOFÁGICAS.

o El LAVADO NASO GÁSTRICO se utiliza para DIFERENCIAR entre el STDA Y STDB.

o La POSITIVIDAD EN EL ASPIRADO del tubo naso gástrico, RUIDOS INTESTINALES HIPERACTIVOS y elevación de los niveles del BUN apoyan el diagnostico de STDA.

o En la hematemesis secundaria a la ingesta de AINES se debe realizar una esofagogastroscopia y administrar un inhibidor de la bomba.

o El hematoma retroperitoneal produce una IMAGEN DE VIDRIO DESPULIDO en la radiografía de abdomen.

Page 195: ENARM  COMPENDIO

ABSCESO ANAL• Cuales son las causas de absceso anorrectal especificas y no especificas? R = ESPECIFICAS: Chron, CUCI, Tb, actinomicosis, ca, leucemias. NO

ESPECIFICAS: Infección bacteriana de glándulas anales

• Cual es la complicación principal que sucede al drenar un absceso anal? R = Posterior formación de FISTULAS

• Cual es el cuadro clínico en un px que presenta un absceso anal? R = DOLOR ANAL INTENSO o una TUMORACIÓN ANAL MUY DOLOROSA

que apareció en forma reciente.

• Cual es el manejo de un absceso perianal? R = DRENAJE Y METRONIDAZOL O CIPROFLOXACINO.

Page 196: ENARM  COMPENDIO

TUMORES DEL ANO

Page 197: ENARM  COMPENDIO

FISTULA ANAL

• Cual es la etiología de una fistula anal? R = Abscesos crónicos, CHRON Y CUCI.

• Cual es el cuadro clínico de las fistulas anales? R = Es característico la historia de un ABSCESO PREVIAMENTE DRENADO en la

misma localización, DOLOR PERIANAL, la principal molestia es la PERMANENTE SALIDA DE MATERIAL PURULENTO a través de un orificio que MANCHA LA ROPA INTERIOR.

• Cual es el estudio de imagen utilizado para el diagnostico de las fistulas anales? R = ULTRASONIDO RECTAL

• Cual es el manejo de las fistulas anales? R = La forma para predecir por donde drena la fistula es la regla de GOODSALL-

SALMON y una vez identificada se realiza FISTULOTOMIA.

Page 198: ENARM  COMPENDIO

FISURA ANAL

• Cual es la etiología de las fisuras anales? R = ISQUEMIA EN MUCOSA POR ESFÍNTER HIPERTÓNICO.

• Cual es el cuadro clínico de las fisuras anales? R = DOLOR intenso TIPO ARDOROSO que se relaciona al inicio de la

evacuación con una SENSACIÓN DE DESGARRO al pasar el bolo fecal. SANGRADO FRECUENTE, suele ser en POCA CANTIDAD.

• Cual es el manejo de las fisuras anales? R = Baños de asiento, laxante, RELAJANTE DE MUSCULATURA

PÉLVICA COMO CREMA DE ISOSORBIDE, DILTIAZEM O NIFEDIPINO.

Page 199: ENARM  COMPENDIO

HEMORROIDES

Page 200: ENARM  COMPENDIO

HEMORROIDES

Page 201: ENARM  COMPENDIO

HERNIAS ABDOMINALES• De donde emerge la hernia inguinal directa? R = Emerge DENTRO DE LOS VASOS EPIGÁSTRICOS originándose en el piso del conducto

inguinal

• De donde emerge la hernia inguinal indirecta? R = Por FUERA DE LOS VASOS EPIGÁSTRICOS y son DE ORIGEN CONGÉNITO siendo mas

FRECUENTES DEL LADO DERECHO.

• De que manera se distingue a la ex fis una hernia inguinal directa de indirecta? R = Una HERNIA INDIRECTA EJERCE CIERTA FUERZA CONTRA LA PUNTA DEL DEDO, en

tanto que la DIRECTA PRESIONA LA PULPA DEL DEDO. Además cuando es colocado el dedo en el orificio inguinal y pedirle al paciente que tosa es posible parar la hernia indirecta y no así la directa.

• Cual es la localización de la hernia crural? R = DEBAJO DEL LIGAMENTO INGUINAL, medial a los vasos femorales manifestándose

CON DOLOR EN LA REGIÓN INGUINAL, progresivo e incapacitante.

Page 202: ENARM  COMPENDIO
Page 203: ENARM  COMPENDIO
Page 204: ENARM  COMPENDIO

CAUSAS DE ELEVACION DEL AST O ALT

Page 205: ENARM  COMPENDIO
Page 206: ENARM  COMPENDIO
Page 207: ENARM  COMPENDIO

HEPATITIS VIRAL

Page 208: ENARM  COMPENDIO

VHA-PICORNAVIRUS

• Cual es el dato de laboratorio que orienta al VHA?

R = IgM VS VHA AGUDO E IgG CUANDO YA HABÍA SIDO INFECTADO EL PACIENTE

• Cual es el manejo del VHA? R = Sintomático, Inmunización pasiva como profilaxis postexposicion.

• Cuales son las indicaciones de la VACUNA PASIVA del VHA? R = PERSONAS QUE PLANEAN UN VIAJE, contactos íntimos, PERSONAL DE

GUARDERÍAS, ASILOS E INSTITUCIONES. NO esta INDICADA en px que tienen DATOS CLÍNICOS DEL VHA

• Cuales son las indicaciones de la VACUNA ACTIVA del VHA? R = ENFERMEDAD HEPÁTICA CRÓNICA, VARONES HOMOSEXUALES, DROGADICTOS.

Page 209: ENARM  COMPENDIO

VHB- HEPADNAVIRIDAE• Cual es el dato de laboratorio que te orienta a VHB?

1) HBs Ag, la persistencia de >6m sugiere infección crónica.

2) Anti HBs confiere inmunidad protectora ante el virus.

3) Hbe Ag es el antígeno que refiere la REPLICACIÓN ACTIVA del virus y la CONTAGIOSIDAD, en px con infección crónica permanece constante.

4) IgM antiHBc infección aguda.

5) - 10% EVOLUCIONA A CRONICIDAD, BUEN PRONOSTICO. TX EN CRONICO

• Cual es el manejo del VHB?1) INTERFERON ALFA 2) LAMIVUDINA (análogo de nucleosidos que inhibe la TRANSCRIPTASA REVERSA),

3) ADEFOVIR (análogo de nucleótidos sintético que inhibe a la DNA polimerasa del VHB),

4) ENTECAVIR ( análogo nucleosido deoxiguanina que inhibe la actividad del DNA polimerasa del VHB).

Page 210: ENARM  COMPENDIO
Page 211: ENARM  COMPENDIO

VHD• Que caracteriza al VHD? R = Es EL VIRUS MAS PEQUEÑO con 36 nm, NECESITA DE LA ENVOLTURA DEL VHB para sobrevivir y

su medio de CONTAGIO ES EL MISMO QUE EL DEL VHB.

• Cual es el curso del VHD en coinfección aguda con VHB? R = Tiende a ser LIMITADO Y ELIMINAR EL HBsAg POR LA RESPUESTA INMUNE, al mismo tiempo que

desaparece el VHD.

• Cual es el curso del VHD en súperinfección con VHB? R = Exposición del VHD EN INDIVIDUO PREVIAMENTE INFECTADO POR VHB. LA REPLICACIÓN DEL

VHD ES MAYOR Y MÁS RÁPIDA al encontrar previamente expresado al HBs Ag. En 70% de pacientes de los casos la infección se vuelve crónica y se encuentran mayores tasas de mortalidad

• Cual es el estudio de laboratorio que orienta al diagnostico del VHD?

R = IgG anti VHD o por detección de RNA DE VHD EN SUERO.

• Cual es el manejo del VHD? R = No hay tratamiento especifico, se ha encontrado mejoría con INTERFERON ALFA.

Page 212: ENARM  COMPENDIO

VHE- CALICIVIRUS

Page 213: ENARM  COMPENDIO

FAMILIAS DE HEPATITIS VIRICA

• VHAPICORNAVIRUS• VHBHEPADNAVIRUS• VHCFLAVIVIRUS• VHECALICIVIRUS

Page 214: ENARM  COMPENDIO

HEPATITIS AUTOINMUNE• Que predisposición genética tienen las personas que padecen hepatitis autoinmune? R = HLA-B8, HLA-DR3 Y DR-52.

• Que clasificaciones tiene la hepatitis autoinmune? TIPO I: Autoinmune clásica o Lupoide, se presenta en mujeres jóvenes donde 30% presenta trastornos como TIROIDITIS

AUTOINMUNE O CUCI. A diferencia de otros grupos se presenta HIPERGAMAGLOBULINEMIA EN SUERO 5-6 GR/DL. TIPO II: Mas común en Europa y los anticuerpos que distinguen a este grupo son los ANTIMICROMOSOMA DE HÍGADO Y

RINON TIPO 1 (ANTI LKM 1), asociada a enfermedades inmunológicas como el VITÍLIGO, DM 1, TIROIDITIS AUTOINMUNE. TIPO III: Se encuentran anticuerpos anti antígeno hepático soluble/hígado- páncreas (anti-SLA/LP) encontrando mas

frecuentemente la TIROIDITIS AUTOINMUNE COMO ASOCIADA.

• Cual es el cuadro clínico de las hepatitis autoinmunes? R = MÚLTIPLES TELANGIECTASIAS, astenia, ARTRALGIAS, ESTRIAS CUTÁNEAS, acné, hirsutismo, AMENORREA Y

HEPATOMEGALIA.

• Cuales son los CRITERIOS diagnósticos para hepatitis autoinmune?1) MAYORES: Elevación persistente de transaminasas, Hipergamaglobulinemia 2.5 o mayor, Ac circulantes y/o células LE

positivas, marcadores serológicos para virus de hepatitis negativos. 2) MENORES: Manifestaciones sistémicas como fiebre-artralgias-erupciones cutáneas, Complicaciones con otras

enfermedades autoinmunes , incluyendo las del colágeno, eritrosedimentacion elevada. 3) HISTOLOGIA: Hepatitis crónica o cirrosis con marcada infiltración de células y necrosis de hepatocitos, ausencia de

hallazgos específicos para el diagnóstico de otras entidades.

• Cual es el manejo de la hepatitis autoinmune? R = De elección son los GLUCOCORTICOIDES SOLOS O COMBINADOS CON AZATRIOPINA. La remisión se considera cuando

los px demuestran mejoría clínica, niveles de transaminasas y gammaglobulinas normales en caso de recidivas se usa tratamiento inmunosupresor de por vida.

Page 215: ENARM  COMPENDIO
Page 216: ENARM  COMPENDIO

HEPATITIS ALCOHOLICA• Cual es el cuadro clínico de un paciente con hepatitis alcohólica? R = Perdida de peso, anorexia, ICTERICIA, FIEBRE Y HEPATOMEGALIA

DOLOROSA.

• Cuales son los hallazgos de laboratorio en la hepatitis alcohólica?1) AUMENTO DE LAS ENZIMAS HEPÁTICAS2) ANEMIA 3) TROMBOCITOPENIA 4) LEUCOCITOSIS CON PREDOMINIO DE NEUTROFILOS Y 5) SE CONFIRMA EL DIAGNOSTICO POR BIOPSIA.

• Cual es el manejo de la hepatitis alcohólica? R = Abstinencia, CORTICOIDES y plan nutricio.

Page 217: ENARM  COMPENDIO

CIRROSIS HEPATICA• Que parámetros definen la presión portal? R = Flujo venoso portal y la resistencia interior del hígado

• Cuales son las causas mas frecuentes de cirrosis hepática? R = Alcohol, HEPATITIS B Y C, METOTREXATO, alfametildopa e hidralazina.

• Cual es la clasificación morfológica de la cirrosis?1) CIRROSIS MICRONODULAR: Nódulos menores de 3 mm. 2) CIRROSIS MACRONODULAR: Nódulos de mas de 3 mm. 3) CIRROSIS MIXTA: Mircro y macronodulares que ocurre comúnmente por

alcohol.

• Cual es el marcador mas útil para el diagnostico de cirrosis biliar primaria?

R = ANTICUERPOS ANTIMITOCONDRIALES.

Page 218: ENARM  COMPENDIO

• Cual es el cuadro clínico de la cirrosis? R = Asintomatico o los siguientes:A. SINTOMAS CONSTITUCIONALES: B. Astenia, anorexia, perdida de peso muscular.C. ASCITIS: D. Por retención de sodio y agua.E. SIGNOS CUTANEOS: F. Ictericia, palidez por la anemia. Telangiectacias cutáneas y eritema palmar (que predomina sobre las eminencias tenar e hipotenar y en las

llemas de los dedos) son datos bastantes caracteristicos.G. CIRCULACION COLATERAL: H. Los sitios mas comunes de aparición con significado patológico son el esófago y el fondo gástrico.I. ALIENTO: J. Olor dulzón debido a la exhalación de mercaptanos, productos derivados de la metionina por defecto de su metilación.K. ALTERACIONES ENDOCRINAS: L. Varones con signos de feminización como ginecomastia y cambios en la distribución del vello coroporal. El hipogonadismo en varones produce

atrofia testicular, perdida de la libido, impotencia. En ambos sexos el vello axilar es escaso. En las mujeres es frecuente la oligomenorrea, amenorrea y esterilidad.

M. ALTERACIONES HEMATOLOGICAS: N. Alteraciones en la coagulación dado que todos los factores se sintetizan en hígado menos el factor de von Willebrand.O. ALTERACIONES PULMONARES: P. Ascitis torácica o derrame pleuralQ. ALTERACIONES DIGESTIVAS: R. Hemorragia digestiva, colelitiasis.S. ALTERACIONES RENALES: T. Retención de sodio con ascitisU. SX HEPATORRENAL: V. Se trata de una insuficiencia renal funcional como respuesta a mecanismos de descompensación tales como el uso exagerado de diuréticos,

AINES, paracentesis. La histología es normal. Este síndrome se caracteriza por una insuficiencia renal funcional con función tubular normal en un px con insuficiencia hepática.

W. INFECCIONES: X. IVU mas comunes.Y. ALTERACIONES NEUROLOGICAS: Z. Encefalopatia hepática donde la primer manifestación en aparecer son los cambios en el comportamiento. El GABA es el principal

neurotransmisor inhibitorio del cerebro, se ha postulado que el GABA producido a nivel intestinal escapa del metabolismo hepático, atraviesa la barrera hematoencefalica y penetra al cerebro desencadenando la encefalopatía hepática.

Page 219: ENARM  COMPENDIO

• 184. Cual es el manejo de la cirrosis hepática? R = Dieta con contenido de proteínas 1 a 1.2 g/gr. Prohibicion del consumo de alcohol.

Hay tratamientos utiles específicos de cirrosis, como las flebotomías en la hemocromatosis, la d-penicilamina en la enfermedad de Wilson, los corticoides en las cirrosis autoinmunes, los antivíricos para cirrosis por VHB y VHC y el acido urodesoxicolico para la cirrosis biliar primaria. La cirugía se asocia a una mortalidad operatoria del 30% pero las indicaciones para la cirugía son rotura de varices o sangrados. A las varices tipo 2 y 3 se administra betabloqueador en caso que no exista contraindicación. Seguimiento ecográfico cada 6m para detectar hepatocarcinoma.

• 185. Cual es el manejo de emergencia en la cirrosis hepática? R = En caso de sangrado alto una endoscopia debe realizarse para dar escleroterapia.

Uso de vasopresina por via IV teniendo cuidado de la necrosis de la piel. Anastomosis portosistemica intrahepática por via transyugular TIPS para controlar la hemorragia digestica por varices esofágicas irreductible por otros métodos.

Page 220: ENARM  COMPENDIO

HEPATOCARCINOMA• Cuales son los factores de riesgo para desarrollar un hepatocarcinoma? R = Aparece en personas que ya tienen alguna enfermedad hepática como HEPATITIS POR

VIRUS B O C, CIRROSIS.

• Cual es el cuadro clínico de un px con hepatocarcinoma? R = Puede cursar asintomático. Cuando existen manifestaciones lo mas habitual es que se

presenten síntomas inespecíficos como baja de peso, dolor abdominal, saciedad precoz o masa palpable.

Cuales son los estudios de gabinete utilizados para diagnosticar hepatocarcinoma? R = TAC DINÁMICA HELICOIDAL MULTICORTE DE ELECCIÓN. Us de seguimiento. La

ALFAFETOPROTEINA es el marcador tumoral mas utilizado para el dx y seguimiento (puede elevarse en embarazo y tumores testiculares). Confirmación por biopsia.

• Cual es el manejo del hepatocarcinoma? R = RESECCIÓN del tumor. INYECCIÓN DE ETANOL en tumor lo reseca y es fácil de realizar.

QUIMIOEMBOLIZACION que frecuentemente se complica con el llamado síndrome postembolizacion caracterizado por fiebre, elevación enzimática, dolor abdominal y nauseas. TRANSPLANTE HEPÁTICO.

Page 221: ENARM  COMPENDIO

ABSCESO HEPATICO AMEBIANO

Page 222: ENARM  COMPENDIO

VESICULA BILIAR• Que fármacos pueden obliterar el esfínter de Oddi? R = Morfina y meperidina

• Que elementos componen el síndrome de Mirizzi? R = COMPRESIÓN de los conductos biliares debida a INFLAMACIÓN intensa por colecistitis

secundaria a la IMPACTACIÓN de un lito en el infundíbulo de la vesícula.

• Cual es el estándar de oro para el diagnostico de coledocolitiasis? R = CPRE

• Cual es el cuadro clínico de la COLANGITIS? R = TRIADA DE CHARCOT que se compone de FIEBRE, DOLOR EN CUADRANTE DERECHO E

ICTERICIA. Cuando se le agregan cambios en el estado mental y choque se conoce como PENTADA DE REYNOLDS.

• Cual es el estándar de oro para diagnostico de colangitis? R = CPRE. La presencia de leucocitosis, elevación de las bilirrubinas, FA y transaminasas apoyan

el diagnostico.

Page 223: ENARM  COMPENDIO
Page 224: ENARM  COMPENDIO

HEMOCROMATOSIS• Que tipo de trastorno es la hemocromatosis y que intercambio de aa se involucran?

1) AR. Se produce por sustitución de CISTINA POR TIROSINA en la posición 282

(C282Y). 2) Trastorno de almacenamiento de hierro en el que el incremento inapropiado de la absorción

intestinal. Rara vez se presenta antes de los 20ª.

• Cual es el cuadro clínico de la hemocromatosis? R = Los síntomas iniciales consisten en debilidad, cansancio, perdida de peso, cambio de coloración de la

piel, dolor abdominal, perdida de la libido y síntomas de DM. La HEPATOMEGALIA 95% , HIPERPIGMENTACION 90 %(color gris pizarra o metálico/ bronceado), POLIARTRITIS 20-

50%, ICC 10%, angiomas en araña, esplenomegalia, artropatía, ascitis, arritmias cardiacas,

HIPOGONADISMO con perdida de vello, atrofia testicular y la ictericia son signos de enfermedad avanzada.

• Como diagnosticas hemocromatosis? R = 1) Medición de hierro sérico y porcentaje de TRANSFERRINA > 50% EN AYUNO 2) Medición de

concentración de ferritina 3) Biopsia hepática

• Cual es el manejo de la hemocromatosis? R = FLEBOTOMIAS de 500 ml una o 2 veces a la semana. DEFEROXAMINA cuando la anemia o

hipoproteinemia son muy graves.

Page 225: ENARM  COMPENDIO

ENFERMEDAD DE WILSON• Que causa la enfermedad de Wilson? R = AR. Causado por mutaciones en el gen ATP 7B, cuyo producto es una ATPasa transportadora de cobre

ligada a membrana.

• Cual es el cuadro clínico de la enfermedad de Wilson? R = MANIFESTACIONES NEUROLÓGICAS manifestadas principalmente con trastornos del movimiento

como DISTONIA, INCOORDINACIÓN Y TEMBLORES. La disfagia y la disartria son comunes. Anillos de KAYSER-FLEISCHER.

• Cuales son las pruebas diagnosticas para la enfermedad de Wilson?

R = Niveles séricos de CERULOPLASMINA BAJOS. Anillos de Kayser-Fleischer 99 %. Biopsia hepática con análisis cuantitativos de cobre con valores >3. 1mol/g.

• Cual es el manejo de la enfermedad de Wilson?1) PENICILAMINA + PIRIDOXINA. 2) La TRIENTINA ha sustituido a la penicilamina. 3) ZINC es menos toxico.

• Cual es el pronostico de la enfermedad de Wilson? R = BUENO, los síntomas neurológicos desaparecen una vez iniciado el tratamiento.

Page 226: ENARM  COMPENDIO

PANCREATITIS AGUDA• Cual es el cuadro clínico de la pancreatitis aguda? R = Aparición súbita de un dolor en la región abdominal superior acompañado de nausea y vomito. El paciente reporta

el dolor en el epigastrio y en la región periumbilical, con irradiación hacia espalda, tórax, flancos y abdomen inferior. Tomando posición antialgica. A la EX FIS: Fiebre, hipotensión, rigidez abdominal, taquipnea y distención abdominal. Es posible encontrar los signos de CULLEN (hematoma periumbilical) o de GREY-TURNER (hematoma en los flancos) relacionados con hemorragias retroperitoneales.

• Cuales son los hallazgos de laboratorio en la pancreatitis aguda?1) Elevación de la amilasa pancreática. 2) LIPASA PANCREÁTICA QUE SE ELEVA LAS PRIMERAS 4-8 HRS. 3) Niveles de AMILASA, TAC Y US TE ORIENTAN AL DIAGNOSTICO. En la pancreatitis aguda grave disminuye el calcio.

• Cual es el hallazgo radiológico sugestivo de pancreatitis aguda? R = ASA EN CENTINELA.

• Cuales marcadores de gravedad de pancreatitis aguda hay? R = Balthazar, Ranson y APACHE II.

• Cuales son los manejos de la pancreatitis aguda?1) TRATAMIENTO MEDICO: La pancreatitis leve se debe tratar con HIDRATACIÓN AGRESIVA hasta que el paciente

muestre balances hídricos positivos. El control del dolor se puede realizar con MORFINA. Nutrición enteral. En los casos graves, se recomienda ELIMINAR LA INGESTIÓN POR VIA ORAL, sin descuidar la nutrición y estado catabólico del paciente.

2) TX QUIRURGICO: En necrosis infectada

Page 227: ENARM  COMPENDIO

• Los pacientes que presentan hasta dos de las caracteristicas previamente mencionadas tienen una mortalidad minima. Aquellos con tres a cinco de estos factores, presentaran una mortalidad 10 y 20% y los que tengan mas de 5 presentan 50% de mortalidad.

Page 228: ENARM  COMPENDIO
Page 229: ENARM  COMPENDIO

PANCREATITIS CRONICA

• Cuales son las causas de pancreatitis crónica? R = CONSUMO EXCESIVO DE ALCOHOL, enfermedades

autoinmunes, hipertrigliceridemia, hiperparatiroidismo, páncreas divisum y FIBROSIS QUÍSTICA.

• Cual es el cuadro clínico de la pancreatitis crónica? R = ATAQUES INTERMITENTES de dolor intenso, de localización a

nivel abdominal superior con irradiación en forma de banda hacia la espalda.

• Cuales son los estudios de imagen utilizados para el diagnostico de pancreatitis crónica?

R = TAC, US y CPRE.

Page 230: ENARM  COMPENDIO

CANCER DE PANCREAS

• Cuales son los factores de riesgo para desarrollar cáncer de páncreas? R = Tabaquismo, diabetes de larga evolución, pancreatitis crónica y NEM 1.

• Cual es el cuadro clínico del cáncer de páncreas? R = Perdida de peso, fatiga, dolor abdominal epigástrico transfictivo, ictericia,

tromboflebitis migratoria, datos de hipertensión portal (hepatomegalia y ascitis) y VESÍCULA PALPABLE (signo de Courvoisier-Terrier).

• Como diagnosticas cáncer de páncreas? R = TAC. Elevación FA y bilirrubina en sangre. Marcador tumoral CA 19-9.

• Cual es el manejo del cáncer de páncreas? R = Quirúrgico. Quimioterapia.

Page 231: ENARM  COMPENDIO
Page 232: ENARM  COMPENDIO
Page 233: ENARM  COMPENDIO
Page 234: ENARM  COMPENDIO

MANEJO DEL PX POLITRAUMATIZADO

Page 235: ENARM  COMPENDIO
Page 236: ENARM  COMPENDIO

TRAUMATISMO TORAXICO

Page 237: ENARM  COMPENDIO

NEUMOTORAX Y HEMOTORAX TRAUMATICO

Page 238: ENARM  COMPENDIO

TRAUMATISMOS ABDOMINALES

Page 239: ENARM  COMPENDIO
Page 240: ENARM  COMPENDIO
Page 241: ENARM  COMPENDIO

TRAUMATISMOS DEL APARATO

GENITOURINARIO

Page 242: ENARM  COMPENDIO
Page 243: ENARM  COMPENDIO

SHOCK HIPOVOLÉMICO¿Cuándo un paciente está en shock hipovolémico?

Pérdida estimada de sangre en un hombre de 70 Kg

CLASE I CLASE II CLASE III CLASE IV

Pérdida de sangre (ml) < 750 750 - 1500 1500 - 2000 > 2000

Pérdida de sangre (%) < 15 % 15 – 30 % 30 – 40 % > 40 %

Pulso < 100 > 100 > 120 > 140

Presión arterial Normal. Normal. Disminuida. Disminuida

Presión pulso Normal Disminuida Disminuida Disminuida

Frecuencia respiratoria 14 - 20 20 - 30 30 - 40 > 35

Diuresis (ml/h) > 30 20 - 30 5 - 15 Mínima

Conciencia Levementeansioso

Moderadaansioso.

Ansioso,confuso.

Confuso,letárgico.

Page 244: ENARM  COMPENDIO
Page 245: ENARM  COMPENDIO

NEFROLOGIA

ENARM

Page 246: ENARM  COMPENDIO

ANION GAP• El Anión GAP es una ECUACIÓN que sirve PARA CALCULAR ANIONES ORGÁNICOS que están presentes a

una CONCENTRACIÓN MUY PEQUEÑA como para ser medidas en un ionograma...Partiendo desde el concepto de electro neutralidad, pareciera que los cationes "superan" a los aniones, pero justamente la "brecha" entre ellos esta representados por estos Aniones...

La utilidad fisiológica/clínica del Anión Gap, es para orientarte acerca de la POSIBLE ETIOLOGÍA ( causa) DE LA ACIDOSIS METABOLICA ( y SOLO SIRVE PARA ESTO) que el paciente esta cursando...

Las AC METAB. pueden ser DE 2 "TIPOS"... por FALTA DE BASE ( el tipo tiene una DIARREA tremenda, esta largando hasta la 1º papilla... ), o por EXCESO DE ACIDO ( esta TOMANDO ASPIRINA como si fueran tic tacs... o CETOACIDOSIS etc.) ( esos son algunos ejemplos... )...

Si calculas el AG... SIENDO AG= [NA+]P - ([CL-]P+[HCO3-]P)

Los valores NORMALES: AG= 12 +- 2

En caso de que el Anion Gap de AUMENTADO (+ de 14) Entonces estamos viendo una ACIDOSIS METABOLICA POR EXCESO DE ACIDO...

Si el AG DA NORMAL, entonces, el paciente tiene un AC METAB. POR FALTA DE BASE...Este tipo de ac metab, también es conocida por ACIDOSIS HIPERCLOREMICA

Page 247: ENARM  COMPENDIO
Page 248: ENARM  COMPENDIO
Page 249: ENARM  COMPENDIO
Page 250: ENARM  COMPENDIO

POTASIO• Cuales son las causas de la hipokalemia?• R = Incremento en el recambio celular, disminución de la ingesta o un aumento de su excreción

• Que medida de laboratorio es útil para el diagnostico de hipokalemia de origen renal o extrarenal?

• R = EL GRADIENTE TRANSTUBULAR DE POTASIO (TTKG), el cual si es < 2 TRADUCE HIPOKALEMIA NO RENAL y UNO > 10 ES

DEBIDO A PERDIDAS RENALES. EL K URINARIO <25 MEQ/DIA es NO RENAL y K URINARIO > 30 MEQ/DIA traduce perdida de ORIGEN RENAL.

• Cuales son lo síntomas de hipokalemia?• R = DEBILIDAD MUSCULAR, fatiga, CALAMBRES. <2.5 Meq parálisis flácida, HIPORREFLEXIA, hipercapnia y rabdomiolisis.

• Que elementos participan en la captación de potasio?• R = La insulina en presencia de glucosa, estimulación B adrenérgica

• Cual es el regulador de potasio mas importante?• R = ALDOSTERONA facilita la excreción urinaria de K en los TCD.

• Cuales son las causas de la hiperkalemia?

• R = > 5.5 mEq. DISMINUCION DE LA EXCRESION: HIPOALDOSTERONISMO, IRA o IRC, ESPIRONOLACTONA, IECA, B bloqueador. Desplazamiento de K al interior: Rabdomiolisis, hemolisis y ACIDOSIS METABOLICA

• Cual es el cuadro clínico de hiperkalemia y EKG?• R = DEBILIDAD MUSCULAR, distención abdominal, DIARREA. EKG con onda T picuda, QT corto y depresión de onda T.

• Cual es el manejo de la hiperkalemia?1) INSULINA + GLUCOSA AL 10%, SALBUTAMOL inhalado, GLUCONATO DE CALCIO IV, HCO3 que produce ingreso a la celula de K.2) EL TRATAMIENTO AGUDO CON GLUCONATO DE CALCIO DEBE RESERVARSE cuando se presentan manifestaciones cardiacas de

toxicidad celular por exceso de potasio (calcitosis).

Page 251: ENARM  COMPENDIO
Page 252: ENARM  COMPENDIO
Page 253: ENARM  COMPENDIO
Page 254: ENARM  COMPENDIO

NTA• A que padecimiento se le conoce como NTA?• R = IRA por lesiones tubulares siendo las principales causas la isquémica y por toxinas

• Que nefrotoxinas endógenas ocasionan NTA?• R = Productos contenedores de HEM, ACIDO ÚRICO, MIOGLOBINURIA por

RABDOMIOLISIS

• Cuales son los datos de lab de NTA?• R = ORINA COLOR CAFÉ LODOSO por Hb y cilindros epiteliales de túbulos renales

• Cual es el manejo de la NTA?• R = Se debe prevenir hipercalcemia, FUROSEMIDE, RESTRICCIÓN DE PROTEÍNAS

DIETÉTICAS Y DIÁLISIS EN PACIENTE CRITICO

• Cual es la complicación mas común y mas temida de la NTA?• Falla cardiaca

Page 255: ENARM  COMPENDIO

NEFRITIS INSTERSTICIAL AGUDA• Que es la nefritis intersticial aguda?• R = Respuesta inflamatoria intersticial con edema y posible daño celular

• Cuales son las causas de nefritis intersticial?• R = FÁRMACOS 70%. Infecciosas e inmunitarias

• Cuales son los datos clínicos de nefritis intersticial aguda?

• R = Se caracteriza por un DETERIORO ABRUPTO DE LA FUNCIÓN RENAL acompañado de

poliuria, Nicturia, ACIDOSIS METABOLICA y glucosuria. FIEBRE 90%, ARTRALGIAS 80%,

EXANTEMA 20%. Bh: EOSINOFILIA EN FASE AGUDA. EGO con EOSINOFILURIA, hematuria, leucocituria y cilindros leucocitarios.

• Cuales son los hallazgos en la biopsia con nefritis intersticial aguda?• R = Infiltración de células inflamatorias en intersticio renal con edema, predominan mononucleares y LT, GRANULOMAS

NO CASEIFICANTES.

• Cual es el manejo de la nefritis intersticial aguda?• R = RETIRAR EL FACTOR predisponente. Puede llegar a REQUERIR DIÁLISIS, MEDIDAS DE SOSTEN,

METILPREDNISOLONA en NI POR FÁRMACOS

• Que caracteriza a la necrosis intersticial crónica?1. Ausencia de proteinuria e hipoalbuminemia2. Piuria estéril y leucocitosis mas que hematuria3. Poliuria y Nicturia4. Otros defectos tubulares como ATR y osteomalacia.

Page 256: ENARM  COMPENDIO

IRC• Cual es la clasificación de la IRC?• ESTADIO FG• I 90• II 60 – 89• III 30 – 59• IV < 15 O DIALISIS

• Cual es el manejo restrictivo de IRC?• R = Restricción de proteínas, restricción de Na y H2 O, restricción de Mg

entre otros.

• Que pasa con la capacidad de dilución y de concentración en la IRC?• La capacidad de DILUCIÓN se deteriora antes que la de CONCENTRACIÓN.

Page 257: ENARM  COMPENDIO
Page 258: ENARM  COMPENDIO

CRIOGLOBULINEMIA• Que caracteriza a la crioglobulinemia y con que enfermedades se relaciona?• R = Es una vasculitis de pequeños vasos DEBIDO A INMUNOGLOBULINAS que se

PRECIPITAN CON EL FRIO relacionándose mucho con el VHC +++ mas que con el VHB, endocarditis o LES.

• Cual es el cuadro clínico de la crioglobulinemia?• R = PURPURA 90 %, ARTRITIS 80% , Neutropenia 70 %, DANO RENAL 50% (de este el

80% se manifiesta como GLOMERULONEFRITIS MEMBRANOPROLIFERATIVA Y TROMBOS EN EL INTERIOR DE LOS CAPILARES GLOMERULARES.

• Cual es el manejo de la crioglobulinemia?1) Proteinuria y daño renal leve dándose TRATAMIENTO SINTOMÁTICO Y ESPECIFICO2) PARA VHC SI ES QUE ESTA ASOCIADO con origen se da INTERFERON Y RIVABIRINA,3) Si la PROTEINURIA se encuentra en RANGO NEFRÓTICO se da METILPREDNISOLONA,

PLASMAFERESIS O RITUXIMAB.

Page 259: ENARM  COMPENDIO

NEFROPATÍA POR IgA O SX DE BERGUER

• Que caracteriza a la nefropatía por IgA o sx de Berguer?

• R = Se deposita IgA en el mesangio glomerular, observándose la MISMA LESIÓN EN PURPURA DE HENOCH- SCHONLEIN

• Cuales son los signos de nefropatía por IgA?

• R = HEMATURIA “SINFARINGITICA” con orina color COCA-COLA 100%. IVRS 50%, GI 10%

• Que datos se encuentran en la biopsia en la nefropatía por IgA ?• R = DEPOSITOS DE IgA acompañados de C3 E IgG.

• Cual es el manejo de la nefropatía por IgA?1) IECAS, ESTEROIDES. 2) En caso de proteinuria se combina CICLOFOSFAMIDA CON AZATRIOPINA.

• Cual es el pronóstico de nefropatía por IgA?

• R = BUENO, pero es MALO para los que desarrollan SEMILUNAS EN LA BIOPSIA

Page 260: ENARM  COMPENDIO

GN PAUCINMUNITARIA• En que enfermedad se presenta GN paucinmunitaria?

1) GRANULOMATOSIS DE WEGENER,

2) ENFERMEDAD DE CHURG- STRAUSS,

3) POLIANGEITIS MICROCITICA.

• Que patogénesis se encuentra hubicada en 80% de las glomerulonefritis paucinmunitarias?

• R = Asociada a ANCA

• Cuales son los datos de laboratorio de la GN paucinmunitaria?• R = Patron citoplasmático c-ANCA, patrón perinuclear p-ANCA.

• Cual es el manejo de la GN paucinmunitaria?• R = ESTEROIDES, CICLOFOSFAMIDA.

Page 261: ENARM  COMPENDIO
Page 262: ENARM  COMPENDIO
Page 263: ENARM  COMPENDIO
Page 264: ENARM  COMPENDIO
Page 265: ENARM  COMPENDIO

AFECCION RENAL X ENF SISTEMICA

Page 266: ENARM  COMPENDIO

NEFROPATIA LUPICA

Page 267: ENARM  COMPENDIO

NEFROPATÍA MEMBRANOSA• Que es la nefropatía membranosa?

• R = Enfermedad glomerular por DEPOSITO DE IgG y complemento constituyendo la forma mas COMÚN DE SÍNDROME NEFRÓTICO EN ADULTOS, es idiopático 70% y el resto se debe a virus, neoplasias, drogas o enfermedades autoinmunes

• Cual es el cuadro clínico de la nefropatía membranosa?• R = SÍNDROME NEFRÓTICO.

• Que etiología tiene la NM secundaria?• R = LES, VHC Y ENDOCARDITIS son las mas comunes. Hepatitis B, sífilis, cáncer, penicilamina y captopril.

• Cuales son los signos y síntomas de NM secundaria?• R = Relacionados a SX NEFRÓTICO

• Que revela la biopsia e inmunohistoquimica en la GMN?• R = HISTOLOGIA: PROLIFERACIÓN EN FORMA DE PARCHES, infiltrado leucocitario intraglomerular y necrosis

intracapilar. INMUNOHISTOQUIMICA: IgG siempre presente, IgM e IgA.

• Cual es el manejo de la nefropatía membranosa?• R = PROTEINURIA < 4 GR DAS IECAS CON META DE TA 125/75. PROTEINURIA > 4 GR y < 8 GR SIN DETERIORO DE

LA FUNCIÓN RENAL EL TRATAMIENTO SE ENCAMINA A DISMINUIR LA PROTEINURIA CON IECAS Y OBSERVAR POR 6M.

Page 268: ENARM  COMPENDIO

GLOMERULONEFRITIS FOCAL Y SEGMENTARIA

• Cual es la etiología de la Glomerulonefritis focal y segmentaria?• R = Primaria (idiopática) o secundaria debido a otras patologías como agenesia renal, ENFERMEDAD DE CÉLULAS

FALCIFORMES o por VIH.

• Dentro de las diferentes variedades de Glomerulonefritis focal y segmentaria cuales producen mas proteinuria?• R = LA VARIEDAD CELULAR Y COLAPSANTE en el 80% de los casos de síndrome nefrótico

• Cual es el cuadro clínico de la Glomerulonefritis focal y segmentaria?• R = PROTEINURIA asintomática o síndrome nefrótico completo

• Cuales son los datos de laboratorio y biopsia de la Glomerulonefritis focal y segmentaria?• R = La PROTEINURIA puede oscilar DESDE <1 GR HASTA 20-30 GR y en la biopsia los hallazgos de HISTOLOGÍA

COMO SON LA VARIEDAD CELULAR Y COLAPSANTE.

• Cual es el manejo de la Glomerulonefritis focal y segmentaria?1) LOS QUE TENGAN PROTEINURIA SUBNEFROTICA 2-3 GR/ no se les dará tratamiento inmunosupresor, siendo el

principal tratamiento el control de la TA CON META DE 130/80, UTILIZANDO IECAS, hipolipemiantes, DIURÉTICOS EN CASO DE EDEMA.

2) En los pacientes que se presentan con SÍNDROME NEFRÓTICO SE USA PREDNISONA, CONTROL DE LA TA CON META DE 125/75, ANTICOAGULANTES (>10 GR/DIA) y en caso de RESISTENCIA SE USARA CICLOSPORINA.

• Cual es el pronostico de la Glomerulonefritis focal y segmentaria?• R = MALO SI LA PROTEINURIA ES > 10 GR/DIA

Page 269: ENARM  COMPENDIO

LIDDLE, GITELMAN Y BARTTER• Que es la enfermedad de Liddle?• R = AD. El síndrome de Liddle (o SEUDOHIPERALDOSTERONISMO) es un trastorno AD caracterizado por

HIPERTENSIÓN SENSIBLE A SAL con expansión de volumen, HIPOKALEMIA y ALCALOSIS METABOLICA, con ACTIVIDAD DE RENINA Y ALDOSTERONA EXTREMADAMENTE BAJAS y FG NORMAL.

• Que es el síndrome de Gitelman?• R = Se caracteriza por HIPOKALEMIA e HIPOMAGNESEMIA, además de contar con HIPOCALCIURIA, perdida de

sal y alcalosis metabólica con HIPERALDOSTERONISMO, HIPERRENINEMIA, asi como HIPERTROFIA E HIPERPLASIA DEL APARATO YUXTAGLOMERULAR.

• Cual es el cuadro clínico del síndrome de Gitelman?

• R = Debilidad muscular y DERMATITIS inespecífica. ESPASMOS DISTALES ocurren EN PERIODOS DE FIEBRE, vomito o diarrea. Algunos sufren condrocalcinosis.

• Que es el síndrome de Bartter?

• R = AR. Se caracteriza por HIPOKALEMIA GRAVE, perdida de sal y alcalosis metabólica con HIPERALDOSTERONISMO, HIPERRENINEMIA, así como HIPERTROFIA E HIPERPLASIA DEL APARATO

YUXTAGLOMERULAR. Casi TODOS TIENEN NEFROCALCINOSIS por la hipercalciuria.

• Cual es el cuadro clínico del síndrome de Bartter?• R = Tipo I y II: Las manifestaciones tempranas incluyen POLIHIDRAMNIOS, retardo en el crecimiento, polidipsia,

deshidratación, avidez por sal y debilidad muscular. En el tipo III: Se manifiesta en la INFANCIA similar al Sx de Gitelman. El tipo IV: TODO LO ANTERIOR MAS SORDERA SENSORINEURAL E IRC TEMPRANA.

Page 270: ENARM  COMPENDIO

SÍNDROME DE FANCONI• Que es el síndrome de Fanconi?• R = Se debe a la DISFUNCIÓN GLOBAL DEL TÚBULO PROXIMAL.

• Cuantos tipos de Sx de Fanconi hay y que los caracteriza?1) HEREDITARIO: Cistinosis, galactosemia, tirosinemia. 2) ADQUIRIDO: INTOXICACIÓN POR PLOMO O CADMIO, TETRACICLINAS

CADUCADAS, tolueno (inhalar pegamento), agentes quimioterapéuticos como el cisplatino.

3) FANCONI LIKE: Pacientes adultos con disproteinemias como amiloidosis, en enfermedad de cadenas ligeras y mieloma múltiple.

• Cual es la anormalidad metabólica mas frecuente en el síndrome de Fanconi?

• R = ACIDOSIS METABOLICA HIPERCLOREMICA.

Page 271: ENARM  COMPENDIO

ATR• Cuales son las tubulopatias asociadas a alcalosis metabólica?• R = Síndrome de CHEEK-PERRY(GITLEMAN), LIDDLE Y BARTTER

• Que es la acidosis tubular renal o ATR?• R = Grupo de entidades patológicas caracterizadas por defectos de transporte, ya sea REABSORCIÓN DE

BICARBONATO (HCO3), EXCRECIÓN DE HIDROGENIONES O AMBAS.

• Donde se lleva a cabo la reabsorción de HCO3 y la excreción de H+?• R = HCO3 EN EL TCP Y H+ EN TCD.

• Que caracteriza a la ATR tipo I DISTAL O CLÁSICA?• R = ACIDOSIS METABÓLICA CON BRECHA ANIONICA NORMAL O HIPERCLOREMICA, HIPOKALEMIA E

IMPOSIBILIDAD DE ACIDIFICAR LA ORINA A < 5.5 bajo el estimulo de acidosis metabólica intensa.

• Cuales son las bases fisiopatológicas de la ATR I?• R = 1) Defecto secretor (DEFICIENCIA EN LA SECRECIÓN DE HIDROGENIONES), formas dominantes,

recesivas y asociadas a síndrome de Sjogren, LES, AR. 2) defecto de gradiente, en el cual hay un reflujo de los hidrogeniones normalmente secretados a nivel distal (anfotericina B) y 3) Incapacidad de generar o mantener una diferencia luminal negativa transepitelial distal como en la uropatia obstructiva, enfermedad de células falciformes, hiperplasia adrenal perdedora de sal.

• Como se diferencia la ATR tipo I de la tipo II?• R = Que estos pacientes a menudo PRESENTARAN NEFROLITIASIS Y NEFROCALCINOSIS.

Page 272: ENARM  COMPENDIO

• Como se maneja la ATR tipo I?

• R = ADMINISTRACIÓN DE ÁLCALI, SE PREFIERE CITRATO DE POTASIO

• Que caracteriza a la ATR TIPO II O PROXIMAL?• R = Puede ocurrir sola o acompañada de otros defectos tubulares como síndrome

de Fanconi y se caracteriza por una DISMINUCIÓN EN EL UMBRAL RENAL PARA EL HCO3, el cual usualmente es de 22 mml/L en infantes y 26 mmol/L en adultos.

• Que caracteriza a la ATR tipo IV hiperkalemica?• R = Usualmente ocurre con la presencia de IR MODERADA, sin embargo la

magnitud de la HIPERKALEMIA y de la acidosis es desproporcionadamente extensa para el nivel de IR.

• Cual es la fisiopatología de la ATR tipo IV o hiperkalemica?• R = Defecto en la AMONIOGENESIS

• Cual es la principal enfermedad relacionada con la ATR tipo IV o hiperkalemia?• R = DM, además del uso de AINES, IECAS y ciclosporina.

Page 273: ENARM  COMPENDIO
Page 274: ENARM  COMPENDIO

SINDROME UREMICO HEMOLITICO

•En el SHE es común observar la prolongación del tiempo de?• R = Sangría

Page 275: ENARM  COMPENDIO
Page 276: ENARM  COMPENDIO
Page 277: ENARM  COMPENDIO

INFECTOLOGIA

ENARM

Page 278: ENARM  COMPENDIO

MENINGITIS• Cual es la presión de apertura a la toma de LCR en meningitis bacteriana?• R = Normal <180 o elevada > 180, siendo común 200-500

• Cuales son los datos del LCR de la meningitis bacteriana aguda?I. Presión de abertura: Normal o > 180 siendo común observar valores entre 2 – 500 II. Apariencia: Leucocitos y bacterias turban el LCRIII. Glucorraquia: Concentración < 40 (NORMAL > 45)IV. Proteinorraquia: Valores arriba de 50 (NORMAL 40)

• Cual es el tratamiento de elección en los contactos de pacientes con enfermedad meningocococica?

• R = RIFAMPICINA

• Que constituye el síndrome de austrian?

• R = MENINGITIS, ENDOCARDITIS y NEUMONÍA por estreptococo pneumoniae en PACIENTE ALCOHÓLICOS

Page 279: ENARM  COMPENDIO

MENINGITIS• Tratamiento de elección para meningitis por NEUMOCOCO?• R = CEFTRIAXONA

• Tratamiento de elección para meningitis por S. AUREUS METICILINO SENSIBLE?• R = Nafcilina, DICLOXACILINA

• Tratamiento de elección para meningitis por S. AUREUS METICILINO RESISTENTE?• R = VANCOMICINA o TEICOPLANINA

• Tratamiento de elección para meningitis por LISTERIA?• R = AMPICILINA

• Tratamiento de elección para meningitis por H. INFLUENZA?• R = CEFTRIAXONA o cefotaxima

• Tratamiento de elección para meningitis por ANAEROBIOS?• R = METRONIDAZOL

• Cual es el tratamiento de elección en la meningitis CRIPTOCOCOCICA?• R = ANFOTERICINA B + 5-fluocitosina

Page 280: ENARM  COMPENDIO

MENINGITIS• Característica de meningoencefalitis subaguda? • Meningoencefelatis por tuberculosis

• Agente etiológicos en meningitis en recién nacidos?• E.COLI (en el trabajo de parto) y estreptococos del grupo B (AGALACTIAE)

• Agente etiológico de meningitis en adultos?• 1.Streptococo pneumoniae 2.Neisseria Meningitidis 3.Streptococo del grupo B

• CASO CLÍNICO: Recién nacido, con fontanela abombadas, petequias en la cara anterior del tórax, esta en cunero y alado entro por deshidratación. Actualmente con fiebre, rigidez de nuca, Kerney y Brusinski, punción lumbar con diplococos GRAM NEGATIVOS en LCR

• Agente etiológico?: • Meningitis por Neisseria Meningitidis (MENINGOCOCO) • Tx: • PGSC a dosis altas, al personal involucrado: cipro o rifampicina

• Medida implantada por la OMS para detección rápida de MENINGITIDIS?• DETERMINACIÓN DE ANTÍGENOS CAPSULARES: Streptococo pneumone, Neisseria Meningitidis,

Hemophilus Influenza se detecta en 3 horas –aunque no los solicites-

Page 281: ENARM  COMPENDIO

MENINGITIS• Meningitis POR LISTERIA (BACILO GRAM +) esta relacionado con:• LECHE O DERIVADOS LACTEOS, CARNES MAL COCIDAS, INMUNODEPRIMIDOS y px de la

tercera edad…. Tx :AMPICILINA como primera opción

• Con respecto del Streptococo Pneumoniae caract de infección?:• Se da principalmente en pacientes de los 2 a los 20 años, px ALCOHÓLICOS, con OTITIS

PREVIA.

• Meningitis por Bacilos GRAM NEGATIVOS caract de infección?• Se presenta en diabéticos, cirróticos, alcohólicos px con IVU, px con cáncer y/o

inmunodeprimidos. El bacilo GRAM POSITIVO y entérico mas importante la listeria monocitogena, otro bacilo que no es entérico es la hemophilus Influenza (disminución en su incidencia actualmente)

• Agente etiológico de la Meningitis en un px que previamente se le realizo procedimiento neuroquirurgico?

• STAPHILOCOCOS AUREUS

• Citocinas proinflamatorias que producen daño a nivel SNC, en la barrera hematoencefalica, plexos coroides?

• IL 1 Y FNT que aumentan en las primeras 2 horas

Page 282: ENARM  COMPENDIO

MENINGITIS• Laboratorio de LCR?• PUNCIÓN LUMBAR: 1.-PRESION NORMAL DEL LCR 180mm de Agua, 2.-POLIMORFONUCLEARES: menos

de 5 polimorfos 3.-GLUCOSA: es el 50% dela glucosa central del px entre 45 y 55 4.-PROTEINAS: 40mg/dl 5.-AGUA DE ROCA 6.-CLORUROS: 90 a 110 en LCR

• En quienes se realiza el cultivo en TINTA CHINA?• CRIPTOCOCO NEOFORMANS EN PX CON VIH

• Caso clinico• Px con rigidez de nuca, brusinski +, irritado Cefalea, fiebre, sin traumatismo craneoencefálico,

ANTECEDENTE DE NADAR EN UN MANANTIAL LANZÁNDOSE CLAVADOS, a la punción lumbar: aspecto turbio, presión:200mmhg glucosa:20mg/dl proteínas:85 mg/dl Tinción de GRAM fue NEGATIVA, el cultivo se realizo, que tipo de meningitis tiene este px:

• a.-M aseptica b.-M purulenta c.-M Tuberculosa (Agente etiológico: NAEGLERIA o gardenelas son amibas de vida silvestre) muy agresivas, con pronostico mortal,

• Tx?: • ANFOTERICINA B

• La meningitis Viral caract del LCR:• CELULARIDAD: NORMAL PROTEÍNAS: ligeramente AUMENTADAS ASPECTO: AGUA DE ROCA

GLUCOSA: NORMAL PRESIÓN: nl o ligeramente NORMAL CLORUROS: NORMALES

Page 283: ENARM  COMPENDIO

MENINGITIS• CASO CLÍNICO: Px con fiebre intensa (40), ingresa al servicio de urg, con trastorno

del edo. De conciencia, con 2 crisis convulsivas.. de ocupación CUIDADOR DE CABALLOS (antecedente de haber enterrado a su caballo y después fue atacado por aves de rapiña)

• LCR: Glucosa: 100 liquido transparente, presión: 185mm de agua, celularidad: linfocitos cloruros normales

• Cual es la impresión dx: • MENINGOENCEFALITIS VIRAL EQUINA VENEZOLANA

• CASO CLÍNICO: Niña Indígena procedente de AREA RURAL, se presenta al servicio de urg. Con 3 semanas de evolución con fiebre persistente, vomito, trastorno del estado de conciencia, ala exploración física presenta papiledema ligero, con afectación del 3er, 4to, 6to par craneal, con rigidez de nuca. LCR: ASPECTO XANTOCROMICO, Presión: 220mm de agua Glucosa: 30 Proteínas: de 75mg/dl , Celularidad por linfocitos CLORUROS: 70 Meningitis tuberculosa

• Tx de meningitis en Recién Nacidos?• AMPICILINA (e.coli)- CEFOTAXIMA (streptococo)

Page 284: ENARM  COMPENDIO

MENINGITIS• Tx de meningitis en niños de 1 a 3 meses?• AMPI + CEFO o CEFTRIAXONA + (valorar administración de DEXAMETASONA) para evitar

VENTRICULITIS

• Tx de meningits en px de 3 meses y menos de 50 años?• CEFOTAXIMA + VANCOMICINA o CEFTRIAXONA + VANCOMICINA (cubrir cocos gram +, Y

bacilos gram -)

• Tx de meningitis en un px mas de 50 años con antecedente de alcoholismo o tiene DM• AMPICILINA + VANCOMICINA + CEFALOSPORINA (tx p cocos gram + bacilos gram -)

• Px neutropenico, o px con meningitis con TRAUMATISMO CRANEOENCEFALICO, o que le hicieron un procedimiento neuroquirurgico que tx se le dara?

• Cubrir Gram - (ceftazidil), cubrir Gram + meticilino resistente (Vancomicina)

• px que en el cultivo presenta meningococo que tx se da? • PENICILINA G si es resistente: CEFTRIAXONA O CEFOTAXIMA y si es alérgico VANCOMICINA (2

gr x dia)

• Px que le hicieron punción lumbar y obtuvieron en el LCR un cultivo Gram – que tx:• Pseudomona aeroginosa: Ceftazidine(6gr x dia) Kliebsella, e coli: CEFOTAXIMA O

CEFTRIAZONA (4gr x dia)

• Tx para: Listeria: AMPICILINA(12 gr al dia) H. Influenza: CEFTRIAXONA o cefotaxima Anaerobios: METRONIDAZOL (2 grx dia)

Page 285: ENARM  COMPENDIO
Page 286: ENARM  COMPENDIO
Page 287: ENARM  COMPENDIO

FOD• Cual es la definición de FOD?• R = Elevación de la temperatura por arriba de 38 GRADOS en varias determinaciones durante

mas de 3 SEMANAS sin llegar al diagnostico DESPUÉS DE 1 SEMANA de estudio hospitalario

• Cual es la definición de FOD en paciente hospitalizados?• R = Paciente hospitalizado 24 HRS CON FIEBRE y 2 CULTIVOS NEGATIVOS después DE 2 DÍAS.

• Define FOD en paciente inmunodeficiente?• R = FIEBRE >38.3 en paciente con NEUTROPENIA < 500 durante > 3 DÍAS y cultivos negativos

después DE 2 DÍAS.

• Define FOD asociado a VIH?• R = Fiebre por encima de 38.3 GRADOS y que permanece sin diagnostico por MAS DE 3

SEMANAS

• Cuales son las principales causas de FOD?1. Infección por TB y CMV2. Neoplasia por linfoma o leucemia3. Idiopática si después de 6m de estudio no hay causa

Page 288: ENARM  COMPENDIO

SEPSIS• Como se diagnostica el síndrome de respuesta inflamatoria sistémica?• R = Cuando se presentan 2 o mas de los siguientes parámetros:1. FIEBRE corporal > 38 o hipotermia < 362. TAQUIPNEA con FR > 24 x minuto3. TAQUICARDIA > 90 x minuto4. LEUCOCITOSIS > 12,000 o leucopenia < 4,000 o 10% de bandas

• Como diagnosticas sepsis grave?• R = Sepsis asociada a disfunción de un órgano con los siguientes datos:1. Cardiovascular: TAS < 90 mm Hg que responde a fluidoterapia2. Respiratoria: Relación FiO2/PaO2 < 2003. Renal: Diuresis < 0.5 ML/KG/HR durante 1 hr a pesar de fluidoterapia4. Trombocitopenia: < 80,000 o su reducción a la mitad con respecto a la obtenida 72 hrs

antes5. Acidosis metabólica: Con PH < 7.306. Choque séptico: Hipotensión con TAS < 90 durante 1 hr y que no responde a fluidoterapia

necesitando vasopresores

Page 289: ENARM  COMPENDIO

SEPSIS

• En caso de bacteriemia o sepsis cuales son algunas características distintivas de los agentes patológicos?

A. Neisseria meningitidis: Se acompaña de PURPURA O PETEQUIASB. Rikettsia: Cuando aparecen LESIONES PETEQUIALES por mordida de

garrapata en zona endémicaC. Pseudomona aureoginosa: ECTIMA GANGRENOSOD. S. Aureus o S. Pyogenes: ERITRODERMIA GENERALIZADA.

• Cual es la complicación mas frecuente de la bacteriemia?• R = SIRPA caracterizado por HIPOXEMIA E INFILTRADOS PULMONARES

DIFUSOS.

Page 290: ENARM  COMPENDIO

NEUROCISTICERCOSIS• Cual es el agente infeccioso en neurocisticercosis?• R = Tenia SOLIUM

• Cual es el cc de la neurocisticercosis?1) Forma activa: Sin evidencia del parasito con hallazgos en la TAC de

calcificaciones o hidrocefalia2) Parenquimatosa: Larvas en corteza y ganglios basales3) Forma subaracnoidea: Con fibrosis leptomeningea provocando

neuropatías por atrapamiento o hidrocefalia

• Cual es el método diagnostico de elección para le neurocisticercosis?• R = TAC de elección e IRM en casos dudosos.

• Cual es el manejo de la neurocisticercosis?• R = ALBENDAZOL 400 mg c/12 1 SEMANA, PRAZICUANTEL X 15 DÍAS. SE

RECOMIENDA EL USO DE PREDNISONA 1 DÍA ANTES DEL FÁRMACO Y CONTINUAR CON DISMINUCIÓN DE LA DOSIS 14 DÍAS DESPUÉS.

Page 291: ENARM  COMPENDIO

TETANOS

• Cual es el cuadro clínico del tétanos?• R = Comienza con ESPASMOS LEVES EN LA MANDÍBULA (trismo), el

cuello y la cara. La RIGIDEZ se desarrolla rápidamente en el TÓRAX, ESPALDA, MÚSCULOS ABDOMINALES y en ocasiones la laringe interfiriendo con la respiración. Los espasmos musculares son contracciones súbitas, fuertes y dolorosas.

• Cual es el tratamiento de elección para tétanos?• R = PENICILINA SÓDICA CRISTALINICA 20, 000 000. Se debe

administrar globulina inmunitaria antitetánica 5000 U/IM y una vez recuperado el paciente se da esquema completo.

Page 292: ENARM  COMPENDIO

CARBUNO O ANTRAX• Cual es el agente etiológico del Carbunco o ANTRAX?• R = BACILLUS ANTHRACIS, bacilo GRAM +

• La forma cutánea de carbunco o ántrax que es la mas común como se manifiesta?• R = Desarrollo de PÁPULA EN ZONA DE INOCULACIÓN, RODEADA DE VESÍCULAS, LA ZONA CENTRAL SE ULCERA Y SECA

DESARROLLÁNDOSE ZONA DEPRIMIDA DE COLOR NEGRO, NO DOLOROSO. Se resuelve espontáneamente pero 20% puede ser fatal.

• Como se manifiesta la forma intestinal de carbunco o ántrax?• R = FIEBRE, DISNEA, CIANOSIS, desorientación y signos de septicemia. Evoluciona rápidamente a choque, coma y muerte.

• Como se manifiesta el carbunco o ántrax respiratorio y quienes lo PADECEN FRECEUNTEMENTE?• R = Se manifiesta en personas que MANIPULAN PIEL Y LANAS. Se produce FIEBRE CON TOS NO PRODUCTIVA y mal estar

general, con buena evolución a 2-3 días y a continuación SÚBITAMENTE PRESENTA DIFICULTAD RESPIRATORIA GRAVE, CIANOSIS Y SEPTICEMIA FATAL CON MUERTE EN <24 HRS.

• Como realizas el diagnostico de carbunco o ántrax?• R = Identificación del bacilo con TINCIÓN GRAM, PCR.

• Cual es el manejo de carbunco o ántrax?• R = CIPROFLOXACINO, levofloxacino o PENICILINA G.

• Como previenes el carbunco o ántrax?• R = PROTECCIÓN DE PIEL, MUCOSAS Y LA VACUNACIÓN

Page 293: ENARM  COMPENDIO

SALMONELOSIS• Cuantos antígenos tiene la salmonella typhi?• R = 3: H FLAGELAR, K CAPSULAR y O SOMÁTICO

• Cuales son los sitios frecuentes de infección secundaria por salmonella typhi?• R = Hígado, bazo, la medula ósea, PLACAS DE PEYER DEL ÍLEON TERMINAL y la vesícula biliar.

• Cual es el cuadro clínico característico de salmonella typhi?• R = ROSÉOLA TIFOIDICA que se caracteriza por MACULAS ERITEMATOSAS DE 2-4 MM QUE BLANQUEAN A LA

PRESIÓN, localizadas en la parte superior del abdomen y tórax anterior y que por lo general dura de 2-3 días.

• Cual es la complicación mas frecuente de la salmonella typhi?• R = La mas común es la HEMORRAGIA GI que resulta de la erosión de un vaso de la pared intestinal secundaria a la

necrosis de las placas de peyer en el íleon terminal

• Cual es el medio diagnostico mas sensible en la primer semana?• R = HEMOCULTIVO, después el mielocultivo.

• Semana en la que se presentan las complicaciones de fiebre tifoidea?• R = Fines de SEGUNDA Y TERCERA SEMANA

• Con que inoculo de S. Tiphy se desarrolla la enfermedad?• R = 100,000

Page 294: ENARM  COMPENDIO

SALMONELOSIS• A que semana aparece la roséola tifoidica, la cual aparece a la digito presión?• R = 2da semana

• Como dx a un portador crónico de salmonella?• R = COPROCULTIVOS + DURANTE 1ª

• En que semana los px se perforan o tienen hemorragias con choque hipovolemico?• R = 3ERA semana

• En que semanas en la fiebre tifoidea se realizan los cultivos específicos?• R = 1era HEMOCULTIVO, 2da MIELOCULTIVO y 3era COPROCULTIVO/urocultivo

• Cual es el cuadro clínico de la salmonelosis?

• R = Fiebre elevada 2-3 semanas, dolor abdominal, DIARREA EN SOPA DE CHICHARO, ROSÉOLA, DELIRIO, ESPLENOMEGALIA.

• Cual es la prueba de laboratorio confirmatoria de fiebre tifoidea?

• R = 1:640 , LEUCOPENIA y cuadro clínico característico

Page 295: ENARM  COMPENDIO

SALMONELOSIS• Cual es el sitio donde suele albergarse la salmonella?• R = Vesícula biliar

• Que se presenta en la primera semana de la salmonelosis?• R = FIEBRE, HIPERSENSIBILIDAD ABDOMINAL, TIFLITIS (CUANDO HAY DOLOR EN FOSA ILIACA

DERECHA POR INVASIÓN DE LA PLACA DE PEYER Y CREPITA AL TACTO)

• Que se presenta en la segunda semana en la fiebre tifoidea?• R = DIARREA, estreñimiento, delirio, EXANTEMA EN CARA ANTERIOR DE TÓRAX QUE SEDE A LA

DIGITO PRESIÓN. El sistema retículo-endotelial se hipertrofia con HEPATO/ESPLENOMEGALIA, hiperplasia de las PLACAS DE PEYER. Bazo e hígado desarrollan nódulos tifoideos.

• Cuales son las complicaciones habituales de la fiebre tifoidea en la 3era semana?• R = HEMORRAGIA Y PERFORACIÓN INTESTINAL, peritonitis con placa simple de abd con aire

libre. Orquitis, meningitis, nefritis.

• Cual es el tratamiento de la salmonella multiresistente o en lugares o zonas endémicas?• R = CIPROFLOXACINO, levofloxacino, ceftriaxona

Page 296: ENARM  COMPENDIO

SALMONELOSIS

• Cual es el mejor método diagnostico en un paciente que se automedico en fiebre tifoidea?

• R = MIELOCULTIVO por que el medicamento no penetra a esa zona • Tratamiento para fiebre tifoidea?• R = CLORAMFENICOL 50/mg/kg por 2 SEMANAS

• Tratamiento para anemia + fiebre tifoidea?• R = AMOXICILINA o ampicilina, NO DARLE CLORAMFENICOL por ke

causa aplasia medular.

• Tx fiebre tifoidea en embarazada?• R = AMOXICILINA o ampicilina

Page 297: ENARM  COMPENDIO

SALMONELOSIS

• Tx fiebre tifoidea en niño con resistencia?• R = QUINOLONAS NO, cefotaxima o CEFTRIAXONA

• En que casos se utiliza la dexametasona en fiebre tifoidea?• R = En el ESTADO TIFOIDICO CON SHOQUE

• Que se le da al portador de salmonella?• R = CIPROFLOXACINA por 3 MESES

• Que tx das a un px portador crónico que no respondió a ciprofloxacino?

• R = COLECISTECTOMÍA, por que ahí vive la salmonella.

Page 298: ENARM  COMPENDIO

BRUCELOSIS• Cuales son los principales vectores de la brucelosis y a quienes afecta

comúnmente?• R= ORDEÑADORES, rastros, CARNICERO, veterinarios son vectores. DERIVADOS DE

LA LECHE como el queso o tejidos del animal afectado.

• Cuales son las cepas de brucella que produce enf en humano/ FIEBRE ONDULANTE?

• R = MELITERSIS DE CABRA + frecuente a nivel mundial, SUIZ DEL CERDO +, ABORTUS DE GANADO VOVINO +++.

• Microbiologicamente que es la brucella?• R = COCOBACILO GRAM -, crece a 37 grados, es inmóvil a pesar de TENER FLAGELO.

• Cual es la prueba de laboratorio para brucelosis?

• R = 2- MERCAPTOETANOL, es una inmunoglubulina IgG Vs BRUCELLA

Page 299: ENARM  COMPENDIO

BRUCELOSIS• Cual es el medio de cultivo de elección para brucelosis?• R = RUIZ CASTAÑEDA medio doble y ROSA DE BENGALA (fines epidemiológicos)

• Que tratamiento utilizas de primera elección contra brucella?• R = Combinado DOXICICLINA + GENTAMICINA de 3-6 semanas.

• Que tx utilizas en brucelosis en hueso y SNC?• R = DOXICICLINA Y RIFAMPICINA

• Cual es el tratamiento de brucelosis en embarazada?• R = TMP/SMZ+ RIFAMPICINA + ACIDO FÓLICO

• Cual es el tratamiento de niños menores de 12ª con brucelosis?• R = TMP/SMZ + RIFAMPICINA

• Cual es el tratamiento en pacientes alérgicos a sulfas en brucelosis?• R = RIFAMPICINA Y CEFTRIAXONA.

Page 300: ENARM  COMPENDIO
Page 301: ENARM  COMPENDIO

BOTULISMO

• Cual es el cuadro clínico del CLOSTRIDIUM BOTULINUM transmitido por alimentos?

• R = Los síntomas comienzan en 6 HRS DESPUÉS A 2 SEMANAS, se manifiesta con DIPLOPÍA, VISIÓN BORROSA, PTOSIS, disfagia, sensación de sequedad de mucosa oral, DEBILIDAD MUSCULAR que afecta únicamente a los hombros, miembros torácicos y mas tarde a los pélvicos.

• Cual es el manejo?• R = Administración de TOXINA BOTULÍNICA con PREVENCIÓN DE

INSUFICIENCIA RESPIRATORIA que amerite ventilación mecánica

Page 302: ENARM  COMPENDIO

PESTE BUBONICA• Cual es el agente causal por la peste bubónica?• R = YERSINIA PESTIS

• Cual es el cuadro clínico de la peste bubónica?• R = Es la forma mas frecuente, se transmite por la PICADURA DE PULGA. Inicio con FIEBRE ALTA, nauseas,

MIALGIAS y ADENOPATÍAS DOLOROSAS REGIONALES, el PACIENTE EVOLUCIONA A ESTUPOR, COMA Y MUERTE.

• Que formas clínicas tiene la peste?• R = Peste bubónica, septicémica y neumónica

• Como diagnosticas peste?• R = BIOPSIA DE GANGLIOS o EXUDADO FARÍNGEO

• Cual es el tratamiento de la peste?

• R = ESTREPTOMICINA, CLORANFENICOL O TETRACICLINAS.

• Cual es la prevención de la peste?• R = Hay una vacuna con CEPA INACTIVADA F1

Page 303: ENARM  COMPENDIO

RABIA• Cual es el agente causal de la rabia?• R = Virus de la familia Rabdovirus

• Cual es el cuadro clínico de la rabia?• R = Dolor en el sitio de la mordedura, fiebre, malestar general, nausea y vomito. Diez días mas

tarde hay manifestaciones del SNC pudiendo ser encefálica o paralitica y las 2 formas evolucionan a coma - muerte.

A. ENCEFALICA: Se caracteriza por delirio, somnolencia, hidrofobia (espasmos laríngeos dolorosos al beber agua)

B. PARALITICA: Da una parálisis ascendente.

• Cual es el manejo de la rabia?I. Es necesario observar al animal los siguientes 10 DÍAS tomando UNA BIOPSIA DEL CUERO

CABELLUDO y analizarla con Ac fluorescente o PCR.II. Atención de la LESIÓN LAVANDO CON AGUA Y JABÓN ABUNDANTE A CHORRO DURANTE 10

MINUTOS, para MUCOSA ORAL O NASAL CON SOLUCIÓN FISIOLÓGICA DURANTE 5 MINUTOS. Desinfectar la herida con agua oxigenada o tintura de yodo.

III. Si se requiere SUTURAR LA HERIDA se debe APLICAR PRIMERO INMUNOGLOBULINA ANTIRRÁBICA HUMANA y se aproximan los bordes o se dan puntos temporales en caso que la herida sea profunda,

IV. En exposición leve se administra vacunación antirrábica en la región deltoidea los días 0-3-7-14 Y 28.

V. Se APLICARA INMUNOGLOBULINA INTRALESIONAL LA PRIMER ½ Y LA OTRA MITAD VÍA IM.VI. Se usara SUERO HETEROLOGO EN CASO EXTREMO DE NO CONTAR CON INMUNOGLOBULINA.

Page 304: ENARM  COMPENDIO

INFLUENZA

Page 305: ENARM  COMPENDIO
Page 306: ENARM  COMPENDIO

DENGUE Y FIEBRE AMARILLA• Cual es el mosquito transmisor del dengue?• R = AEDES AEPYPTI.

• Cual es el cc del dengue?1) El DENGUE CLÁSICO se inicia con FIEBRE, BRADICARDIA, pulso lento, EXANTEMA MACULAR

PUNTIFORME, tos, ardor faríngeo, MIALGIAS, ARTRALGIAS. 2) El DENGUE HEMORRÁGICO se caracteriza por aumento en la permeabilidad vascular con

clasificación del I-IV.

• Que es lo que distingue a la fiebre amarilla del dengue laboratorialmente?• R = CUERPOS DE INCLUSIÓN VIRAL llamados CUERPOS DE COUNCILMAN EN LA SANGRE.

• Cual es el cuadro clínico de la fiebre amarilla?• R = FIEBRE, cefalea, mialgias, DOLOR LUMBOSACRO, ERITEMA OCULAR Y FACIAL, PETEQUIAS EN

ENCIAS Y MUCOSA NASAL, HEMATEMESIS, MELENA, ALBUMINURIA, encefalitis o meningoencefalitis.

• Cual es el manejo de la fiebre amarilla y del dengue?• R = SINTOMÁTICO

Page 307: ENARM  COMPENDIO

VIH• Que células ataca el VIH?• R = CD4

• Cual es el ciclo vital del VIH?• R = A través de la proteína gp41, se realiza la fusión, posteriormente la RNA del VIH se descubre e

interna en la célula afectada, la enzima transcriptasa inversa del virion cataliza la transcripción inversa del RNA en DNA, este se transfiere hacia el núcleo en el cual se integra en los cromosomas por medio de la integrasa. Los macrófagos actúan como reservorio del VIH y lo diseminan a otros sistemas

• Que se debe realizar como prevención en una persona que halla sido puncionada de manera accidental con una aguja utilizada previamente en un paciente con VIH?

• R = INHIBIDORES NUCLEOSIDOS DE LA TRANSCRIPTASA REVERSA

• Como se manifiesta el síndrome retroviral agudo SRA en VIH?• R = SÍNTOMAS SIMILARES AL RESFRIADO O MONONUCLEOSIS, fiebre, escalofríos, sudores nocturnos

y erupciones en la piel que dura de 1-3 SEMANAS.

• De acuerdo a la CDC como se realiza dx de SIDA?• R = Recuento <200/MM DE CD4 + la presencia de ENFERMEDAD OPORTUNISTA

Page 308: ENARM  COMPENDIO

VIH• Como dx VIH?• R = ELISA y confirmatorio con WESTERN BLOOD

• Cuales son parte del grupo de fármacos que inhiben la TRANSCRIPTASA INVERSA en VIH?• R = ZIDOVUDINA, didanosina, zalcitabina.

• Cuales son las indicaciones actuales para uso de tratamiento retro vírico en VIH?1. Síndrome de infección aguda2. INFECCIÓN CRÓNICA: Enfermedad sintomática o asintomática con T CD4 <350/NL o RNA DEL

VIH CON > 50, 000 COPIAS3. PROFILAXIS POST- EXPOSICIÓN: Se recomienda utilizar combinaciones como ZIDOVUDINA +

LAMIVUDINA + NEVERIPINA

• Cuales son las indicaciones para cambiar el tratamiento antirretrovirico?1. Disminución <1 log del RNA del VIH con plasma a las 4 semanas de haber iniciado el

tratamiento2. Incremento DEL TRIPLE O MAS DEL VALOR PLASMÁTICO DEL RNA NO ATRIBUIBLE A INFECCIÓN

concomitantes o vacuolas.3. DISMINUCIÓN DEL PORCENTAJE DE CÉLULAS TCD44. Deterioro clínico5. Efectos adversos

Page 309: ENARM  COMPENDIO

VIH• Como se manifiesta la neumonía por Pneumocystis Jiroveci?

• R = CD4 < 200/MM mas AUMENTO DE LA DHL, el diagnostico definitivo se obtiene por la TINCIÓN DE ESPUTO CON

WRIGHT-GIEMSA

• Cual es la causa mas frecuente de retinitis en pacientes con VIH?• R = CITOMEGALOVIRUS

• Mecanismo por el que el VIH penetra a la célula?• R = Pinocitosis

• Cual es la neoplasia maligna mas común en pacientes con VIH?• R = Sarcoma de Kaposi

• Cual es el manejo del Sarcoma de Kaposi?

• R = Para la FORMA CUTÁNEA SE DA DOXORRUBICINA, para el INTESTINAL O VISCERAL SE DA

DOXORRUBICINA, BLEOMICINA Y VINBLASTINA.

• Como se maneja la profilaxis en pacientes con VIH de acuerdo al conteo de las células CD4?

1) CD4 < 200: VS P. JIROVECI con TMP/SMZ 1Tab/24 hrs C/3 DIA

2) CD4 75-100: VS M. AVIUM con CLARITROMICINA y se SUSPENDE CUANDO AUMENTAN LOS CD4

3) CD4 < 50: VS CITOMEGALOVIRUS con GANCICLOVIR4) PPD con induración > 5 MM iniciar profilaxis VS M. TUBERCULOSIS con ISONIACIDA a 300 mg al día +

PIRIDOXINA durante 9-12 MESES.

Page 310: ENARM  COMPENDIO

VIH• Tratamiento de primera elección para la retinitis por CITOMEGALOVIRUS?• GANCICLOVIR

• Tratamiento para el herpes tipo 2 en pacientes con VIH?• ACICLOVIR

• De que manera se puede inhibir el síndrome de desgaste?• TALIDOMIDA

• Agente infeccioso de la lengua vellosa?

• Virus de EPSTEIN BARR ( relacionada con virus de mononucleosis y linfomas) VIRUS TIPO 4

• Sitio mas común de lesión dermatológica, “SARCOMA DE KAPOSI” manchas color de piel?• El sitio de afectación mas frecuente es la CARA EN PARPADOS Y PUENTE NASAL RELACIONADO CON EL

VIRUS TIPO 8

• Tratamiento de toxoplasmosis?• PIREMITAMINA o sulfas + CLINDAMICINA en dosis altas

Page 311: ENARM  COMPENDIO

VIH• Posibilidad de contagio por coito anal receptivo en el VIH?• 1:100 1:30

• Zona donde se alojan el virus de la inmunodeficiencia es alta y útil para seguimiento del tx de VIH?• Biopsia de mucosa rectal otra opción es PCR

• Transmisión perinatal del VIH?: • 13% al 40%

• Coito con inserción anal: 1: 1,000• Transfusión: 1: 100,000• Coito inserción vaginal: 1: 10,000 varia si la mujer se encuentra en periodo menstrual o si tiene una

ETS

• Aguja infectada?: • 1-300

• Compartir agujas?: • 1-150 (drogadictos)

• A que se refiere la regla de los 3 con relación a aguja infectada?• Hepatitis B : 30% Hepatitis C: 3% HIV: .3%

Page 312: ENARM  COMPENDIO

VIH• Diagnostico para valorar el tratamiento “respuesta terapéutica”• RCP se realiza al mes y medio y a los 3 meses y posteriormente CADA 6 MESES (no se usa para diagnostico)

• Que pasa con una persona que tiene VIH y las siguientes enfermedades Concomitantes?• Hepatitis B: Indiferente (no pasa nada)• Hepatitis C: Agresividad• HEPATITIS G: FRENA LA PROGRESIÓN DEL VIH

• Con respecto a la estatificación de etapas A-B-C y grados I al III?• A: asintomáticos B: Síntomas Constitucionales (fiebre, perdida de peso, diarrea) C: Infecciones Oportunistas

(neumocistis jirovecci, linfoma del SNC, Criptosporidiasis, Toxoplasma del SNC, Micovacterium Avium intra celular)• I.- 500 copias II.-MAS DE 200 Y MENOS DE 500 III.-MENOS DE 200 copias

• INHIBIDORES DE TRANSCRIPTASA INVERSA -no análogo de los Nucleosidos- NNRTI• De la ZIDOVUDINA AZT cual es el efecto indeseable? PROBLEMAS MEDULARES, anemia severa, trombocitopenia,

leucopenia

• DE LA EFEVIRENZ cual es el efecto indeseable? • Insomnio y pesadillas (DEPRESIÓN)

• INHIBIDORES DE PROTEASAS • Del Indinavir efecto indeseable mas frecuente: FORMADOR DE CÁLCULOS cólico renoureteral• Ritonavir + lopinavir : Es el caletra efecto sinérgico

Page 313: ENARM  COMPENDIO

VIH• Profilaxis (exposición en trabajadores de la salud)(exposición SEXUAL) que maniobras o

procedimientos?• Tiene una posibilidad de infección de 1:300 SE REALIZA UN ELISA con fines legales, ( AL MES)… si se

considera EXPOSICIÓN DE BAJO RIESGO dar AZT+3TC X UN MES se REALIZA ELISA SI ES NEGATIVO SE SUSP… si es exposición de alto riesgo AZT+3TC +UN TERCER MEDICAMENTO…. NO DAR TX A LAS 72 POSTEXPOSICION

• Copias de CD4?• Entre 50 y 100 copias en el momento del dx: MVAIC, CMV, Linfomas del SNC• MENOS DE 200: Criptocococis, toxoplasmosis, NEUMOCISTIS JIROVECCI• Entre 200 y 500: LENGUA VELLOSA, Sarcoma de Kaposi, TB Pulmonar, Infección por virus herpes,

candidiosis

• Px con HIV y tiene exposición con px con tb?• Profilaxis: PPD MAS DE 5MM ISONIACIDA + RIFAMPICINA + PIRAZINAMIDA POR 12 MESES +

PIRIDOXINA

• En que consiste el síndrome retroviral agudo?1) Paciente con infección de VIH con SÍNTOMAS A CORTO PLAZO (de 2 A 3 SEMANAS y sucede en el

40%de los casos) Y CONSISTE EN FIEBRE ADENOPATÍA, RASH CUTÁNEO COLOR SALMON Y HEPATOMEGALIA… CUADRO MUY PARECIDO AL DE LA MONONUCLEOSIS INFECCIOSA,

2) DX ELISA A LAS 6 SEMANAS Y 3 MESES si sale positivo se realiza el WESTERN BLOT PARA CONFIRMARLO

Page 314: ENARM  COMPENDIO

VIH• Combinaciones aprobadas:I. Tenofovir (TDF)+ emtricitabina (FTC) y efavirenz (EFV)II. Atazanavir (ATV) + ritonavir (rtv)+tenofovir (TDF) / emtricitabina (FTC) (si no funciona el primer

esquema)III. Caletra +tenofovir (TDF) / emtricitabina (FTC) (El caletra se puede cambiar con AZT + 3TC)

• Efectos colaterales?I. Zidovudina….. ANEMIA por hipoplasia medular II. Nerfinavir…. DIARREAIII. Didanosina…. PANCREATITIS IV. Zalcitabina….. NEUROPATÍA PERIFÉRICAV. Estaduvidina…. NEUROPATÍA PERIFÉRICA (lamivudina +zidovudina) 3TC….. Neuropatia periféricaVI. Abacavir….. FIEBRE RASH, HIPERSENSIBILIDAD FATAL VII. Tenofovir…. TOXICIDAD RENALVIII. Saquinavir…. LÍPIDOS ELEVADOS

• Px con embarazo + VIH Cual es el tx?

I. ZIDOVUDINA (2do y 3er trimestre)… se continua en el trabajo de parto (para evitar la infección vertical)

II. POSPARTO (NO LACTANCIA 10-20 tiene posibilidad de infectar) 2da opción LAMIVUDINA

Page 315: ENARM  COMPENDIO

VIH

• Cuales son parte del grupo de fármacos INHIBIDORES DE PROTEASA en VIH?

• R = SaquinAVIR, ritonAVIR, indinAVIR.

• Cuales son parte del grupo de fármacos INHIBIDORES DE LA FUSIÓN?• R = Enfubitida, que entre sus efectos adversos se encuentra la

hipersensibilidad y neumonía bacteriana

• Cuales son parte del grupo de fármacos que INHIBEN LA TRANSCRIPTASA INVERSA en VIH?

• R = ZIDOVUDINA, didanosINA, zalcitabINA.

Page 316: ENARM  COMPENDIO
Page 317: ENARM  COMPENDIO
Page 318: ENARM  COMPENDIO
Page 319: ENARM  COMPENDIO

ASPERGILOSIS• Como se adquiere la aspergilosis?• R = Por inhalación de ESPORAS DEL HONGO, las cuales se encuentran en HOJAS SECAS, GRANEROS

DE MAIZ, ESTIÉRCOL Y VEGETALES EN DESCOMPOSICIÓN.

• En los pacientes con rinitis crónica por aspergilus, que datos de laboratorio encuentras en el moco?

• R = Rico en EOSINOFILOS y cristales de CHARCOT-LEYDEN

• En que consiste la aspergilosis pulmonar endobronquial saprofita o aspergiloma?• R = En el CRECIMIENTO DEL HONGO dentro de las cavidades pulmonares, SECUNDARIAS

GENERALMENTE A TB, SARCOIDOSIS, HISTOPLASMOSIS O BRONQUIECTASIAS.

• Como dx aspergilus?• R = CULTIVO DE SABOURAD, detección de ANTICUERPOS, galactomanano en suero (chbts en su

pared celular) y biopsia

• Cual es el tratamiento de aspergilus?• R = Esteroides, ANFOTERICINA B, LOBECTOMÍA EN CASO DE ASPERGILOMA o debridacion de

tejido infectado.

• Cual es el tratamiento de elección en caso de toxicidad secundaria al DESOXICOLATO de ANFOTERICINA B en caso de aspergilosis invasiva?

• R = VORICONAZOL 6mg/kg/dia.

Page 320: ENARM  COMPENDIO

CANDIDOSIS• Como se manifiesta la CANDIDIASIS DIGESTIVA y como se trata?• R = Se ASOCIA a lesiones ORALES Y PERIANALES, produce DIARREA CON MOCO, el

diagnostico se realiza con el EXAMEN DIRECTO DE LAS HECES CON KOH y se trata con NISTATINA ORAL.

• Como se manifiesta la CANDIDIOSIS URINARIA y como se trata?• R = Se presenta como CISTITIS, pielonefritis, BOLAS FÚNGICAS RENALES, ABSCESOS RENALES

o necrosis de las papilas. Se trata con fluconazol o ANFOTERICINA B.

• Como se diagnostica CANDIDIOSIS PULMONAR y como se trata?

• R = Cuando realizas HEMOCULTIVOS POSITIVOS o ANTÍGENO MANAN DEL SUERO. El tratamiento se realiza con ANFOTERICINA B, en los pacientes que no la toleran se utiliza FLUCONAZOL.

• Como se manifiesta la ENDOCARDITIS por candidiosis?• R = FIEBRE persistente, SOPLOS CARDIACOS y ESPLENOMEGALIA, en algunos casos hay

embolización a grandes arterias, como la iliaca o femoral. LA VÁLVULA MAS AFECTADA ES LA MITRAL.

Page 321: ENARM  COMPENDIO

CANDIDOSIS

• Como diagnosticas ENDOCARDITIS por cándida?• R = Ecocardiagrafia, hemocultivos seriados y detección de

ANTÍGENO MANAN en suero?

• Cual es el tratamiento de endocarditis por cándida?• R = RESECCIÓN DE VÁLVULA afectada y ANFOTERICINA B

• Como se presenta la candidiasis diseminada aguda o sepsis?• R = Se presenta en PACIENTES NEUTROPENICOS, CON FIEBRE,

LEUCOCITOSIS, TROMBOCITOPENIA, inestabilidad hemodinámica y choque.

Page 322: ENARM  COMPENDIO

CRIPTOCOCOCIS (AVES, PALOMAS)

• Cual es el agente causal de la criptococosis y donde se alberga?• R = CRYPTOCOCCUS NEOFORMANS, se encuentra en HECES DE

PALOMAS (CUIDADORES DE AVES) O NIDOS

• Como diagnosticas criptococosis?

• ANTÍGENO MANAN en suero o biopsia. • En LCR con TINCIÓN DE TINTA CHINA

• Como se trata la criptococosis?• R = ANFOTERICINA B y 5-fluorocitosina

Page 323: ENARM  COMPENDIO

HISTOPLASMOSIS (CAVERNAS)

• Donde se alberga el Histoplasma capsulatum?• R = En forma de moho en suelo RICO EN NITRÓGENO, como el de los

GALLINEROS, o donde duermen las aves y CAVERNAS DE MURCIÉLAGOS

• Como diagnosticas histoplasmosis?• R = El método de elección es el CULTIVO, pero tarda mucho y haces

frotis de los fluidos o tejidos infectados, las muestras para cultivo en HISTOPLASMOSIS DISEMINADA SE OBTIENEN DE MEDULA ÓSEA, HÍGADO, GANGLIOS LINFÁTICOS O LESIONES MUCOCUTANEAS.

• Cual es el manejo de la hispoplasmosis?• R = Anfotericina B

Page 324: ENARM  COMPENDIO

MUCORMICOSIS• Cual es el agente etiológico de la mucormicosis?• R = MUCOR, ABSIDIA, RIZOPUS denominado ficomicetos.

• Como se manifiesta la infección de los senos paranasales en mucormicosis?

• R = Predominantemente a PACIENTES CON DM mal tratada, con FIEBRE, congestión nasal, SECRECIÓN NASAL SANGUINOLENTA, DOLOR SINUSAL Y DIPLOPÍA, a la exploración física se observa DISMINUCIÓN DE LOS MOVIMIENTOS OCULARES, quemosis y PROPTOSIS y SI INVADE EL SENO FRONTAL PRODUCE COMA.

• Como se manifiesta la mucormicosis gastrointestinal?• R = Se disemina al peritoneo produciendo la muerte en 70 días

• Como se diagnostica mucormicosis?• R = FROTIS EN FRESCO donde se ven HIFAS HIALINAS o en BIOPSIAS donde se observa

NECROSIS ISQUÉMICA o hemorrágica

• Cual es el manejo de la mucormicosis?• R = El tratamiento consiste en altas dosis de ANFOTERICINA B, ASEO QUIRÚRGICO DE LOS

TEJIDOS.

Page 325: ENARM  COMPENDIO

COCCIDIOIDOMICOSIS• Cuales son los signos y síntomas de la coccidioidomicosis?• R = TOS, RINORREA, febricula, dolores musculares y en la RADIOGRAFÍA DE

TÓRAX CON CAVITACIÓN.

• - A 43-year-old man developed a cough shortly after returning from a 1-month hiking trip in California. While there, he was hiking in the central California valleys. During his trip, he had developed a “flu-like” illness consisting of fever, cough, and muscle pains, which resolved spontaneously. A CXR shows a thin-walled cavity in the right upper lobe, and the sputum reveals fungal elements. Which of the following is the most likely causative organism?

• (A) ringworm• (B) Cryptococcus neoformans• (C) Candida albicans• (D) mycobacteria• (E) coccidioidomycosis

Page 326: ENARM  COMPENDIO

BLASTOMICOSIS• Cual es el agente causal de blastomicosis?• R = BLASTOMYCES DERMATITIDIS

• Cuales son las manifestaciones clínicas de blastomicosis?• R = PULMONAR Y CUTÁNEA. NEUMONÍA aguda que no responde al tratamiento y CURA

ESPONTÁNEAMENTE. Las LESIONES CUTÁNEAS predominan en las ZONAS EXPUESTAS, característicamente son PAPULAS, NÓDULOS O PLACAS BIEN DELIMITADAS, únicas o múltiples, NO DOLOROSAS NI PRURIGINOSAS, que evolucionan a lesiones verrugosas, costras o ulceras.

• Como diagnosticas blastomicosis?• R = De ELECCIÓN CULTIVO pero tarda mucho, se pueden identificar organismos con la

TINCIÓN DE PLATA, METENAMINA O ACIDO PERIODICO DE SCHIFF en la biopsia de los tejidos infectados o en la citología de esputo. Donde se observan CÉLULAS LEVADURIFORMES CON YEMAS DE BASE AMPLIA.

• Cual es el tratamiento de blastomicosis?• R = En la forma LEVE - ITRACONAZOL, en la forma GRAVE - ANFOTERICINA B

Page 327: ENARM  COMPENDIO
Page 328: ENARM  COMPENDIO

PALUDISMO• Cual es el agente causal de paludismo o malaria?

• R = PARASITOSIS intracelular eritrocitaria por PLASMODIUM, que se transmite por el

MOSQUITO ANOPHELES

• Cuantas especies de plasmodium existen?• R = 4, P. falciparum, P. vivax, P. ovale y P. malariae

• Cual es el ciclo biológico del plasmodium?• R = Los mosquitos se infectan al ingerir sangre infectada con micro y macrigametocitos.

Dentro del mosquito, se multiplica sexualmente en su intestino, produciendo los ooquistes, y al madurar se rompen liberando ESPOROZOITOS, que migran a las glándulas salivales del mosquito y son la forma infectante en los seres humanos. Ya en el TORRENTE SANGUÍNEO viajan a hígado donde se reproducen y forman los MEROZOITOS que tienen la capacidad de infectar los eritrocitos.

• Que produce la lisis eritrocitaria al contraer paludismo?• R = La PLASMEPSINA, que es una proteasa del acido aspartico que DEGRADA LA

HEMOGLOBINA.

Page 329: ENARM  COMPENDIO

PALUDISMO• Cual es el tipo de genero en paludismo mas frecuente?• R = P. FALCIPARUM, es la mas frecuente y de peor pronostico con fiebre >

40

• Como se diagnostica el paludismo?• R = OBSERVACIÓN DE PARASITO EN FROTIS SANGUÍNEO O GOTA

GRUESA. La TINCIÓN DE GIEMSA permite observar las granulaciones de Schuffner.

• Como se trata el paludismo?• R = QUININA Y CLOROQUINA. El SULFATO DE QUININA se usa en

RESISTENCIA A CLOROQUINA

Page 330: ENARM  COMPENDIO

BORRELIOSIS O ENFERMEDAD DE LYME

• Cual es el agente causal de la enfermedad de Lyme?

• R = Espiroqueta BORRELIA BURGDORFERI GRAM (-) de color violeta con tinción Giemsa que se CULTIVA EN MEDIO NOGUCHI. Se adquiere por MORDEDURA DE GARRAPATA del genero IXODES.

• Cual es el cuadro clínico de la enfermedad de Lyme?• R = Se divide en 3 etapasI. ERITEMA CRÓNICO O MIGRANS, pápulas eritematosas que se EXTIENDEN DE

FORMA CENTRIFUGA DEJANDO LA PARTE CENTRAL DEL CUERPO SANA, acompañada de síntomas generales como astenia, adinamia, fiebre, mialgias y cefalea.

II. Alteraciones neurológicas, cardiacas y reumáticasIII. POLIARTRITIS MIGRATORIA, alteraciones dermatológicas y neurológicas

• Como se diagnostica la enfermedad de Lyme?• R = ELISA o PCR

Page 331: ENARM  COMPENDIO

BORRELIOSIS O ENFERMEDAD DE LYME

• Tratamiento de elección para enf de Lyme?• R = DOXICICLINA por mes y medio

• Cual es el tratamiento para borreliosis en su etapa precoz?• R = DOXICICLINA.

• Cual es el tratamiento para borreliosis en su etapa tardía?• R = CEFTRIAXONA

Page 332: ENARM  COMPENDIO

LEPTOSPIROSIS• Cual es el vector de la leptospirosis, que enzimas produce y que nutrientes necesita?• R = Leptospira interrogans en ORINA DE RATA. PRODUCE CATALASA Y HIALURODINASA y requieren TIAMINA

B1 Y VITAMINA B12 PARA SU DESARROLLO

• Cuales son las manifestaciones clínicas de la Leptospirosis?• R = De inicio súbito, con FIEBRE, derrame conjuntival, epistaxis, MIALGIAS, cefalea intensa, EXANTEMA Y

HEPATOMEGALIA.

• Que es el síndrome de Weil en Leptospirosis?• R = Al cursar la primera fase, viene una ASINTOMÁTICA para posteriormente PRESENTARSE LEPTOSPIROSIS

ANICTERICA.

• Como se manifiesta el síndrome de Weil?• R = CON FIEBRE, CEFALEA, MIALGIAS, EXANTEMA, miocarditis, ICTERICIA, insuficiencia hepática,

INSUFICIENCIA RENAL, FENÓMENOS HEMORRÁGICOS, LEPTOSPIURIA y daño al SNC (MENINGITIS ASEPTICA).

• Cual es la triada clásica del síndrome de Weil?• R = Daño en HÍGADO, RIÑÓN y SNC.

• Cual es el tratamiento de la Leptospirosis?• R = PENICILINA, estreptomicina o tetraciclinas

Page 333: ENARM  COMPENDIO

TULAREMIA• Cual es el agente causal de Tularemia?

• R = FRANCISELLA TULLARENSIS POR PICADURA DE GARRAPATA

• Cual es el cuadro clínico de Tularemia?• R = De inicio súbito con FIEBRE, ASTENIA, ADINAMIA, hiporexia, ataque al estado

general, posteriormente hay diversas formas clínicas: ULCEROGANGLIONAR, manifestada con adenomegalias y ulceras cutáneas; TIFOIDE GLANDULAR, donde se afecta el pulmón, ganglios, hígado y bazo. SE PRESENTA CON NEUMONÍA, NECROSIS VISCERAL Y SEPSIS.

• Como diagnosticas Tularemia?

• R = En medio de CULTIVO DE AGAR SANGRE cisteína o ELISA.

• Como tratas la Tularemia?

• R = ESTREPTOMICINA, tetraciclina.

Page 334: ENARM  COMPENDIO
Page 335: ENARM  COMPENDIO

RICKETSIA

Page 336: ENARM  COMPENDIO

FIEBRE Q

Page 337: ENARM  COMPENDIO

LEISHMANIASIS• Que vectores transmiten leishmaniasis?• R = MOSQUITOS del genero LUTZOMYA Y PHLEBOTOMUS, LEISHMANIA pertenece a la familia del

tripanosoma

• Cual es el ciclo biológico de la leishmaniasis?• R = Al picar el mosquito a un mamífero infectado, ingiere los macrófagos parasitados, que posteriormente

se destruyen, dejando libre al parasito en su forma flagelar e infectante (promastigote), se reproducen por fision binaria longitudinal, y migran hacia la porción anterior del aparato digestivo, hasta la probocide (boca). La infección se produce al alimentarse de un nuevo huésped, inocula los PROMASTIGOTES, que al ser fagocitados se transforman en AMASTIGOTES, donde se multiplican, causan la lisis de las células que los contienen e infectan a otras células.

• En la leishmaniasis viceral o Kala-Azar a que células afecta el parasito?• R = A los MACRÓFAGOS del sistema retículo endotelial

• Como se manifiesta la leishmaniosis cutánea?• R = En el sitio de inoculación, PAPULA ERITEMATOSA/PRURIGINOSA – NODULO – ULCERA PIEL/TEJ. CEL.

SUB – EXUDA LIQ. SEROHEMATICO – ADENOPATÍA REGIONAL.

• Como se manifiesta la leishmaniosis mucocutanea o espundia?• R = ULCERA CUTÁNEA – pasan years – LESIÓN OBSTRUCTIVA ÚNICA/MULTIPLE – LARINGE/TABIQUE

NASAL/ANO Y VULVA.

Page 338: ENARM  COMPENDIO

LEISHMANIASIS• Como se manifiesta la leishmaniasis viceral o Kala-Azar (fiebre negra)?• R = Hay HIPERPIGMENTACIÓN CUTÁNEA DE LAS EXTREMIDADES, mal estar general, DOLOR ABDOMINAL, HEPATO-

ESPLENOMEGALIA, LINFADENOPATIAS, DIARREA es fatal si no se trata adecuadamente.

• Como diagnosticas leishmaniasis?• R = VER EL AMASTIGOTE EN BIOPSIAS DE PIEL, lesiones mucosas, hígado, ganglios linfáticos, medula ósea tenidas con

Giemsa. En los CULTIVOS de los tejidos se busca el PROMASTIGOTE.

• Con que prueba de laboratorio diagnosticas leishmaniasis?• R = INTRADERMORREACCIÓN DE MONTENEGRO y ELISA.

• Cual es el tratamiento de la leishmaniasis viceral?

• R = Para la FORMA VISCERAL el tratamiento DE ELECCIÓN ES ESTIBOGLUCONATO DE SODIO O ANTIMONIATO DE GLUTAMINA IM POR 50 DÍAS.

• Que medicamentos empleas en caso de que la leishmaniasis visceral no responda a los antiamoniales?• R = ANFOTERICINA LIPOSOMAL POR 5 DÍAS Y EL ISETHIONATO DE PENTAMIDINA.

• Que medicamento utilizas en leishmaniasis cutánea?

• R = ISETIONATO DE PENTAMIDINA, paromomicina APLICACIÓN TÓPICA 2 VECES AL DÍA POR 15 DÍAS.

Page 339: ENARM  COMPENDIO
Page 340: ENARM  COMPENDIO

GIARDIASIS

• Como diagnosticas giardiasis?• R = Observación de QUISTE EN HECES, ELISA.

• Que componente de los trofozoitos de giardiasis te causa hipotrofia de las vellosidades intestinales

• R = El DISCO SUCTOR

• Cual es el tratamiento de elección para la giardiasis?• R = El TINIDAZOL con dosis única de 2 gr. METRONIDAZOL. La

furazolidona causa hemolisis en pacientes con déficit de 6-GDP.

Page 341: ENARM  COMPENDIO

AMEBOSIS• Cual es el cuadro clínico abdominal de la amebiasis por E. Hystolitica?1. COLITIS AMEBIANA AGUDA manifestada por dolor abd tipo cólico, tenesmo y EVACUACIONES

DIARREICAS CON MOCO Y SANGRE. 2. COLITIS FULMINANTE con mortalidad mayor al 50% se presenta con diarrea sanguinolenta intensa,

ataque al estado general, fiebre, dolor abdominal intenso y PERFORACIÓN INTESTINAL con choque séptico secundarios a la extensión de la ulcera hasta la serosa.

3. AMEBOMA O GRANULOMA AMEBIANO que cursa con DIARREA SANGUINOLENTA Y OBSTRUCCIÓN INTESTINAL.

• Cual es el cuadro clínico de la amebosis cutánea?1. ULCERAS FUGEDENICAS (de COLOR PURPURA, de crecimiento rápido, MUY DOLOROSAS y

ABUNDANTE MATERIAL NECRÓTICO), 2. LESIONES VEGETANTES (friables, de evolución subaguda, LOCALIZADAS EN LOS PLIEGUES

CUTÁNEOS), 3. ENTAMOEBOSIS (se presenta por una reacción de hipersensibilidad, CON LESIONES SEMEJANTES A

URTICARIA.

• Como se manifiesta el absceso hepático amebiano?• R = De inicio agudo, CON HEPATOMEGALIA Y DOLOR EN HIPOCONDRIO DERECHO QUE SE IRRADIA

HACIA EL HOMBRO.

Page 342: ENARM  COMPENDIO

AMEBOSIS• Como diagnosticas amibiasis?• R = COPROPARISOTOSCOPICO EN FRESCO, hematoxilina férrica o TINCIÓN

DE GOMORI, en busca de TROFOZOITOS, ELISA. En la piel se busca trofozoito.

• Cual es el tratamiento de amibiasis?1) METRONIDAZOL, TINIDAZOL. 2) Dehidrohemetina y emetina son amebicidas de la pared intestinal, pero

no sobre la luz intestinal siendo estos YODOQUINOL y paramomicina.

• Que medicamento es eficaz para la amebiasis hepática?• R = CLOROQUINA

Page 343: ENARM  COMPENDIO

TRIPANOSOMIASIS AFRICANA O ENFERMEDAD DEL SUENO

• Cual es el agente etiológico de la TRIPANOSOMIASIS AFRICANA O ENFERMEDAD DEL SUEÑO y cual es su vector?

• R = Tripanosoma BRUCEI, Tripanosoma RHODESIENSE y Tripanosoma GAMBIENSE. Su

vector es la MOSCA TSE TSE.

• Cual es el agente causal de la tripanosomiasis americana o ENFERMEDAD DE CHAGAS y cual es su vector?

• R = TRIPANOZOMA CRUZI y su vector es la chinche

• Cual es el ciclo biológico de los tripanosomas?• R = Al alimentarse de un individuo con la infección, los vectores ingieren el amastigote,

que se multiplica dentro del tracto digestivo y las FORMAS INFECCIOSAS (TRIPOMASTIGOTE) son eliminadas en las heces; la infección se produce al depositar las heces en una herida de la piel antes de picar en las membranas mucosas o en la conjuntiva. Posteriormente, se multiplican en el sitio de la entrada, y entran a la sangre como tripomastigotes circulantes e invaden a las células, CON PREDILECCIÓN POR EL MIOCARDIO, MUSCULO LISO Y SNC.

Page 344: ENARM  COMPENDIO

TRIPANOSOMIASIS O ENFERMEDAD DE CHAGAS

• Como se manifiesta la tripanosomiasis o ENFERMEDAD DE CHAGAS POR T. CRUZY?• R = En la piel, en el sitio de entrada se observa CHAGOMA que es PARECIDO A LA FURUNCULOSIS CON

LINFADENOPATIA REGIONAL. En conjuntiva, edema bipalpebral unilateral, CONJUNTIVITIS Y LINFADENITIS PRE AURICULAR (SIGNO DE ROMANA) CON FIEBRE Y ESPLENOMEGALIA LEVE. CRONICAMENTE se manifiesta por INSUFICIENCIA CARDIACA PREDOMINANTE DEL LADO DERECHO, con arritmias ventriculares y episodios de tromboembolia sistémica o pulmonar. MEGA ESÓFAGO Y MEGACOLON con síntomas de disfagia, constipación intensa, hipertrofia parotidea.

• Como se diagnostica la tripanosomiasis americana o ENFERMEDAD DE CHAGAS?• R = En la FASE AGUDA se observa con la TINCIÓN DE GIEMSA de sangre anti coagulada. Diagnostico

serológico se basa en hemaglutinación indirecta o ELISA.

• Como se diagnostica la TRIPANOSOMIASIS AFRICANA?• R = Para T. GAMBIENSE por medio de PRUEBAS DE AGLUTINACIÓN y para T. RHODESIENSE es mediante la

observación del parasito en la tinción de GIEMSA.

• Cual es el tratamiento de elección para la TRIPANOSOMIASIS AMERICANA O CHAGAS?

• R = NIFURTIMOX por 30-90 DÍAS o BENZIDIMAZOL de 30-90 DIAS.

• Cual es el tratamiento de la TRIPANOSOMIASIS AFRICANA?• R = Para T. GAMBIENSE ES PENTAMIDINA POR 10 DÍAS. Para T. RHODESIENSE ES LA SURAMINA POR 7

DÍAS.

Page 345: ENARM  COMPENDIO

TENIASIS• Cual es el agente causal de la teniasis?• R = T. saginata y T. solium

• Cuales son las partes estructurales que componen a la tenia?• R = En su extremo anterior tienen una estructura llamada ESCÓLEX (CABEZA), seguida de un cuello. En su

escólex presentan cuatro ventosas orales. En sus ventosas tiene un róstelo, con una corona doble de ganchos (escolices). El CUERPO (ESTRÓBILO) se conforma por varios segmentos llamados PROGLOTIDOS, que en su interior tienen ÓRGANOS SEXUALES.

• Cuanto miden los adultos de T. SAGINATA?• R = 2-7 METROS de largo

• Cuanto puede llegar a medir la T. SOLIUM?

• R = 15-18 METROS

• Cual es el ciclo biológico de la teniasis?• R = La teniasis intestinal es causada por la INGESTA DE CISTICERCOS. Al legar a intestino delgado, se unen

a la pared intestinal. De los 2 a los 4 meses de la infección, alcanzan su madurez, caracterizada por la presencia de proglotidos gravidos que liberan huevos y se excretan por las heces. La cisticercosis ocurre por la ingestión de los alimentos o agua contaminados con excremento humano que contiene huevos o proglotidos. Los huevos se abren en el intestino, liberando a las oncoesferas, que invaden la pared intestinal pasando al torrente sanguíneo y linfático para migrar hacia musculo estriado, cerebro e hígado.

Page 346: ENARM  COMPENDIO

TENIASIS• Cual es el cuadro clínico de la teniasis?• R = Normalmente es asintomática. Cuando cursa con síntomas son: PLENITUD

ABDOMINAL, FATIGA, DOLOR LEVE OCASIONAL E HIPOREXIA.

• Que déficit vitamínico cursa con teniasis?• R = Déficit de VITAMINA B12

• Cual es el cuadro clínico de CISTICERCOSIS en el SNC?• R = CRISIS CONVULSIVAS, cefaleas, vomito, HIPERTENSIÓN INTRACRANEAL.

• Como diagnosticas la teniasis?• R = COPROPARASITOSCOPICO SE OBSERVAN HUEVOS, ELISA.

• Cual es el tratamiento para teniasis?• R = PRAZICUANTEL a DOSIS ÚNICA DE 10 MG/KG. Ninguna droga mata a los huevos

de tenia solium. NICLOSAMIDA para TENIA SAGINATA

Page 347: ENARM  COMPENDIO

NEUROLOGIA

ENARM

Page 348: ENARM  COMPENDIO

MANIFESTACIONES DE LESION AL SNC• Como se muestra una lesión del TRACTO ÓPTICO?• R = Como CUADRANTOPSIA homónima contralateral.

• Que caracteriza al síndrome de Horner?• R = Miosis, ptosis y anhidrosis debido a lesión de las vías del SISTEMA SIMPÁTICO ispolaterales.1) Miosis: Se debe a denervación simpática del musculo tarsal2) Anhidrosis: Consecuencia de la denervación simpática del musculo tarsal3) Para localizar la lesión se realizan pruebas con cocaína, hidroxifentamina y fenilefrina.

• Que ocasiona las pupilas de Argyll Robertson?1) Son PUPILAS IRREGULARES Y MÁS PEQUEÑAS de lo normal en la oscuridad.2) Demuestran falta del reflejo fotomotor, tanto directo como consensual, sin embargo conservan la acomodación.3) Se asocia a NEUROSIFILIS.

• Que ocasiona las pupilas de Holmes-Adie?1) Se trata de una PUPILA GRANDE Y TÓNICA de forma irregular2) Tiene reacción pobre ante estimulo luminoso3) Suele indicar una lesión de las vías parasimpáticas posganglionares, a nivel del ganglio ciliar o los nervios ciliares

cortos.4) Se ve una REACCIÓN CONSTRICTORA EXAGERADA ante la administración de agonistas muscarinicos como la

meticolina o la PILOCARPINA.5) Se asocian a reflejos de estiramiento muscular disminuidos o ausentes.

Page 349: ENARM  COMPENDIO

MANIFESTACIONES DE LESION AL SNC

• Cuales son las manifestaciones oculares en caso de obstrucción de las RADIACIONES ÓPTICAS?• R = CUADRANTOPSIAS HOMONIMAS

• -A 75-year-old woman presents with symptoms of visual change and facial weakness. On examination, the pupils are equal and reactive to light, the fundi appear normal, and there is a right homonymous visual field defect. Which of the following is the most likely cause of the right homonymous hemianopia?

• (A) right optic nerve• (B) chiasm• (C) right optic radiations• (D) right occipital lobe• (E) left optic radiations

• - The hemianopia is due to a lesion of the left optic radiations. The posterior cerebral artery arises from the basilar artery but is sometimes a branch of the internal carotid. With posterior cerebral artery lesions affecting the occipital cortex, it is possible for the hemianopia to be an isolated finding.

Page 350: ENARM  COMPENDIO

• CORRELACIÓN TOPOGRÁFICA DE DEFECTOS DEL CAMPO VISUAL. A: abolición CV izquierdo (lesión nervio óptico izquierdo). B: escotoma

yuncional. C: hemianopsia bitemporal heterónima (lesión quiasmática). D: hemianopsia homónima derecha (lesión quiasmática). E:

sectoronopsia cuádruple homónima derecha (lesión ganglio geniculado izquierdo). F: Sectoranopsia horizontal derecha (lesión ganglio

geniculado izquierdo). G: Cuadrantanopsia homónima inferior derecha (lesión lóbulo parietal izquierdo). H: Cuadrantanopsia homónima

superior derecha (lesión lóbulo temporal izquierdo). I: Hemianopsia homónima derecha con respeto macular (lesión occipital izquierda).

Page 351: ENARM  COMPENDIO
Page 352: ENARM  COMPENDIO

MANIFESTACIONES DE LESION AL SNC

• Que es la disautonomia?• R = Proceso patológico central o periférico que se manifiesta por anomalías de la regulación de la

TA, diaforesis termorreguladora, función sexual.

• Que es el mioclono?1) Es común en epilepsia, presentando ESPASMOS. Se presenta en casos de anoxia, hiperuricemia. 2) El tratamiento es con levodopa, privación alcohólica y responde a acido valproico y

benzodiacepinas.

• Que motoneurona se ve afectada en la espasticidad?

• R = MOTONEURONA SUPERIOR

• Cual es tratamiento de espasticidad?• R = Fisioterapia, dantroleno debilita la contractura muscular al interferir con el calcio, diazepam,

inyección de fenol para disminuir la espasticidad selectiva o toxina botulínica.

Page 353: ENARM  COMPENDIO

MANIFESTACIONES DE LESION AL SNC• En que zona se encuentra la lesión en un paciente estuporoso/comatoso en ausencia unilateral

de respuesta a pesar de la aplicación de estímulos en ambos lados del cuerpo?• R = CORTICOESPINAL

• En que zona se encuentra la lesión en paciente en ausencia bilateral de respuesta a estímulos?• R = Afección de TALLO ENCEFÁLICO, lesión bilateral de vía piramidal

• La postura de decorticacion a que daño se debe?• R = Lesión de CAPSULA INTERNA y del PEDÚNCULO CEREBRAL ANTERIOR.

• La postura de decerebracion a que daño se debe?• R = DESTRUCCIÓN MESENCÉFALO y de la porción anterior del puente de vacilio

• Que causa el síndrome de encerramiento o estado desaferentado?1) Lesiones agudas que afectan principalmente la PARTE ANTERIOR DEL PUENTE DE VACILIO Y

RESPETAN EL TEGUMENTO. 2) Tienen respuesta ocular y están consientes pero en estado comatoso y se RECUPERAN DE 2-3ª.

Page 354: ENARM  COMPENDIO

Posición Decorticación Posición Descerebración

Page 355: ENARM  COMPENDIO
Page 356: ENARM  COMPENDIO

MOTONEURONAS• Que caracteriza a las lesiones de la motoneurona?• R = Debilidad sin perdida sensorial

• Cual es la clasificación de la enfermedad de motoneurona y que caracteriza a cada una de ellas?1) Parálisis bulbar progresiva: degeneración que predomina en núcleos motores de pares craneales 2) Parálisis pseudobulbar: se debe a enfermedad cortico bulbar alterando neurona motora superior 3) Atrofia muscular espinal progresiva: Hay degeneración de las astas anteriores de la medula espinal

afectando a neurona motora inferior 4) Esclerosis lateral primaria: Déficit puro de neurona motora superior en extremidades inferiores5) Esclerosis lateral aminotrofica: Déficit mixto de NMS e INFERIOR en extremidades pudiendo

alterar el estado cognitivo con DEMENCIA FRONTOTEMPORAL O PARKINSONISMO.

• Cual es el cuadro clínico de enfermedad degenerativa de neuronas motoras?1) Alteración bulbar: dificultad para la deglución, masticación, tos, respiración y disartria. 2) Parálisis bulbar progresiva: descanso del paladar disminuyendo reflejo nauseoso, tos débil y

desgaste muscular. Lingual con fasciculaciones.3) Parálisis pseudobulbar: Contractura y espasmo lingual, disminución de la motilidad de la lengua.

Hay trastorno motor en extremidades con esfínteres normales 4) ESCLEROSIS LATERAL AMINOTROFICA: ES LETAL de 3-5 años comúnmente POR INFECCIÓN

PULMONAR, presentándose con periodos de REMISIÓN Y EXACERBACIÓN DE DIFICULTAD PARA CAMINAR, TRAGAR , TORPEZA Y FINALMENTE RESPIRAR CON CARÁCTER AUTOSOMICO DOMINANTE EN GEN 21.

Page 357: ENARM  COMPENDIO

MOTONEURONAS• Cuales son los signos positivos y negativos que indican lesión a la MOTONEURONA SUPERIOR o síndrome piramidal?1) Positivos: ESPASTICIDAD, HIPERREFLEXIA en reflejos de estiramiento muscular, clonus, distonía, atetosis y exaltación de reflejos cutáneos

como signo de Babinski2) Negativos: Debilidad o paresia hemicorporal y pérdida de la destreza, coordinación y control motor.3) La combinación de ambos afectan la ejecución normal del movimiento con un deterioro en la calidad de vida del paciente.

• Cual es el cuadro clínico del síndrome de la MOTONEURONA INFERIOR?• R = ATROFIA muscular por denervación, parálisis o PARESIA FLÁCIDA de todos los músculos de la unidad motora afectada, HIPOTONÍA,

hiporreflexia o ARREFLEXIA al interrumpirse la parte eferente del arco reflejo y fasciculaciones producidas por la actividad espontanea de la unidad motora.

• Cuales son las características clínicas del síndrome de la columna posterior?1) Se caracteriza por perdida o DISMINUCIÓN DE LA SENSACIÓN DE VIBRACIÓN, SENSACIÓN DE POSICIÓN, DISCRIMINACIÓN DE DOS PUNTOS,

TACTO Y RECONOCIMIENTO DE LA FORMA2) Estimulos que no eran dolorosos SE CONVIERTEN EN DOLOROSOS y LOS ESTÍMULOS DOLOROSOS SE DESENCADENAN CON UMBRALES DE

ESTIMULACIÓN MAS BAJOS.

• Que caracteriza al síndrome de la arteria espinal anterior?• R = También conocido como síndrome medular ventral que ocasiona PARAPLEJIA O CUADRIPLEJIA, perdida bilateral de la sensación de dolor y

temperatura por debajo del nivel de la lesión.

• Que estudios realizas para diagnosticar la enfermedad de la motoneurona?• R = ELECTROMIOGRAFÍA: muestra DENERVACIÓN, EN ESCLEROSIS LATERAL AMINOTROFICA debe presentarse en 3 extremidades mínimo.

LINFOMA DE HODKING puede cursar con NEUROPATÍA MOTORA.

• Cual es el tratamiento de lesiones de neuronas motoras?

• R = RICUZOL: disminuye la LIBERACIÓN PRESINAPTICA DE GLUTAMATO haciendo lenta la ELA progresiva. PLASMAFERESIS logra

mejoría. ANTICOLINERGICOS: atropina y amitriptilina

Page 358: ENARM  COMPENDIO

MOTONEURONAS• - A 53-year-old man complains of clumsiness with both hands, like having difficulty doing up buttons or using his keys. Physical

examination reveals fasciculations of his thigh and forearm muscles; diffuse muscle weakness, loss of muscle bulk, and increased tone in the upper and lower limbs. There is generalized hyperreflexia, and positive Babinski signs bilaterally. Which of the following is the most likely natural progression of this condition?

• (A) a long history of remissions and exacerbations (sclerosis lateral aminotrofica)• (B) sensory loss in the distribution of peripheral nerves• (C) focal seizures• (D) a progressively downhill course• (E) cogwheel rigidity

• - This man has amyotrophic lateral sclerosis (ALS). The disease causes neuronal loss in the anterior horns of the spinal cord and motor nuclei of the lower brain stem. The disease is one of constant progression, rather than remissions and exacerbations, and death usually occurs within 5 years. There is no sensory loss and no seizure diathesis, because only the motor system is involved. There can be signs ofhyperreflexia and spasticity, depending on the balance of upper and lower motor neuron damage, but not cogwheel rigidity.

• -A 45-year-old man presents with weakness and fasciculations in his arms and legs . His cranial nerves are normal, but there is weakness of his left handgrip and right leg quadriceps with loss of muscle bulk. There are obvious fasciculations over the left forearm and right thigh. Tone is increased in the arms and legs and the reflexes are brisk. Which of the following is the most likely diagnosis?

• (A) amyotrophic lateral sclerosis (ALS)• (B) myotonic muscular dystrophy• (C) amyotonia congenita• (D) tabes dorsalis• (E) migraine

• - The most common initial symptom of ALS is weakness and wasting of the extremities. The fasciculations can be a very prominent part of the disease. This is rare in other neurologic disorders.

Page 359: ENARM  COMPENDIO
Page 360: ENARM  COMPENDIO

ESLEROSIS LATERAL AMINOTROFICA• Que es la esclerosis lateral aminotrófica?1) Se refiere a una condición neurología de INICIO EN LA VIDA ADULTA caracterizada por

una DEGENERACIÓN PROGRESIVA DE LAS NEURONAS MOTORAS SUPERIORES E INFERIORES.

2) La degeneración de las neuronas de la corteza motora se manifiesta como hiperreflexia, signos positivos de Hoffman, Babinski y clonus.

3) El termino esclerosis lateral se refiere a la DUREZA CUANDO SE PALPAN LAS COLUMNAS LATERALES DE LA MEDULA ESPINAL en las autopsias de pacientes con esta enfermedad.

4) El termino aminotrófica se refiere a la ATROFIA MUSCULAR, DEBILIDAD Y FASCICULACION debidas a la degeneración de las neuronas motoras inferiores.

• Que hallazgos muestra la electromiografía al respecto de la esclerosis lateral aminotrófica?

• R = Demuestra DENERVACIÓN de POR LO MENOS 3 EXTREMIDADES.

• Cual es el manejo de la esclerosis lateral aminotrófica?• R = RILUZOLE QUE ES UN ANTAGONISTA DEL GLUTAMATO Y LA VITAMINA E. Deben ser

tratados como si tuvieran lesión de la columna cervical hasta que se demuestre lo contrario.

Page 361: ENARM  COMPENDIO

CHARCOT-MARIE-TOOTH

• Cuales son las características de la mononeuropatia hereditaria de Charcot-Marie-Tooth?

1) Carácter AD en CROMOSOMA 17. 2) Clínicamente con DEFORMIDADES DEL PIE,

TRASTORNOS DE LA MARCHA. 3) El electrodiagnóstico muestra DISMINUCIÓN DE LA

CONDUCCIÓN MOTORA Y SENSITIVA (neuropatía motora y sensitiva hereditaria tipo I) NMSH tipo I.

4) NMSH tipo II disminuye solo un poco. 5) Histológicamente hay PERDIDA AXONAL.

Page 362: ENARM  COMPENDIO

TRAUMATISMO• Que te indica el signo de mapache?• R = Equimosis en parpados debido a fractura de base del craneo por TCE

• Que maniobra resulta útil en el TCE que causa salida de LCR por nariz u oídos (contiene glucosa)?• R = CONSERVADOR, con ELEVACIÓN DE LA CABEZA, RESTRICCIÓN DE LÍQUIDOS, administración de acetazolamida

EN CASO DE PERSISTENCIA SE REALIZA DRENAJE LUMBAR SUBARACNOIDEO

• Cual es el cuadro clínico de traumatismo en medula espinal?1) La transección medular completa resulta en PARÁLISIS FLÁCIDA INMEDIATA Y PERDIDA DE LA SENSIBILIDAD

DEBAJO DE LA LESIÓN. 2) Se pierde retención urinaria y fecal. 3) Al final hay paraplejia en las piernas en flexión o extensión. 4) En las LESIONES MENORES los pacientes que dan con DEBILIDAD LEVE DE LAS EXTREMIDADES, TRASTORNOS

SENSITIVOS DISTALES O AMBOS.

• Que caracteriza al síndrome de BROWN-SEQUARD?• R = Es una LESIÓN UNILATERAL de la MEDULA ESPINAL que da lugar a un TRASTORNO MOTOR IPSOLATERAL con

deterioro concomitante de la PROPIOSEPCION y PERDIDA CONTRALATERAL de las apreciaciones del DOLOR Y TEMPERATURA

• Cual es el manejo del traumatismo medular?• R = INMOBILIDAD. GRANDES DOSIS DE ESTEROIDES metilprednisolona 300 mg en bolo seguidos por 5.4 mg/kg/hr

x 24 hrs.

Page 363: ENARM  COMPENDIO

TRAUMATISMO• Cual es la principal etiología de un HEMATOMA SUBDURAL y sus principales

características clínicas?• R = A subdural hematoma is almost always of VENOUS ORIGIN and secondary to a minor

or severe INJURY TO THE HEAD, but may occur in blood dyscrasias or cachexia in the absence of trauma. ACUTE subdural hematomas commonly present with a FLUCTUATING LEVEL OF CONSCIOUSNESS and significant cerebral damage. CHRONIC subdurals may also present with SEIZURES OR PAPILLEDEMA

• -A 94-year-old man presents with progressive headaches, light-headedness, drowsiness, and unsteady gait over 6 weeks. On examination, his blood pressure is 160/90 mm Hg, pulse 70/min, lungs clear, and he has no focal weakness. His gait is unsteady but sensation in the feet is normal. A CT scan reveals a hyperintense clot over the left cerebral cortex. Which of the following is the most likely cause for this clot?

• (A) is venous in origin• (B) is arterial in origin• (C) is from injury to the middle meningeal artery• (D) is from a subarachnoid hemorrhage• (E) is from injury to the middle cerebral artery

Page 364: ENARM  COMPENDIO
Page 365: ENARM  COMPENDIO
Page 366: ENARM  COMPENDIO
Page 367: ENARM  COMPENDIO

ESCALA DE GLASGOW MODIFICADA (NIÑOS)

PRUEBA RESPUESTA PUNTUACI ÓN

APERTURA DE OJ OS

Espontanea A órdenes

Al estimulo doloroso Nula

4 3 2 1

LLANTO COMO RESPUESTA

VERBAL

Palabras apropiadas y sonrisas, fi ja la mirada y sigue los objetos.

Tiene llanto, pero consolable. Persistente e irritable Agitado. Sin respuesta.

5 4 3 2 1

RESPUESTA MOTORA

Obedece ordenes Localiza el dolor

Retirada ante el dolor Flexión inapropiada

Extensión Nula

6 5 4 3 2 1

Page 368: ENARM  COMPENDIO

Si EGC: 15-----------------------Consciente      Si EGC: 13-14-------------------Estupor ligero      Si EGC: 11-12-------------------Estupor moderado      Si EGC:  9-10--------------------Estupor profundo      Si EGC:   7-8---------------------Coma superficial      Si EGC:   5-6---------------------Coma moderado      Si EGC:   3-4---------------------Coma profundo 

Page 369: ENARM  COMPENDIO

Según eltamaño

Mioticas Diámetro < 2mmMedias Diámetro 2-5mmMidriaticas Diámetro > 5mm

Según relaciónentre ellas

I socoricas I gualesAnisocoricas DesigualesDiscoricas Forma irregular

Reactivas Contracción al f ocoluminoso

Segúnrespuesta

a la luzArreactivas

I nmóviles al f ocoluminoso

REACTIVIDAD PUPILAR.

Page 370: ENARM  COMPENDIO

SÍNDROME DEL TÚNEL CARPIANO• A que se asocia el síndrome del túnel carpiano?• R = HIPOTIROIDISMO, EMBARAZO, diabetes, AR o ACROMEGALIA.

• Cuales son las manifestaciones clínicas del síndrome del túnel carpiano?1) HORMIGUEO, dolor, ENTUMECIMIENTO EN LA MANO, sobre todo al

comienzo de la mañana y en ocasiones con DEBILIDAD DE LA MUSCULATURA TENAR con atrofia del abductor corto del pulgar.

2) Puede observarse el SIGNO DEL TINEL que se provoca con un golpesito en el túnel carpiano REPRODUCIENDO DOLOR Y HORMIGUEO.

• Como dx el sx del túnel carpiano?• R = CLÍNICA + ELECTROMIOGRAFÍA.

• Cual es el manejo del sx del túnel carpiano?• R = Descompresión quirúrgica.

Page 371: ENARM  COMPENDIO

COMPRESIÓN DEL NERVIO CUBITAL

• Como se manifiesta clínicamente la compresión del nervio cubital?1) ADORMECIMIENTO EN EL QUINTO DEDO y en el lado MEDIAL de la

mano. 2) El dolor y las parestesias pueden iniciar en el codo e irradiarse a la mano.

3) LAS PARESTESIAS SE EXACERBAN CON LA FLEXIÓN DEL CODO.

• Como diagnosticas la compresión del nervio cubital?• R = Electrofisiología.

• Cual es el manejo de la compresión del nervio cubital?• R = Los ESTEROIDES Y LOS AINES. Se recomienda cirugía.

Page 372: ENARM  COMPENDIO

NERVIO CUBITAL• Cual es la sintomatología en caso de daño al nervio cubital?• R = Injury to the ulnar nerve results in impaired adduction and abduction of the

fingers. The nerve is commonly injured in ELBOW DISLOCATIONS and fractures. The fibers arise from the eighth cervical and the first thoracic segments. The ulnar is a mixed nerve with sensory supply to the medial hand.

• - A 40-year-old man is injured in a car accident and fractures his left elbow. He now complains of numbness of his fourth and fifth fingers, and weakness in his hand grip. Neurologic findings confirm weakness of handgrip with weakness of finger abduction and adduction, and decreased sensation over the fifth finger and lateral aspect of fourth finger. Which of the following is the most likely diagnosis?

• (A) ulnar nerve injury• (B) radial nerve injury• (C) median nerve injury• (D) carpal tunnel syndrome• (E) axillary nerve injury

Page 373: ENARM  COMPENDIO

COMPRESIÓN DEL NERVIO RADIAL

• Como se manifiesta la compresión del nervio radial?

1) “Parálisis de la noche del sábado” por dejar el brazo colgando después de una borrachera.

2) MUÑECA FLÁCIDA y debilidad a la extensión de los dedos.

Page 374: ENARM  COMPENDIO

CIATICO Y TARSO

• Cual es la etiología mas común de la parálisis del ciático?

• R = INYECCIONES INTRAMUSCULARES profundas

• Que sintomatología da el síndrome del túnel del tarso?• R = NERVIO TIBIAL, rama del ciático, cuando se

comprime da origen a DOLOR, PARESTESIAS Y ENTUMECIMIENTO SOBRE BASE DEL PIE PREDOMINANDO DURANTE LA NOCHE sin afectar talón.

Page 375: ENARM  COMPENDIO

MERALGIA PARESTESICA

• Que causa y como se manifiesta clínicamente la meralgia parestesica?

1) Se debe al atrapamiento del NERVIO CUTÁNEO LATERAL DEL MUSLO POR DEBAJO DEL LIGAMENTO INGUINAL.

2) Se produce DOLOR URENTE, PARESTESIAS Y ENTUMECIMIENTO DE LA CARA ANTEROLATERAL DEL MUSLO.

3) Se presenta en pacientes con EXCESO DE PESO por lo que el TRATAMIENTO ES BAJAR DE PESO.

4) Nervio FEMOROCUTANEO EXTERNO, se origina en RAÍCES L2 Y L3 EL CUAL SE PRESIONA EN CASO DE EMBARAZO, DIABETES Y OBSESOS.

Page 376: ENARM  COMPENDIO

LESIONES DE LOS PARES CRANEALES• Que datos clínicos se encuentran en el daño al III (OCULOMOTOR) PAR CRANEAL?1) Frecuentemente causa ANISOCORIA, en la que la pupila mas dilatada denota al LADO AFECTADO.2) El fenómeno de MARCUS GUNN consiste en que con el ESTIMULO LUMINOSO NO OBTENEMOS

RESPUESTA EN EL LADO AFECTADO después de presentar contracción por efecto de reflejo consensual al estimular contralateralmente

3) Suele deberse a CAUSAS HIPOXICAS4) En pacientes con compresión cerebral se mostrara midriasis ipsolateral al sitio afectado como resultado

de compresión del III par craneal en cualquier parte de su recorrido debido a que las fibras que inervan el esfínter de la pupila discurren en porción periférica del nervio.

5) PTOSIS PALPEBRAL CONTRALATERAL a la lesión.6) La lesión en este par craneal afecta los MÚSCULOS ELEVADOR DEL PARPADO, recto medial, recto

superior, recto inferior y oblicuo inferior (TODOS MENOS EL RECTO EXTERNO Y EL OBLICUO SUPERIOR), lo que provoca que los movimientos de ADUCCIÓN, ELEVACIÓN Y DEPRESIÓN DEL OJO IPSOLATERAL ESTÉN AFECTADOS.

7) Lo anterior provoca que cuando el paciente este de frente el ojo tiende a DESVIARSE HACIA ABAJO Y AFUERA.

• Que datos clínicos se encuentran en el daño al IV (PATÉTICO O TROCLEAR) PAR CRANEAL?1) Presentan debilidad del MUSCULO OBLICUO SUPERIOR2) DIPLOPIA CON ADUCCION Y HACIA ABAJO

• Que datos clínicos se encuentran en el daño al VI (OCULOMOTOR EXTERNO) PAR CRANEAL?1) Afecta al MUSCULO RECTO LATERAL2) El ojo afectado se dirige A LA LINEA MEDIA CON INCAPACIDAD DE ABDUCIR EL OJO.

Page 377: ENARM  COMPENDIO

LESIONES DE LOS PARES CRANEALES• Que datos clínicos se encuentran en el daño al V (TRIGÉMINO) PAR CRANEAL?

1) La parálisis de la primera división (OFTÁLMICA) involucra PERDIDA DE LA SENSIBILIDAD EN LA FRENTE junto con parálisis del III par y IV.

2) La parálisis de la segunda división (MAXILAR) resulta en la PERDIDA DE SENSIBILIDAD DE LA MEJILLA y se debe a lesiones del seno cavernoso.

3) La parálisis de la tercera división (MANDIBULAR) resulta en DÉFICIT EN LA MASTICACIÓN

• Que cuadro clínico resulta del síndrome ocasionado por la afección del ganglio de Gasser?• R = Síndrome caracterizado por DOLOR FACIAL, CEFALEA, PERDIDA DE LA SENSIBILIDAD SENSORIAL, SORDERA Y PARÁLISIS

DEL VI Y VII PARES. EL DOLOR AUMENTA AL MOVER LA MANDÍBULA.

• Cual es el cuadro clínico resultante de la neuralgia del glosofaríngeo?• R = Es un trastorno unilateral que afecta principalmente a V3 presentándose en forma de ESPASMOS DOLOROSOS que

pueden durar de segundos a minutos y pueden estar asociados a estímulos cutáneos y auditivos.

• Cual es la etiología de la parálisis de Bell?• R = Se asocia a HERPES VIRUS

• Que características clínicas tiene la parálisis de Bell?

1) PARÁLISIS DEL VII PAR CON INICIO SÚBITO DE DEBILIDAD FACIAL y pérdida del reflejo acústico. 2) Hay PRÓDROMO VIRAL y adormecimiento o dolor en el oído, lengua o cara. 3) La CUERDA DEL TÍMPANO SE VE ERITEMATOSA y HAY REDUCCIÓN EN LA LAGRIMACIÓN IPSOLATERAL o salivación.4) El paciente no puede cerrar los ojos por lo que se debe evitar el daño a la cornea5) Se diagnostica con ELECTROMIOGRAFÍA denotándose una DISMINUCIÓN EN LA VELOCIDAD DE CONDUCCIÓN

Page 378: ENARM  COMPENDIO

LESIONES DE LOS PARES CRANEALES• Cual es el manejo de la parálisis de Bell?• R = VALACICLOVIR o famciclovir y PREDNISONA

• Cual es la característica clínica de la afectación del VIII (VESTIBULOCOCLEAR) par craneal?1) Porción coclear: El principal síntoma asociado a la lesión o atrofia de esta porción es la HIPOACUSIA

SENSORINEURAL. Se realiza el diagnostico por AUDIOMETRÍA.2) Porción vestibular: VÉRTIGO POSTURAL PAROXÍSTICO BENIGNO.

• Que características clínicas se observan por daño al XI (ESPINAL ACCESORIO) par craneal?• R = Dolor significativo acompañado de una DISMINUCIÓN DE LA MOVILIDAD DEL HOMBRO, siendo la lesión

iatrogena la mas común (p.e la BIOPSIA DE LOS GANGLIOS LINFÁTICOS).

• Cuales son los datos clínicos que involucra una lesión en los pares craneales IX Y X (GLOSOFARÍNGEO Y VAGO)?

1) DISFAGIA, parálisis de la cuerda vocal ipsolateral, así como DEBILIDAD DEL PALADAR Y LA FARINGE. 2) La ELEVACIÓN DEL PALADAR ES ASIMÉTRICA y la ÚVULA SE DESVÍA AL LADO CONTRARIO DE LA LESIÓN.3) Se pierde la capacidad sensorial de UN TERCIO POSTERIOR DE LA LENGUA, con disminución en la

sensibilidad en la faringe posterior y laringe.4) La LESIÓN más común del VAGO es la que involucra al NERVIO LARÍNGEO RECURRENTE, lo que provoca

PARÁLISIS DE LA CUERDA VOCAL IPSOLATERAL Y SUBSECUENTE DISFONÍA.5) La neuralgia del glosofaríngeo tiene CARACTERÍSTICAS SIMILARES A LA NEURALGIA DEL TRIGÉMINO, pero

es MAS EVIDENTE EN LA LENGUA, LAS AMÍGDALAS, OÍDO Y ANGULO DE LA MANDIBULA

Page 379: ENARM  COMPENDIO

LESIONES DE LOS PARES CRANEALES

• Que características clínicas tiene el daño al XII (HIPOGLOSO) par craneal?• R = DEBILIDAD en la lengua, FASCICULACIONES y DESVIACIÓN DE LA LENGUA hacia el lado de la lesión.

• Que medicamento es de elección en neuralgia del trigémino y del glosofaríngeo?• R = CARBAMACEPINA, pero también puede incluir baclofen y fenitoina.

• - A 52-year-old man complains of episodes of severe unilateral, stabbing facial pain that is intermittent for several hours, and then disappears for several days. The pain is describes as “electric shock-like” and only lasts a few seconds. Physical examination of the face and mouth is entirely normal. Which of the following treatments is most effective for this condition?

• (A) morphine• (B) indomethacin• (C) cimetidine• (D) carbamazepine• (E) lidocaine (Xylocaine) gel

• - This patient has trigeminal neuralgia. Carbamazepine (an anticonvulsant drug) is given in doses varying from 600 to 1200 mg/day. Phenytoin has also been used. The two drugs can also be used in combination. Operative procedures include alcohol injection of the nerve or ganglion, partial section of the nerve in the middle or posterior fossa, decompression of the root, and medullary tractotomy.Radiofrequency surgery can destroy pain fibers but spare motor fibers.

Page 380: ENARM  COMPENDIO

VERTIGO• Cual es la diferencia entre vértigo de origen central y de origen periférico?• R = In CENTRAL VERTIGO, the vertigo can be MILD AND CHRONIC. In PERIPHERAL disease, THE SYMPTOMS ARE

GENERALLY MORE SEVERE, BUT FINITE (although often recurrent).

• - A 63-year-old woman develops symptoms of nausea, vomiting, and dizziness, which she describes as a to-and-fro movement of the room like as if she is on a boat. Which of the following findings suggests the vertigo is central in origin?

• (A) deafness• (B) symptoms are more protracted but less severe• (C) unidirectional nystagmus• (D) visual fixation inhibits vertigo and nystagmus• (E) spinning sensation is toward the fast phase of nystagmus

• Cuales son las características del vértigo de ORIGEN PERIFÉRICO?• R = TINNITUS AND DEAFNESS may be found in PERIPHERAL VERTIGO, but NOT CENTRAL. The nystagmus is usually

unidirectional and is never vertical. VISUAL FIXATION INHIBITS VERTIGO AND NYSTAGMUS during testing in peripheral vertigo

• - A 47-year-old man presents to the emergency room with symptoms of dizziness and difficulty walking. He describes his dizziness as a spinning sensation of the room with associated nausea and vomiting. Which of the following findings suggests the vertigo is peripheral in origin?

• (A) optic neuritis• (B) tinnitus• (C) bidirectional nystagmus• (D) vertical nystagmus• (E) visual fixation does not affect vertigo or nystagmus

Page 381: ENARM  COMPENDIO

VERTIGO• Cual es la sintomatología del vértigo benigno posicional paroxístico y que maniobra utilizas para

demostrarlo?• R = Benign paroxysmal positional vertigo (BPPV), which is characterized by sudden-onset brief

episodes of vertigo LASTING LESS THAN A MINUTE. The symptoms are USUALLY BROUGHT ON BY HEAD MOVEMENT. The cause is commonly attributed to CALCIUM DEBRIS IN THE SEMICIRCULAR CANALS, KNOWN AS CANALITHIASIS. The debris is loose otoconia (calcium carbonate) within the utricular sac. Although BPPV can occur after head trauma, there is usually no obvious precipitating factor. IT GENERALLY ABATES SPONTANEOUSLY AND CAN BE TREATED WITH VESTIBULAR REHABILITATION. A DIX-HALLPIKE MANEUVER REPRODUCES HIS SYMPTOMS

• - A 48-year-old man complains of recurrent episodes of sudden-onset dizziness. He notices an abrupt onset of a spinning sensation when sitting up or lying down in bed. The symptoms last for 30 seconds and then resolve completely. He has no hearing change or other neurologic symptoms, and his physical examination is completely normal. A Dix-Hallpike maneuver reproduces his symptoms. Which of the following is the most likely mechanism for his vertigo symptoms?

• (A) basilar migraine• (B) brain stem ischemic events• (C) benign cerebellar tumors• (D) calcium debris (calcium carbonate crystals) in the semicircular canals• (E) Meniere’s disease

Page 382: ENARM  COMPENDIO

EVC• Cual es la duración de un ataque isquémico transitorio?• R = Dura < 24 HRS y el DÉFICIT CLÍNICO SE RESUELVE EN MENOS DE 1-2 HRS. Se debe

utilizar ASA COMO PROFILAXIS.

• Cuanto debe medir un infarto lacunar y como se debe ver en la TAC?• R = Mide <5 MM de diámetro, en la TAC se observan como LESIONES PEQUEÑAS EN

SACABOCADO HIPODENSAS, con RESOLUCIÓN parcial o completa de 4-6 SEMANAS

• Que sintomatología da el EVC en arteria cerebral anterior?• R = DEBILIDAD y perdida sensitiva en la pierna contralateral y debilidad leve en brazo.

• Que cambios produce el infarto cerebral anterior?• R = CAMBIOS CONDUCTUALES y alteración de la MEMORIA.

• Que sintomatología da la obstrucción de la arteria cerebral media?• R = AFASIA, HEMIPLEGIA CONTRALATERAL, perdida sensitiva, HEMIANOPSIA HOMÓNIMA,

desviación ocular al lado de la lesión.

Page 383: ENARM  COMPENDIO

EVC• Que sintomatología causa la obstrucción de la ARTERIA CEREBRAL ½ en su PORCIÓN POSTERIOR?• R = Causa AFASIA DE WERNIKE y HEMIANOPSIA HOMONIMA

• - A 74-year-old woman develops acute neurologic symptoms and presents to the emergency room. An urgent magnetic resonance imaging (MRI) scan demonstrates acute occlusion in the right posterior cerebral artery. Which of the following clinical symptoms is she most likely to have?

• (A) homonymous hemianopia• (B) total blindness• (C) expressive aphasia• (D) ataxia and dysarthria• (E) a right-sided hemiplegia

• -Occlusion of the right posterior cerebral artery is most likely to cause homonymous hemianopia. This artery conveys blood to the inferior and medial portion of the posterior temporal and occipital lobes and to the optic thalamus.

• - A 63-year-old man developed a transient episode of vertigo, slurred speech, diplopia, and paresthesias. He is symptom-free now, and clinical examination is entirely normal. His pastmedical history is significant for hypertension and dyslipidemia. Which of the following is the most likely cause for symptoms?

• (A) posterior circulation transient ischemic attack (TIA)• (B) anterior communicating artery aneurysm• (C) hypertensive encephalopathy• (D) pseudobulbar palsy• (E) occlusion of the middle cerebral artery

• -Posterior circulation TIA is suggested by the transient episodes. The basilar artery is formed by the two vertebral arteries and supplies the pons, the midbrain, and the cerebellum. With vertebrobasilar TIAs, tinnitus, vertigo, diplopia, ataxia, hemiparesis, and bilateral visual impairment are common findings.

Page 384: ENARM  COMPENDIO

EVC• Cual es el cuadro clínico de la obstrucción de la arteria basilar?• R = Alteración del estado de alerta, parálisis de los nervios craneales y caídas.

• Como manejas la hipertensión maligna?• R = Esta entidad se da con una TAD >130, esta se debe REDUCIR A 1/3 pero NUNCA a un VALOR MENOR DE 95 MM HG.

NO se debe usar NEFEDIPINO sublingual. En el caso de HIPERTENSIÓN INTRACRANEAL NO SE DEBE USAR ESTEROIDES.

• Como se procede ante un EVC < 3 hrs?• R = TROMBOLISIS IV con activador del plasminogeno no tisular. En los que esta CONTRAINDICADA ESTA TERAPIA son

aquellos con:1) TAS > 185 mm Hg o TAD > 1102) Sintomatología leve o que mejore de forma rápida3) CONVULSIONES al inicio de TCE en los 3 MESES PREVIOS4) CIRUGÍA MAYOR 2 semanas previas5) Glucemia < 50 o > 4006) PLAQUETAS < 100,0007) MUJERES EMBARAZADAS o en lactancia

8) Ingesta actual de anticoagulantes orales con TP > 15

• Cuales son los datos clínicos de una hemorragia intracraneal?• R = Además de depender de la localización del hematoma suele observarse CEFALEA, CRISIS CONVULSIVAS, NAUSEA Y

VOMITO.

Page 385: ENARM  COMPENDIO

EVC• Cual es el cuadro clínico de la HEMORRAGIA SUBARACNOIDEA?

1) Se debe principalmente a la ROTURA DE UN ANEURISMA SECUNDARIA A UN TRAUMATISMO en pacientes con

ANTECEDENTE DE HAS2) CEFALEA SUBITA, intensa y pulsátil espontanea o ASOCIADA A ALGÚN ESFUERZO FISICO.

3) El px lo refiere como EL DOLOR MAS INTENSO QUE HA TENIDO EN SU VIDA4) SE PUEDE ACOMPAÑAR DE NAUSEA, VOMITO, FONOFOBIA Y FOTOFOBIA.5) Si la evolución es rápida el px puede evolucionar RAPIDAMENTE A ESTADO DE COMA.6) Este tipo de hemorragia puede presentarse como MUERTE SUBITA.7) Para su diagnostico se utiliza la TAC, si hay DUDA DX SE REALIZA PUNCIÓN LUMBAR PARA BILIRRUBINA Y CRENOCITOS QUE CONFIRMAN EL

DIAGNOSTICO8) LA HIPERGLUCEMIA Y EL AUMENTO DE LA TEMPERATURA SE ASOCIAN A MAL PRONÓSTICO.

• Cual es el manejo de la HEMORRAGIA SUBARACNOIDEA?1) REPOSO ABSOLUTO2) Monitorización3) Vendaje de miembros inferiores4) No estimulación5) ANALGÉSICOS IV

6) NIMODIPINA ORAL para vasoespasmo cerebral sintomático

7) HIPERVOLEMIA PROFILÁCTICA para prevenir vasoespasmo cerebral

• Cuales son las características de la HEMORRAGIA SUBDURAL?1) Son hematomas localizados entre la DURAMADRE Y LA ARACNOIDES2) TCE CON GLASGOW < 83) AGUDO < 24 HR, SUBAGUDO >24 HRS Y < 2 SEMANAS Y CRÓNICO > 2 SEMANAS4) Los síntomas son FOCALIZACIÓN E HIPERTENSIÓN INTRACRANEANA; en las FORMAS SUBAGUDA Y CRÓNICA PREDOMINAN LA CEFALEA Y

DEBILIDAD.5) Aquellos hematomas que DESPLACEN EL SEPTUM PELUCIDUM > 5 MM de la línea media DEBEN SER EVACUADOS.

Page 386: ENARM  COMPENDIO

EVC• Cuales son las características de la HEMORRAGIA EPIDURAL?1) DISMINUCIÓN DEL ESTADO DE ALERTA seguido de 2) INTERVALO DE LUCIDEZ y posteriormente 3) DETERIORO NEUROLÓGICO PROGRESIVO

• Cuales son las características de la trombosis venosa cerebral?1) 90% presenta como síntoma principal CEFALEA de inicio gradual2) Debe sospecharse ante un paciente con TAC Y PUNCIÓN LUMBAR NORMAL3) 40% convulsiones4) En la TROMBOSIS DEL SENO SAGITAL puede haber DELIRIUM Y AMNESIA.5) A la exploración física con papiledema y en trombosis venosa focalización6) El ESTÁNDAR DE ORO PARA EL DIAGNOSTICO ES UNA ANGIOGRAFÍA EN FASE VENOSA ASÍ

COMO RM CON VENOGRAFÍA7) El tratamiento de elección es la ANTI COAGULACIÓN formal por lo menos SEIS MESES

debido al riesgo de recurrencia así como de tromboembolia pulmonar.

Page 387: ENARM  COMPENDIO
Page 388: ENARM  COMPENDIO
Page 389: ENARM  COMPENDIO
Page 390: ENARM  COMPENDIO
Page 391: ENARM  COMPENDIO
Page 392: ENARM  COMPENDIO

TUMORES SNC• Cuales son las manifestaciones clínicas de una TUMORACIÓN EN EL LÓBULO FRONTAL?

• R = Declinación progresiva del intelecto y CAMBIOS EN LA PERSONALIDAD.

• Cuales son las manifestaciones clínicas que te da una TUMORACIÓN EN EL LÓBULO TEMPORAL?1) Si afectan el TERCIO ANTERIOR se presentara EPILEPSIA Y TRASTORNOS DE LA CONDUCTA como

agresividad e impulsividad

2) SI afecta los 2/3 POSTERIORES se pueden manifestar con HEMIANOPSIA LATERAL HOMÓNIMA

3) Convulsiones con alucinaciones olfativas o gustativas. Alteraciones en el campo visual.

• Cuales son las manifestaciones clínicas que da una TUMORACIÓN EN EL LÓBULO PARIETAL?

• R = TRASTORNOS CONTRA LATERALES DE LA SENSIBILIDAD. ESTEREOGNOSIA (no reconocen

objetos colocados en la mano), SINDROME DE GERTSMAN “combinación de alexia, agrafia, acalculia y agnosia digital”.

• Que manifestaciones clínicas da una TUMORACIÓN EN EL LÓBULO OCCIPITAL?

• R = HEMIANOPSIA HOMÓNIMA CRUZADA o defecto del campo visual,

PROSOPAGNOSIA ( no reconoce familiares)

• Que manifestaciones clínicas da la lesión en el TALLO CEREBRAL Y CEREBELOSAS?

• R = PARÁLISIS DE LOS PARES CRANEALES, ATAXIA, incoordinación, nistagmo.

Page 393: ENARM  COMPENDIO

TUMORES DEL SNC• Cual es el tipo de tumor mas frecuente en la medula espinal?• R = Ependimoma

• Que es la enfermedad de Von Recklinghausen?• R = Es una alteración en el CROMOSOMA 17, llamado NEUROFIBROMATOSIS TIPO I que consiste en maculas

MÚLTIPLES HIPERPIGMENTADAS Y NEUROFIBROMAS

• - A 10-year-old boy has multiple tan-colored patches on his skin, and freckle-like skin changes in his armpit area. The rest of clinical examination is normal. Which of the following conditions is also found in patients withthis disorder as they get older?

• (A) bilateral eighth nerve tumors• (B) irregular small pupils• (C) multiple cutaneous and subcutaneous tumors• (D) cataracts• (E) hip involvement

• -The two common forms of neurofibromatosis (NF-1 and NF-2) are genetically distinct. NF-1 is the type with multiple café au lait spots and is associated with axillary or inguinal freckling, iris hamartomas (Lisch nodules), peripheral neurofibromas, and bony abnormalities (including kyphoscoliosis). NF-2 is associated with CNS tumors, particularly bilateral eighth nerve tumors. Skin lesions are spare or absent, and early lens opacities can occur.

Page 394: ENARM  COMPENDIO

TUMORES DEL SNC• Que caracteriza a la neurofibromatosis tipo II?• R = Es una alteración en el CROMOSOMA 22. Provoca TUMORES DEL 8vo PAR CRANEAL

• Que es el síndrome de STURGE-WEBER?• R = ANGIOMA CAPILAR CUTÁNEO congénito que puede ocasionar ANGIOMA DE COROIDES

• Cuales son las características de los astrocitomas (gliomas)?1) Tumores intracraneales MAS FRECUENTE EN NIÑOS y jóvenes entre 20 y 40ª.2) EL ASTROCITOMA PILOCITICO es la FORMA MAS BENIGNA de los astrocitomas, PREDOMINA EN NIÑOS y adultos jóvenes,

afectando PRINCIPALMENTE AL CEREBELO.

• Cuales son las características de los OLIGODENDROGLIOMAS?1) Estos tumores suelen tener zonas de CLACIFICACIONES Y HEMORRAGIAS2) Tumor de crecimiento lento

• Cuales son las características de los ependimomas?1) Junto al sistema ventricular REVESTIDO DEL EPÉNDIMO2) En los niños aparecen principalmente en la primera década de la vida DENTRO DE LOS VENTRÍCULOS.

• Cuales son las características de los germinomas?• R = Estos tumores tienen muchas características comunes a las neoplasias de células germinales que se localizan en las

gónadas, incluidos los marcadores tumorales

• Cual es el cuadro clínico del meduloblastoma?1) CRISIS CONVULSIVAS PARCIALES O GENERALIZADAS2) Síndrome de hipertensión intracraneana caracterizado por la triada de: CEFALEA, VOMITO Y EDEMA DE PAPILA BILATERAL.3) Alteraciones cognitivas que se pueden presentar en tumores del lóbulo frontal.

Page 395: ENARM  COMPENDIO
Page 396: ENARM  COMPENDIO

PARKINSON• Que tratamiento resulta efectivo en el temblor esencial benigno?• R = PROPANOLOL y si falla primidona o CLONACEPAM

• Que medicamentos causan PARKINSON-LIKE?• R = Neurolépticos, METOCLOPRAMIDA o reseprina.

• Cual es el neurotransmisor involucrado en Parkinson y cual es la fisiopatologia?1) DOPAMINA, habiendo un desequilibro con acetilcolina2) HISTOLOGICAMENTE la enfermedad de Parkinson se caracteriza por DEGENERACIÓN NEURONAL DE LA SUSTANCIA

NIGRA, PARS COMPACTA (PRODUCTORA DE DOPAMINA) y las fibras que la comunica con el estriado.3) En el microscopio se encuentran los CUERPOS DE LEWY en la sustancia nigra pars compacta (CUERPOS DE INCLUSION

INTRANEURONAL, EOSINOFILOS CONSTITUIDOS POR ALTAS CONCENTRACIONES DE LA PROTEINA ALFA-SINUCLEINA).

• Cual es el cuadro clínico del Parkinson?

1) Motoras: TEMBLOR EN REPOSO, RIGIDEZ y bradiscinecia. CON LOS MOVIMIENTOS VOLUNTARIOS SE ATENUA EL TEMBLOR Y SE EXACERBA CON LAS EMOCIONES. La bradicinecia se manifiesta como lentitud de movimientos voluntarios. El paciente presenta PERDIDA DE LA EXPRESIÓN FACIAL, menores movimientos de los labios y lengua al hablar (hipofonia), PERDIDA DE LOS MOVIMIENTOS FINOS DE ESCRITURA (micrografía) y manipulación de objetos pequeños, DIFICULTAD PARA LEVANTARSE DE UNA SILLA E INICIAR LA MARCHA la cual se describe como

FESTINANTE CON PASOS CORTOS Y RÁPIDOS (APRESURADA).

2) Dermatológica: SEBORREA DE CUERO CABELLUDO y cara con aumento de la sudoración. 3) Psiquiátrica: DEPRESIÓN.

Page 397: ENARM  COMPENDIO

PARKINSON• Que es la enfermedad de Wilson en el dx diferencial de Parkinson?• R = Inicio a temprana edad, ANILLOS DE KAYSER-FLEISCHER, hepatitis crónica y aumento de la CONCENTRACIÓN

TISULAR DE COBRE.

• - A 24-year-old man presents with mild jaundice, tremor, and personality changes. Examination reveals slowness of finger movement, rigidity, and coarse tremor of the outstretched hands. As well there is abnormal slow movement of the tongue and pharynx resulting in a change in speech and occasional difficulty swallowing. He is icteric, the liver span is 10 cm, and no spleen is palpable. Which of the following findings is most likely seen in this condition?

• (A) a reduction of copper excretion in the urine• (B) an increase of the serum ceruloplasmin content• (C) no renal involvement• (D) retention of normal neurologic movements• (E) a peculiar greenish-brown pigmentation of the cornea

• - In Wilson’s disease, there is usually a reduction of the serum ceruloplasmin content. Signs and symptoms of injury to the basal ganglia are accompanied by cirrhosis of the liver. Renal involvement is characterized by persistent aminoaciduria. The most common neurologic finding is tremor. The corneal pigmentation (Kayser-Fleischer ring) is the most important diagnostic finding on physical examination. If it is absent, any neurologic findings cannot be ascribed to Wilson’s disease.

• Cual es el estándar de oro en el diagnostico de Parkinson?1) La NECROPSIA2) RM donde se observa REDUCCIÓN DE LA SUSTANCIA NEGRA PARS COMPACTA en la porción mesencefalica superior.3) PET donde hay MENOR CAPTACIÓN del REACTIVO FLUORO-1-DOPA en el cuerpo estriado.

Page 398: ENARM  COMPENDIO

PARKINSON• Cual es el tratamiento del Parkinson?1) LEVODOPA en pacientes MAYORES A 70ª que es AGONISTA DE LA DOPAMINA utilizada en

combinación con CARBIDOPA prevenir la conversión periférica de la levodopa a dopamina por acción de la ENZIMA DOPADESCARBOXILASA.

2) ANTICOLINERGICOS: en pacientes MENORES A 70ª, CONTRAINDICADOS EN HPB Y GLAUCOMA DE ÁNGULO CERRADO.

• Que mecanismo de acción tiene la carbidopa?• R = INHIBE LA ENZIMA QUE CONVIERTE LA LEVODOPA A DOPAMINA

• Cuales son los agonistas de la dopamina utilizados en Parkinson?• R = BROMOCRIPTINA y pergolida.

• Como actúan los inhibidores de la COMT?• R = Inhibidores de la Catecolamina-0-Metiltransferasa REDUCEN EL METABOLISMO DE LA

LEVODOPA A 3- METILDOPA y por lo tanto alteran la farmacocinética de la levodopa en plasma llevando a NIVELES MAS SOSTENIDOS DE DOPAMINA PLASMÁTICA: TOLCAPONA/ ENTAGAPONA

• Cuales son los fármacos ANTIPSICOTICOS en Parkinson?• R = RISPERIDONA, clozapina: pudiendo esta causar depleción de MO

Page 399: ENARM  COMPENDIO

PARKINSON• Que medida quirúrgica se puede utilizar en Parkinson refractario a tx medico?• R = TALAMOTOMIA o PALIDOTOMIA de 1 solo lado.

• - Which of the following is the most likely finding in a 79-year-old woman with Parkinson’s disease?

• (A) constant fine tremor• (B) muscle atrophy• (C) akinesia• (D) pupillary constriction• (E) spontaneous remission

• - The characteristic triad in Parkinson’s disease (Tremor, Rigidity, Akinesia) has been expanded to include Postural instability. This forms the mnemonic TRAP. Autonomic instability is also common. Findings on examination also include masklike facies, dysarthria, stooped posture, and abnormal gait

• Que fármacos producen Parkinsonismo?1) FENOTIACINAS, tioxantenos, butiprofenonas, neurolépticos, valproato y fluoxetina. 2) Produciendo SÍNTOMAS PARECIDOS A LOS DEL PARKINSON causando distonias, rigidez, fascies en mascara, bradicinesia y temblor con

menor frecuencia.3) Los síntomas desaparecen una vez suspendido el fármaco.

• Que entidades conforman al Parkinsonismo plus?• R = Son síndromes que incluyen SIGNOS Y SÍNTOMAS DE LA ENFERMEDAD DE PARKINSON RELACIONADOS CON ANORMALIDADES EN

PROTEÍNA TAU tales como:1) Paralisis supranuclear progresiva: Es un trastorno neurodegenerativo que se presenta en la 6ta a 7ma década de la vida y se caracterizan

por rigidez, acinesia o bradicinesia, inestabilidad, caídas, disartria, disfagia y demencia. En la RM se observa atrofia del coliculo superior en el mesencéfalo. La respuesta al tratamiento con agonistas de dopamina y levodopa es pobre.

2) Degeneracion corticobasal: Los síntomas se desarrollan en la sexta década de la vida incluyendo rigidez focal o asimétrica, bradicinesia, temblor de acción y reposo y distonía marcada. En la RM se observa perdida cortical localizada en la zona superior de los lóbulos frontales y parietal contralaterales.

3) Enfermedad difusa de los cuerpos de Lewy: Es un TRASTORNO NEURODEGENERATIVO caracterizado por la presencia de síntomas parkinsonianos y alteraciones neuropsiquiatricas comúnmente acompañados por DEMENCIA.

Page 400: ENARM  COMPENDIO
Page 401: ENARM  COMPENDIO
Page 402: ENARM  COMPENDIO

ALZHEIMER• Cual es la manifestación principal de la enfermedad de Alzheimer?• Déficit de ACETILCOLINESTERASA• La PERDIDA DE MEMORIA a corto plazo

• Cual es la manifestación encontrada en TAC y Biopsia en Alzheimer?1) Alzheimer’s disease can be quite diffuse, but there is particular INVOLVEMENT OF THE MEDIAL

TEMPORAL LOBES AND CORTICAL ASSOCIATION AREAS. 2) THE ATROPHY OF THE HIPPOCAMPUS IS PARTICULARLY MARKED. 3) Microscopic examination reveals NEUROFIBRILLARY TANGLES AND AMYLOID PLAQUES.

• - A 69-year-old woman presents to the clinic with memory difficulty. The patient’s daughter is concerned because she is having difficulty doing her finances, such as paying bills. Memory impairment testing reveals the poor ability to generate lists of words or copy diagrams (intersecting pentagons). Her remaining physical examination is normal. Which of the following anatomic findings is most likely with her condition?

• (A) atrophy of the medial temporal lobes• (B) atrophy of the entire frontal and temporal lobes• (C) cranial nerve involvement• (D) transient episodes of hemiplegia• (E) atrophy of the caudate

Page 403: ENARM  COMPENDIO

HUNTINGTON• En que gen se ha identificado la enfermedad de Huntington?

• R = Herencia AD CROMOSOMA 4• Cual es el cuadro clínico de la enfermedad de Huntington?• R = Aparece entre los 30-50 años. MOVIMIENTOS ANORMALES y CAMBIOS INTELECTUALES CON IRRITABILIDAD, ESTADO DEL

ANIMO, TERMINANDO EN DEMENCIA.

• - A 38-year-old man presents with involuntary facial grimacing, shrugging of the shoulders, and jerking movements of the limb. His father was similarly affected. There is also a history of mood changes for the past 3 months. On examination, he appears restless with intermittent slow movements of his hands and face. He has difficulty performing rapid finger movements, and tone is decreased in the upper and lower limbs. Which of the following is most likely to represent the progression of his illness?

• (A) a normal life span• (B) a 50% chance of only male children being similarly affected• (C) mental deterioration• (D) eventual development of rigidity• (E) development of hemiparesis

• - This is a case of Huntington’s chorea. It is an autosomal dominant gene (found on the short arm of chromosome 4), and male and female children are equally affected. Movement disorder, mental deterioration, and personality change are the hallmarks of the disease, but can be very subtle initially. The disease starts typically between ages 35 and 40 (although the variation is wide) and runs its course in about 15 years. The akinetic rigid variety (Westphal variant) of Huntington’s typically has a childhood onset.

Page 404: ENARM  COMPENDIO

HUNTINGTON• Que datos de TAC se obtienen en corea de Huntington?• R = Atrofia cerebral y del núcleo caudado

• Cuando hay corea sin antecedentes se duda del dx, pero que causa la corea de Syndenham?

• R = Es autolimitada por infección por estreptococo del Gpo A

• Que padecimiento AD con datos clínicos similares a CH se debe a mutación del gen 12 y con ascendencia japonesa?

• R = Atrofia dentorubro palidolisiana con tx igual a CH

• Cual es el neurotransmisor involucrado en corea de hungtinton y que tratamiento bloqueador de receptores de ese neurotransmisor controlan la discinecia y cambio conductual?

• R = GABA y se utiliza HALOPERIDOL

Page 405: ENARM  COMPENDIO

DEMENCIA POR CUERPOS DE LEWY

Page 406: ENARM  COMPENDIO
Page 407: ENARM  COMPENDIO
Page 408: ENARM  COMPENDIO

GILLES DE LA TOURETTE• Que caracteriza al síndrome de Gilles de la Tourette?• R = Los TICS MOTORES constituyen el 80% siendo común >21ª

afectando PRINCIPALMENTE LA CARA.

• Cual es el cuadro clínico del síndrome de Gilles de la Tourette?1) Tics motores: Olfateo, PARPADEO, fruncir el ceno, ENCOGER

LOS HOMBROS O ASENTIR CON LA CABEZA. 2) Tics fónicos: Murmullos, quejidos, ladridos y aullidos,

coprolalia.

• Cual es el tratamiento de elección del sx de Gilles de la Tourette?

• R = HALOPERIDOL

Page 409: ENARM  COMPENDIO

ESCLEROSIS MULTIPLE• Que caracteriza a la esclerosis múltiple?1) Quizá tenga base autoinmune con desmielinizacion con gliosis reactiva en sustancia blanca encefálica y de la medula espinal2) Se le ha asociado con Chlamydia pneumoniae, virus herpes 6 y adenovirus

3) El gen HLA-DRB1 en el cromosoma 6p21 es, entre los muchos genes HLA asociados con esclerosis múltiple.

4) El OLIGODENDROCITO encargado de la mielinizacion ES EL BLANCO PRINCIPAL del proceso inflamatorio principalmente mediado por LT CD 8 CITO TÓXICOS y macrófagos.

5) Patológicamente se distingue por placas de desmielinizacion afectando mas frecuentemente la sustancia blanca periventricular

• Cual es la clasificación de la esclerosis múltiple?1) Existen 4 tipos básicos de esclerosis múltiples según su progresión en brote - remisión, primaria progresiva, secundaria progresiva y

progresiva-recurrente.

2) 90% de los pacientes comienza con el TIPO BROTE- REMISIÓN caracterizado por la presentación de uno a dos ataques por year con remisiones y recuperaciones en los meses posteriores.

• Cual es el cuadro clínico de la esclerosis múltiple?

1) La PRESENTACIÓN COMÚN ES DEBILIDAD, ENTUMECIMIENTO, HORMIGUEO, INESTABILIDAD DE UNA EXTREMIDAD, paraparesia espástica, NEURITIS RETROBULBAR, trastornos esfinterianos. Los síntomas desaparecen de días a semanas. En el examen aparecen ATROFIA ÓPTICA, nistagmo, disartria y déficit sensitivo en extremidades.

2) Patrón ataque - recaída - exacerbación, definiéndose ataque como el cuadro agudo de disfunción del SNC que dure mas de 24 hrs en ausencia de fiebre, infecciones o alteraciones metabólicas.

3) FENÓMENO DE UTHOFF, el cual consiste en la aparición o exacerbación de los síntomas tras el ejercicio o un baño con agua caliente

4) TANTO LAS TAREAS COGNITIVAS COMO LA ACTIVIDAD FÍSICA PROVOCAN CANSANCIO PROFUNDO Y LOS PACIENTES TARDAN MAS TIEMPO EN RECUPERARSE

5) El CANSANCIO es uno de los síntomas mas comunes y discapacitantes de la enfermedad.

6) La afectación de la vía visual es casi una regla con la presencia de NEURITIS ÓPTICA que se manifiesta como CEGUERA MONO O BINOCULAR.

7) EL SIGNO DE LHERMITTE (sensación de electricidad que va de la medula hacia las extremidades al flexionar el cuello).

Page 410: ENARM  COMPENDIO

ESCLEROSIS MULTIPLE• Cual es el estudio de elección para dx de esclerosis múltiple?

1) IRM de ENCÉFALO o MEDULA ESPINAL que demuestra MIELOPATIA FOCAL O MÚLTIPLE EN LA SUSTANCIA BLANCA. Se aplican los criterios de Barkhof y Tintore.

2) El estudio del LCR permite valorar la inflamación intratecal. SE BUSCAN BANDAS OLIGOCLONALES DE

IgG que no están presentes en suero. Un total de PROTEÍNAS MENOR A 100 Y LEUCOCITOS MENORES A 50 ES LO COMÚN

3) Los POTENCIALES EVOCADOS VISUALES son los mas SENSIBLES Y ESPECÍFICOS, los cuales DENOTAN DESMIELINIZACION.

• En que consiste la malformación de Arnold Chari, la cual debe descartarse en esclerosis múltiple?• R = Observándose la región del agujero magno UNA PARTE DEL CEREBELO Y PARTE INFERIOR DEL TALLO

ENCEFÁLICO SE DESPLAZAN HACIA EL CONDUCTO CERVICAL produciendo déficit piramidal y cerebeloso en extremidades.

• Cual es el tratamiento de esclerosis múltiple?

• R = INTERFERON B disminuye exacerbaciones al igual que inmunoglobulina IV. NO HAY FORMA

DE DETENER EL AVANCE, se utiliza PREDNISONA EN RECIDIVAS.

• Cual es el pronostico de la esclerosis múltiple?• R = Mientras mayor sea el numero y volumen de las lesiones peor será el pronostico.

Page 411: ENARM  COMPENDIO

ESCLEROSIS MULTIPLE• -A 22-year-old woman presents with acute vision loss and pain in the left eye, but no other symptoms. On examination she

appears well, visual acuity is not possible, and she can only perceive movement and bright light. The direct papillary reflex is absent but the indirect (consensual) response is normal. The optic disc is edematous. Which of the following symptoms is also most likely present in patients with this condition?

• (A) limb weakness• (B) hemiplegia• (C) cervical myelopathy• (D) sphincter impairment• (E) seizures

• - This patient has multiple sclerosis (MS). Weakness or numbness in one or more limbs is the initial manifestation of disease in about half the patients. Other common initial presentations include optic neuritis (25%) and acute myelitis. Hemiplegia, seizures, and cervical myelopathy (in older patients) occur occasionally as the initial manifestation. Sphincter impairment usually occurs later in the disease.

• -A 27-year-old woman presents with acute vision loss and pain in the left eye, but no other symptoms. On examination, she appears well, visual acuity is not possible, and she can only perceive movement and bright light. The direct pupillary reflex is absent but the indirect (consensual) response is normal. The optic disc is edematous. Which of the following is the most likely diagnosis?

• (A) diabetic microvascular disease• (B) arteriosclerosis• (C) trauma• (D) multiple sclerosis• (E) Creutzfeldt-Jakob disease

• -Visual loss in multiple sclerosis varies from slight blurring to no light perception. Other eye symptoms include diplopia and pain. The classic syndrome of optic or retrobulbar neuritis occurs commonly at some point in the disease, and it is the presenting symptom in 25% of cases.

Page 412: ENARM  COMPENDIO
Page 413: ENARM  COMPENDIO

EPILEPSIA

Page 414: ENARM  COMPENDIO

EPILEPSIA

Page 415: ENARM  COMPENDIO
Page 416: ENARM  COMPENDIO
Page 417: ENARM  COMPENDIO

ATAXIA DE FRIEDREICH• Que es la ataxia de Friedreich?

1) AR, en CROMOSOMA 9 q13-9 q21.

2) Es una MARCHA ATÁXICA, manos torpes, respuesta extensora plantar y disminución de ROTS.

3) The pathologic changes are found in the SPINAL CORD TRACTS. Degeneration is seen in the POSTERIOR COLUMNS, the lateral corticospinal tract, and the spinocerebellar tracts.

4) PIE CAVO BILATERAL

• -A 19-year-old man has had progressive ataxia of gait and great difficulty in running. In the past year, he has developed hand clumsiness. Physical examination reveals pes cavus, kyphoscoliosis, and both cerebellar and sensory changes in the legs. There is a positive family history of Friedreich’s ataxia. Where are the pathologic changes seen in this condition most likely to be found?

• (A) spinal cord tracts• (B) basal ganglia• (C) cerebral cortex• (D) peripheral autonomic nerves• (E) peripheral motor nerves

• - This young man has Friedreich’s ataxia, associated with a gene defect on chromosome 9. The pathologic changes are found in the spinal cord tracts. Degeneration is seen in the posterior columns, the lateral corticospinal tract, and the spinocerebellar tracts. Ataxia, sensory loss, nystagmus, reflex changes, clubfeet, and kyphoscoliosis are the characteristic findings. The heart is frequently involved, and cardiac disease is a common cause of death.

Page 418: ENARM  COMPENDIO

DEFICIT VITAMINA E y B12• Que produce el déficit de vitamina E a nivel SNC?

• R = Se presenta degeneración medulocerebelosa que afecta

principalmente al cordón posterior de la medula espinal.

• Cuales son los datos clínicos de déficit de vitamina E?

• R = Ataxia de extremidades, disminución sensitiva, disminución de ROTS,

confusión y degeneración pigmentaria de la retina.

• Cual es el tratamiento para el déficit de vitamina E?• R = Alfa tocoferilo

• Que trastorno en el SNC te ocasiona el déficit de vitamina B12?

• Degeneración combinada subaguda de medula espinal.

• Se acompaña de polineuropatía, cambios mentales o neuropatía óptica.

Page 419: ENARM  COMPENDIO

ENCEFALOPATÍA DE WERNICKE• Que caracteriza la encefalopatía de Wernicke?

• R = Déficit de “TIAMINA B1” y es COMÚN EN BORRACHOS.

• Cual es el cc de la encefalopatía de Wernike?

• R = CONFUSIÓN, ATAXIA Y NISTAGMO que da lugar a oftalmoplejia (parálisis de la mirada conjugada, debilidad del musculo recto externo)

• -A 43-year-old man is referred from the emergency department with memory loss and difficulty walking. He was brought in by his wife who has noticed personality changes, truancy from work, and lack of personal care over the past 2 years. On examination he appears unkempt, smells of urine, and is uncooperative. He cannot recall the date or season, and gets angry when asked questions. His answers are often fabricated when checked with his wife. His gait is wide-based, and there is loss of sensation in his feet up to the shins. His motor strength and reflexes are normal. His ocular movements are normal and there is no nystagmus. In the past he has had multiple admissions for alcohol withdrawal. Which of the following is the most likely diagnosis?

• (A) Wernicke’s encephalopathy• (B) Wernicke-Korsakoff syndrome• (C) Alzheimer’s dementia• (D) Charcot-Marie-Tooth disease• (E) vascular dementia

• -The combination of symptoms is typical of chronic alcohol abuse. The mental symptoms are suggestive of Wernicke-Korsakoff syndrome. Adistal limb sensory-motor neuropathy is also typical of alcoholism. Confusion, tremulousness, and disorientation are typical for acute alcohol intoxication. Wernicke’s encephalopathy is a symptom complex of ophthalmoplegia, ataxia, nystagmus, and acute confusional state

Page 420: ENARM  COMPENDIO

DEGENERACIÓN COMBINADA SUBAGUDA

• Cual es la etiología de la degeneración combinada subaguda y cual es el cuadro clínico característico?

1) Es causado por la deficiencia de COBALAMINA (VB12)2) En un inicio encontramos signos y síntomas de lesión de la

COLUMNA POSTERIOR (PARESTESIAS DE MANOS Y PIES, INESTABILIDAD DE LA BIPEDESTACIÓN Y MARCHA, ALTERACIONES DE VIBRACIÓN Y POSICIÓN)

3) Después de un tiempo a las 2 semanas posteriores se identifica una paraparesia atáxica simétrica con hiperreflexia o hiporreflexia tendinosa y signo de Babinski.

Page 421: ENARM  COMPENDIO

VITAMINAS HIDROSOLUBLES

Page 422: ENARM  COMPENDIO

VITAMINAS LIPOSOLUBLES

Page 423: ENARM  COMPENDIO

LYME

• Cuales son las manifestaciones neurológicas de la enfermedad de Lyme?

1) Meningitis, 2) Poliradiculopatia, 3) Mononeuropatia multiple y 4) Neuropatía cutánea.

Page 424: ENARM  COMPENDIO

SARCOIDOSIS

• Cuales son las manifestaciones neurológicas de la sarcoidosis?

• R = Parálisis de los pares craneales, en especial el 7 que responde a esteroides

Page 425: ENARM  COMPENDIO

SÍNDROME DE GUILLAIN-BARRE• Cual es la etiología del síndrome de Guillain-Barre?1) Suele asociarse a una infección de vías respiratorias o gastrointestinal.

2) Principalmente el CAMPYLOBACTER JEJUNI Y CMV.

• Cuales son los signos y síntomas de la poliradiculoneuropatia progresiva aguda/sub aguda conocido como síndrome de Guillain- Barre?

• R = Debilidad ascendente de comienzo en piernas.

• Cuales son los estudios para el diagnostico de síndrome de Guillain- Barre?

1) LCR con AUMENTO DE PROTEÍNAS2) Células normales3) Electrofisiología con alteraciones

4) Histología que muestra DESMIELINIZACION PRIMARIA

• Cual es el manejo del síndrome de Guillain-Barre?• R = Administración de INMUNOGLOBULINA IV O PLASMAFERESIS.

Page 426: ENARM  COMPENDIO

MIASTENIA GRAVE• Cual es el cuadro clínico de la miastenia grave?

1) DIPLOPÍA, debilidad de músculos oculares, debilidad progresiva pudiendo causar parálisis respiratoria, PTOSIS.

2) El cuadro clínico confirma DEBILIDAD Y FATIGA DE MÚSCULOS EL CUAL MEJORA CON EL REPOSO.

• Como se realiza el diagnostico de miastenia grave?

1) Respuesta a ANTICOLINESTERASA de acción corta con EDROFONIO con vida media de 1 minuto en el cual los PACIENTES CON RESPUESTA TIENEN MEJORÍA NOTORIA DURANTE 5 MINUTOS.

2) NEOSTIGMINA, la respuesta dura 2 hrs, con sulfato de atropina en caso de efectos muscariniocos adversos.

3) ELECTROFISIOLOGIA que muestra DISMINUCIÓN DE LA TRANSMISIÓN.

4) LAB con Ac VS RECEPTOR DE ACETILCOLINA

• Cual es el tratamiento de miastenia grave?• R = NEOSTIGMINA. Timectomia en caso de timoma. PROSCRITOS

AMINOGLUCOSIDOS por que causan bloqueo neuromuscular

Page 427: ENARM  COMPENDIO

MIASTENIA GRAVE• - A 30-year-old woman complains of double vision, and easy fatigue with exercise. The

fatigue improves with resting, but it is interfering with her work. Examination reveals ptosis and impaired eye movements with normal pupillary response. The double vision is brought out by asking her to look at the ceiling, and after a sustained interval, the eyes slowly drift down. Which of the following is the most likely diagnosis?

• (A) optic atrophy• (B) ophthalmic zoster• (C) paralysis agitans• (D) Horner syndrome• (E) myasthenia gravis

• - In myasthenia gravis, weakness of the facial and levator palpebrae muscles produces a characteristic expressionless face, with drooping of the eyelids. Weakness of the ocular muscles may cause paralysis or weakness of individual muscles, paralysis of conjugate gaze, ophthalmoplegia, or a pattern similar to internuclear ophthalmoplegia. The presence of normal pupillary responses to light and accommodation with weakness of extraocular muscles, levators, and orbicularis oculi is almost completely diagnostic of myasthenia

Page 428: ENARM  COMPENDIO

SÍNDROME DE LAMBERT-EATON O MIASTENIFORME

• Que puede ocasionar el síndrome de lambert-eaton o miasteniforme?

1) En este hay liberación DEFECTUOSA DE ACETILCOLINA. 2) Se presenta con DEBILIDAD DE MÚSCULOS PROXIMALES y se diferencia

de miastenia gravis por que LA FUERZA SE AUMENTA CON CONTRACCIÓN SOSTENIDO.

• Como diagnosticas el síndrome de lambert-eaton?

• R = Electrofisiología por AUMENTO DE LA RESPUESTA MUSCULAR A LA ESTIMULACIÓN

• Cual es el manejo del síndrome de lambert-eaton?

• R = PLASMAFERESIS, inmunodepresores (azatropina, prednisona).

Page 429: ENARM  COMPENDIO

NEUROINFECCION• Ayuda la toma de LCR en el caso de absceso cerebral?• R = NO, de echo ESTA PROSCRITO por que puede causar herniación.

• Cual es la complicación mas frecuente del SIDA en el SNC?• R = This is one of the most common neurologic complications of AIDS. Its pathologic substrate is degeneration

of the spinal tracts in the posterior and lateral columns, which have a vacuolated microscopic appearance. Although the morphologic changes and clinical manifestations are similar to those associated with vitamin B12 deficiency, the pathogenetic mechanism is probably not related to dietary deficiencies. Since there is no specific clinical or laboratory test available for the diagnosis of this syndrome, VACUOLAR MYELOPATHY IN AIDS PATIENTS REMAINS A DIAGNOSIS OF EXCLUSION. This implies that other HIV-related neurologic complications must be ruled out

• -A 56-year-old man is brought to the emergency department by his wife because of memory loss and difficulty walking. She has noticed personality changes, truancy from work, and lack of personal care over the past 1 year. On examination he appears unkempt, smells of urine, and is uncooperative. He cannot recall the date or season,and gets angry when asked questions. His answers are often fabricatedwhen checked with his wife. The blood pressure is 150/90 mm Hg, pulse 100/min, and he is diaphoretic and tremulous. His gait is wide based, and motor strength and reflexes are normal. His ocular movements are normal but there is nystagmus on lateral gaze. In the past he has had multiple admissions for alcohol withdrawal. Which of the following is the most appropriate next step in management?

• (A) prophylactic phenytoin administration• (B) prophylactic diazepam administration• (C) prophylactic carbamazepine administration• (D) calcium administration• (E) steroid administration

• - Prophylactic administration of diazepam in a withdrawing alcoholic can prevent or reduce severe syndromes such as delirium tremens (DTs). Prophylactic phenytoin, however, is not helpful. Acalm, quiet environment with close observation and frequent reassurance is very important. Vitamin administration (especially thiamine) is important, but frequently, severe magnesium depletion slows improvement.

Page 430: ENARM  COMPENDIO

LCR

Page 431: ENARM  COMPENDIO

BOTULISMO• Cual es la fisiopatología del botulismo?• R = La toxina EVITA LA LIBERACIÓN DE ACETILCOLINA en las uniones neuromusculares

y sinapsis autónomas, transmitida por COMIDAS ENVASADAS

• Cual es el cuadro clínico del botulismo?1) Desarrollo SÚBITO DE DEBILIDAD INTENSA DESPUÉS DE 72 HRS DE LA COMIDA. 2) El típico cuadro es de DIPLOPÍA, VISIÓN BORROSA, midriasis no reactiva,

estreñimiento por íleo paralitico, PTOSIS, debilidad facial, DIFICULTAD RESPIRATORIA Y DEBILIDAD EN EXTREMIDADES INFERIORES CON SENSIBILIDAD CONSERVADA.

• Como se dx el botulismo?• R = SUERO CON TOXINA. Electrofisiológico con estimulación repetida con aumento de

la respuesta muscular de manera progresiva

• Cual es el manejo del botulismo?1) ANTITOXINA en pacientes NO ALÉRGICOS AL SUERO DE CABALLO. 2) GUANIDINA facilita la liberación de acetilcolina

Page 432: ENARM  COMPENDIO

MIGRANA

• Cual es el cuadro clínico de la migraña?1) Cefalea intensa, pulsátil, unilateral en la región frontal o temporal. 2) La migraña con aura consiste en un inicio con alteraciones visuales con luces-

centilleos o luces de colores, palidez, vértigo, alteraciones sensitivas o debilidad unilateral,

3) Afasia transitoria o alteraciones en el lenguaje que aparecen previamente al establecimiento de la migraña.

• A que se refiere el termino estatus migranoso?• R = Se refiere a pacientes ya conocidos con migraña con manifestaciones clínicas

por mas de 72 Hrs

• Cual es el manejo de la migraña?• R = Triptanos y derivados de la ergotamina

Page 433: ENARM  COMPENDIO

MIGRANA• Cuales son las 2 variantes de migraña a considerar?1) Los síntomas prodrómicos son el resultado de una disfunción en el territorio de la circulación

cerebral posterior con sintomatología visual bilateral, ataxia, disartria, vértigo, parestesias de las extremidades e incluso debilidad. Puede haber perdida de la conciencia antes de comenzar la cefalea.

2) Migraña hemipléjica: Es poco frecuente e incluye hemiplejia, que puede persistir días después de calmarse la cefalea.

• - An 18-year-old woman has periodic episodes that begin with severely decreased vision, followed by ataxia, dysarthria, and tinnitus. The symptoms last for 30 minutes and are then followed by a throbbing occipital headache. Which of the following is the most likely diagnosis?

• (A) vertebral-basilar insufficiency• (B) chronic basilar artery dissection• (C) classic migraine• (D) ophthalmoplegic migraine• (E) basilar migraine

• Basilar migraine can be very dramatic, and can resemble ischemia in the territory of the basilar posterior cerebral arteries. The visual symptoms of basilar migraine typically affect the whole of both visual fields, and can even cause temporary cortical blindness. There can also be an alarming period of coma or quadriplegia.

Page 434: ENARM  COMPENDIO

CEFALEA TENSIONAL

• Cual es el cuadro clínico de la cefalea tensional?1) Dolor difuso, en banda, de carácter sordo, que

puede acompañarse de hiperestesia en el cuero cabelludo y agravarse con el ruido y la luz.

2) Típicamente la distribución del dolor es en forma de banda, bilateral y se extiende desde la frente hacia los temporales y nuca.

3) Como profilaxis pueden utilizarse antidepresivos triciclicos y la amitriptilina es la más eficaz.

Page 435: ENARM  COMPENDIO

CEFALEA EN RACIMOS O ENFERMEDAD DE HORTON

• Que caracteriza a la cefalea en racimos o enfermedad de Horton?

1) Intenso dolor unilateral periorbitario acompañado de inyección conjuntival, lagrimeo, rinorrea y síndrome de Horner

2) El comienzo del dolor suele ser en la madrugada o al despertar muy temprano.

3) Para su tratamiento son útiles los fármacos utilizados en migraña.

4) Nifedipino y berapamil como parte del manejo profiláctico.

Page 436: ENARM  COMPENDIO

NARCOPLEPSIA• Cual es la sintomatología en caso de narcolepsia?• R = The early age of onset and otherwise good health suggest a diagnosis of narcolepsy, which is usually accompanied by other

symptomatology. Hypnagogic hallucinations are almost always visual. They occur most frequently at the onset of sleep, either during the day or at night. They are generally very vivid. Cataplexy is a brief loss of muscle power without loss of consciousness. The patient is fully aware of what is going on. The paralysis may be complete or partial.

• -A 31-year-old woman complains of excessive sleepiness during the daytime for years despite adequate nighttime sleep. She has episodes of intense drowsiness three to four times a day, even when at work or while eating meals. She has sought medical attention in the past, after falling asleep while driving. She is slender and otherwise healthy and on no medications. Which of the following treatments is most likely indicated for her condition?

• (A) a device providing continuous positive airway pressure (CPAP) at night• (B) oral surgery• (C) tracheostomy• (D) amphetamines• (E) benzodiazepines at bedtime

• -This woman does not have risk factors for sleep apnea (older age, snoring, obesity) and likely has narcolepsy. Adrenergic stimulant drugs such as methylphenidate or amphetamines help the sleepiness, and tricyclic compounds can help the cataplexy. Strategically planned naps can also be helpful.

• -A 25-year-old man complains of excessive sleepiness during the daytime for years despite adequate nighttime sleep . He has sought medical attention after falling asleep while driving. He is slender and otherwise healthy and on no medications. Which of the following symptoms might he also complain about?

• (A) excessive snoring (wife’s report)• (B) automatic behavior (wife’s report)• (C) restless sleep (wife’s report)• (D) paresthesias• (E) morning headache

• -Automatic behavior with amnesia is a common manifestation of the narcolepsycataplexy syndromes, occurring in 50% of cases. Automatic behavior can be confused with complex partial seizures. Paresthesias are not part of narcolepsy syndrome. Snoring, restless sleep, and morning headache suggest sleep apnea.

Page 437: ENARM  COMPENDIO

ENDOCRINOLOGIA

ENARM

Page 438: ENARM  COMPENDIO

SINDROME METABOLICO

• PARA EL DIAGNOSTICO DE ESTE SÍNDROME ES NECESARIO LA PRESENCIA DE 3 O MAS DE LOS SIGUIENTES CRITERIOS:

– OBESIDAD ABDOMINAL MUJERES > 88 CMS Y HOMBRE > 102 CMS.

– TRIACILGLICERIDOS >O IGUAL A 150 MG/DL O CON TRATAMIENTO.

– HDL <40 MG/DL (HOMBRES) Y < 50 MG/DL (MUJERES).– TENSIÓN ARTERIAL SISTÓLICA >O IGUAL A 130 MMHG Y

DIASTÓLICA >O IGUAL A 85 MMHG.– GLICEMIA > 110 MG/DL O CON TRATAMIENTO

HIPOGLICEMIANTE.

Page 439: ENARM  COMPENDIO
Page 440: ENARM  COMPENDIO

HIPERPROLACTINEMIA• Cual es una hormona inhibidora de la prolactina?• R = DOPAMINA, el aumento de estrógenos la inhibe.

• Cuales son los signos y síntomas de hiperprolactinemia?

1) Hombres: DISFUNCIÓN ERÉCTIL, HIPOGONADISMO HIPOGONADOTROFICO, INFERTILIDAD y disminución de la libido. En ocasiones GINECOMASTIA SIN GALACTORREA.

2) Mujeres: OLIGOMENORREA, amenorrea, GALACTORREA, si el hipogonadismo no es tratado da osteoporosis.

• Que lab se solicitan en hiperprolactinemia y que padecimientos se encuentran relacionados?

• R = HCG en caso de embarazo, HIPOTIROIDISMO, IR y cirrosis.

• Que estudio realizas en hiperprolactinemia no asociada a hipotiroidismo, embarazo o inducida por fármacos?

• R = IRM buscando micro <2 cm o macroadenoma hipofisiario >3 cm.

• Que medicamentos no deben recibir las mujeres con prolactinomas?

• R = ESTRÓGENOS (<2cm no afectan), TETOSTERONA o EMBARAZO.

• Cual es el tratamiento para hiperprolactinemia?

• R = BROMOCRIPTINA. Antagonistas de la dopamina como CENTENO O CABERGOLINA al acostarse, Quinagolida a los intolerantes derivados del cornezuelo de centeno

Page 441: ENARM  COMPENDIO

SSIADH• En que región del hipotálamo se secreta la ADH y que funciones tiene?1) En el NÚCLEO SUPRA ÓPTICO

2) La hormona antidiurética (ADH), o arginina vasopresina (AVP), es una hormona LIBERADA PRINCIPALMENTE EN RESPUESTA A CAMBIOS EN LA OSMOLARIDAD SÉRICA O EN EL VOLUMEN SANGUÍNEO. También conocida como argipresina. Hace que los riñones conserven agua mediante la concentración de orina y la reducción de su volumen, estimulando la reabsorción de agua. Recibe su nombre de esta importante función como regulador homeostásico de fluidos.

• Cual es la forma mas común de HIPONATREMIA NORMOVOLEMICA?• R = SSIADH

• Cual es el cc del SSIADH?1) Neuromusculares: IRRITABILIDAD, debilidad muscular, letargo, CONFUSIÓN y coma.2) Gastrointestinales: NAUSEA, VOMITO y anorexia3) La intensidad de los síntomas se relaciona con el grado de hiponatremia

• Como diagnosticas SSIADH?1) Por exclusión2) Prueba de sobrecarga hídrica y hallazgos de niveles plasmáticos inapropiadamente elevados de vasopresina en relación a

la osmolaridad plasmática

• Cual es el manejo del SSIADH?1) RESTRICCIÓN HÍDRICA 500-1000 ml/24 hrs2) DEMECLOCICLINA (inhibe la respuesta renal a la vasopresina)

Page 442: ENARM  COMPENDIO
Page 443: ENARM  COMPENDIO

PATOLOGIA DE TIROIDES• Que características clínicas acompañan a la tirotoxicosis?• R = Nerviosismo, PALPITACIONES, TAQUICARDIA, fatiga, debilidad, pérdida de peso con buen apetito,

DIARREA, INTOLERANCIA AL CALOR, PIEL CALIENTE, transpiración excesiva, labilidad emocional, alteraciones menstruales, TEMBLOR FINO EN LAS MANOS, alteraciones oculares y aumento variable de la glándula. FA.

• Como se manifiesta la CRISIS O TORMENTA TIROIDEA?

• SE MANIFIESTA POR DELIRIO, TAQUICARDIA, VOMITO, DIARREA y FIEBRE.

• Cual es el manejo de la tormenta tiroidea?

• R = Dosis altas de PROPILTIUORACILO, yodo o contrastes yodados, PROPANOLOL Y

DEXAMETASONA

• Cuales son los análisis mas específicos para enfermedad de Graves?

• R = TSH-R AB 2da generación y TSH-R recombinante son > 75%

• Que diferencia hay entre la captación de yodo de las siguientes patologías?1) Enfermedad de Graves y bocio multinodular toxico: ALTO2) Tiroiditis sub aguda: BAJO

Page 444: ENARM  COMPENDIO

PATOLOGIA DE TIROIDES• La enfermedad de GRAVES es la causa mas común de TIROTOXICOSIS, que la caracteriza?1) Enfermedad de Basedow en Europa. 2) ETIOLOGÍA AUTOINMUNE, aumento de hormonas tiroideas, AUMENTO DE GLÁNDULA, AC VS TSH. 3) Mujeres 8:1. SE RELACIONA con enfermedades autoinmunes como ANEMIA PERNICIOSA, MIASTENIA GRAVE.

4) Asociación con HLA-B8 y HLA DR3, AC VS PEROXIDASA, AC VS TIROGLOBULINA y ANA están presentes

• Que es la enfermedad de Plummer en tirotoxicosis?• R = Son adenomas tóxicos tiroideos simples

• Que es la enfermedad de Jodbasedow en tirotoxicosis?• R = Hipertiroidismo inducido por yodo

• Que patologías extraglandulares originan tirotoxicosis?

• R = Aumento en la HCG, EMBARAZO MOLAR, CORIOCARCINOMA y neoplasias testiculares. Administración de AMIODARONA en arritmias.

• Cuales son los signos y síntomas de tirotoxicosis?

1) Mirada fija, CAÍDA DE PARPADO, taquicardia o FA, TEMBLORES FINOS, PIEL HUMEDA/CALIENTE.

2) GRAVES: Oftalmopatia con quemosis, conjuntivitis y PROPTOSIS. DERMOPATIA “MIXEDEMA PRETIBIAL” con ACUMULACIÓN DE GLUCOSAMINOGLUCANOS que ocasiona edema con textura rugosa.

Page 445: ENARM  COMPENDIO

PATOLOGIA DE TIROIDES• Cual es el manejo de la enfermedad de Graves?• 1. PROPANOLOL: taquicardia, temblores y ansiedad• 2. Derivados de la tiourea (METIMAZOL O PROPILTIURACILO): Puede causar AGRANULOSITOSIS• 3. Metimazol: < riesgo de necrosis hepática, fulminante

• 4. Propiltiuracilo: Es de ELECCIÓN en EMBARAZO O LACTANCIA• 5. Agente de contraste yodados: ayudan en tirotoxicosis de cualquier etiología los cuales inhibe la

trimonoyodinacion de T4 por lo tanto aumenta T3 y disminuye tirotoxicosis• 6. YODO RADIACTIVO: destruye tejido tiroideo sobre activo

• Que medidas se usan para complicaciones de enfermedad de Graves:• 1. Oftalmopatia de Graves: EXOFTALMUS; ESTEROIDES, progresivo-radioterapia• 2. Cardiacas: taquicardia sinusal; PROPANOLOL• 3. Insuficiencia cardiaca: DIURÉTICOS Y DIGOXINA

• Que causa dermopatia de Graves?• R = Engrosamiento de piel por glucosaminoglucanos, se le conoce como MIXEDEMA PRETIBIAL Y SU

TRATAMIENTO SON ESTEROIDES TÓPICOS.

Page 446: ENARM  COMPENDIO

PATOLOGIA TIROIDES• Que caracteriza en la tirotoxicosis a la PARÁLISIS PERIÓDICA HIPOKALEMICA TIROTOXICA?• R = PARÁLISIS FLÁCIDA SIMÉTRICA REPENTINA, HIPOKALEMIA E HIPOFOSFATEMIA. El tratamiento

es con PROPANOLOL que revierte la parálisis en 3 hrs. Dextrosa IV o CHBTS lo agravan.

• Cual es el manejo de los nódulos tiroideos solitarios tóxicos?• R = PROPANOLOL, YODO RADIACTIVO

• Cual es el manejo del bocio multinodular toxico?• R = PROPANOLOL, YODO RADIACTIVO

• Cual es el cc de la TIROIDITIS DE HASHIMOTO O TIROIDITIS LINFOCITICA CRÓNICA y a que enfermedad progresa?

1) Tiroides con AUMENTO DE TAMAÑO SIN DOLOR, de consistencia dura, ASIMÉTRICO y fijo

2) PROGRESIÓN A HIPOTIROIDISMO. 3) Depresión, fatiga crónica, XEROSTOMÍA, XEROFTALMIA.

• Que anticuerpos son los principales implicados en la tiroiditis de Hashimoto?

• R = AC ANTIMIELOPEROXIDASA y LINFOCITOS TCD 4

Page 447: ENARM  COMPENDIO

PATOLOGIA DE TIROIDES• Cual es el manejo de tiroiditis de Hashimoto al causar hipotiroidismo, bocio y Hashitoxicosis?1) Hipotiroidismo: LEVOTIROXINA2) Bocio: T4 para disminuir tamaño de glándula3) Hashitoxicosis: Aumenta liberación de T4 por acumulo, PROPANOLOL o yodato sódico

• Cuales son los medicamentos principalmente vinculados con la tiroiditis inducida por fármacos?• R = AMIODARONA, Interferon alfa, LITIO e interleucina 2.

• Cual es el cuadro clínico de la tiroiditis por fármacos?• R = Tiroiditis INDOLORA

• Que caracteriza a la TIROIDITIS DE RIEDEL /tiroiditis fibrosa invasiva, tiroiditis leñosa, tiroiditis lingeosa o tiroiditis invasiva?

1) Provoca HIPOTIROIDISMO e HIPOPARATIROIDISMO. 2) Se presenta en EDAD AVANZADA.

3) GLÁNDULA DURA CON ADHERENCIAS al cuello ocasionando DISNEA, DISFAGIA, DOLOR Y RONQUERA. 4) Los trastornos incluyen FIBROSIS RETROPERITONEAL.

• Cuales son las características laboratoriales en la producción de T3 o T4 de distinción entre los padecimientos causantes de hipertiroidismo?

1) TIROIDITIS SUB AGUDA (QUERVAIN)/ HASHIMOTO: >T4 + que T32) GRAVES/ BOCIO NODULAR TOXICO: >T3

Page 448: ENARM  COMPENDIO

PATOLOGIA DE TIROIDES• Cual es la causa de la tiroiditis aguda o supurativa?• R = Infecciosa

• Cual es el cuadro clínico de la tiroiditis aguda o supurativa?

• R = El paciente cursa con FIEBRE, disfagia, disfonía, DOLOR EN LA REGIÓN ANTERIOR DEL CUELLO, eritema y GLANDULA TIROIDEA SENSIBLE

• Que muestra la biopsia en la tiroiditis aguda o supurativa?• R = INFILTRADO LEUCOCITARIO

• Cual es el manejo de tiroiditis supurativa?• R = Antibióticos + drenaje quirúrgico

• Cual es la etiología de la tiroiditis subaguda posparto?• R = Es causada por la INFLAMACIÓN LINFOCITICA de la tiroides en los primeros 3 A

6M POSTERIORES AL PARTO.

Page 449: ENARM  COMPENDIO

PATOLOGIA DE TIROIDES• Cual es el cc de la tiroiditis subaguda posparto?

• R = EL BOCIO es INDOLORO

• Como se encuentra la VSG en la tiroiditis subaguda posparto?• R = VSG NORMAL que la DIFERENCIA DE LA DE QUERVAIN. • Cual es el manejo de la tiroiditis subaguda posparto?1) Rara vez requiere tratamiento

2) En la FASE DE TIROTOXICOSIS es útil un BETABLOQUEADOR

3) En la fase de HIPOTIROIDISMO puede usarse LEVOTIROXINA4) Los fármacos anti tiroideos y los esteroides NO ESTÁN INDICADOS

• Cual es la etiología de la tiroiditis subaguda o de Quervain?1) Suele ser precedida por un PROCESO INFECCIOSO DE LAS VAS2) Se ha relacionado con ECHOVIRUS, ADENOVIRUS, COXACKIE VIRUS Y VIRUS DE LA

PAROTIDITIS3) La edad de presentación es entre 20-40ª

Page 450: ENARM  COMPENDIO

PATOLOGIA DE TIROIDES• Cual es el cuadro clínico de la TIROIDITIS DE QUERVAIN? EL cuadro clínico comienza con pródromos de :1) MIALGIAS GENERALIZADAS2) Faringitis3) Fatigas

4) FIEBRE5) DOLOR Y EDEMA DEL CUELLO

• Como se encuentra la VSG en la tiroiditis de Quervain?

1) ELEVACIÓN DE LA VSG2) Elevación de PCR

• Cual es el manejo de tiroiditis subaguda o de Quervain?• R = ASA es de elección, PROPANOLOL para SÍNTOMAS TIROTOXICOS

• Cual es el manejo del ESTROMA DE YEDEL?• R = TAMOXIFENO de elección por años

Page 451: ENARM  COMPENDIO

PATOLOGIA DE TIROIDES• Un teratoma ovárico que patología puede ocasionar, relacionado a la producción de hormonas?• R = TIROTOXICOSIS por producción de TSH

• Como afecta el aumento de hormonas tiroideas a los huesos?• R = Aumenta la estimulación de la RESORCIÓN ÓSEA.

• Cuales son las manifestaciones clínicas del CRETINISMO?• R = Alteración del desarrollo esquelético y del SNC, CARA TOSCA, PROTRUSIÓN DE LA LENGUA

Y HERNIA UMBILICAL.

• A que se le llama síndrome de plumer?• R = Cuando uno de los nódulos del BOCIO MULTINODULAR se hace HIPERFUNCIONANTE (bocio

multinodular toxico), puesto que la mayoría de los pacientes son eutiroideos

• Porque en un paciente que utiliza corticosteroides de manera crónica se adelgaza y se ve fina la piel?

• R = Debido al CATABOLISMO PROTEICO DE LAS PROTEÍNAS consistentes en perdidas de colágeno y resorción ósea.

• Que alimentos se consideran bociogenos?• R = Mandioca, col de brucelas, repollo y coliflor

Page 452: ENARM  COMPENDIO

PATOLOGIA DE TIROIDES• Cual es la estirpe histológica mas común del carcinoma de tiroides y que lo caracteriza?

1) Carcinoma PAPILAR en 70-90%, 2) Asociado al GEN RET3) El antecedente de RADIACIÓN EN CUELLO en la infancia se relaciona 4) Manifestándose como NÓDULO ÚNICO,

5) Donde la citología muestra CÉLULAS NUCLEARES SOBREPUESTAS CON APARIENCIA DE VIDRIO ESMERILADO y surcos longitudinales.

6) Mejor pronostico,

• Con que enfermedades se relaciona el carcinoma papilar de tiroides?1) Síndrome de GARDNER (poliposis adenomatosa familiar de colon) 2) Enfermedad de COWDEN (bocio familiar y hamartromas cutáneos)3) Síndrome de TURCOT (tumores cerebrales mas poliposis en intestino grueso)

• A que tipo de cáncer tiroideo son mas propensos los pacientes con enfermedad de Cowden?1) FOLICULAR, AD,2) Lo causa perdida de gen supresor de tumores. 3) Se caracteriza comúnmente por macrocefalia, HAMARTROMAS MÚLTIPLES, cáncer mamario o

PÓLIPOS INTESTINALES.

Page 453: ENARM  COMPENDIO

PATOLOGIA DE TIROIDES• Que distingue al carcinoma FOLICULAR tiroideo del papilar?

1) Que ESTA ENCAPSULADO,

2) INVADE VASOS SANGUÍNEOS.

3) Su VARIEDAD extensamente INVASIVA son las CÉLULAS DE HURTLE4) Tiene MAL PRONÓSTICO.5) Al extirparlo se hace SEGUMIENTO CON TIROGLOBULINA para descartar metástasis

• Que origen tiene el CARCINOMA MEDULAR DE TIROIDES y que sustancia produce?

1) Se origina de las CÉLULAS C PARAFOLICULARES,

2) Secretor de CALCITONINA.3) Secretor de ACTH4) Asociado a mutación en oncogen RET, debido a eso SE DEBEN ESTUDIAR TAMBIÉN A

FAMILIARES

• Cual es el carcinoma de tiroides con peor pronostico?1) ANAPLASICO con mortalidad del 95% en los primeros 6 meses.2) Crecimiento rápido3) Se han descrito supresión del gen supresor de tumores p53

Page 454: ENARM  COMPENDIO

PATOLOGIA TIROIDES

• Cual es el tratamiento de elección en carcinoma tiroideo?

1) Extirpación quirúrgica 2) Tratamiento medico con la INHIBICIÓN DE LA TSH

DANDO LEVOTIROXINA A DOSIS ALTAS3) Yodo radiactivo indicado en pacientes

POSTOPERADOS DE CA TIROIDEO4) Radioterapia externa: Solo en pacientes con

EXTIRPACIÓN QUIRÚRGICA INCOMPLETA5) Vigilancia clínica y por us

Page 455: ENARM  COMPENDIO

SX DE CUSHING• Cual es la principal causa del Sx Cushing y su cuadro clínico?1) Exógena por esteroides y 2da causa es por micro adenoma hipofisario2) OBESIDAD CENTRAL3) HAS por aumento del ANGIOTENSINOGENO HEPATICO4) Intolerancia a la glucosa por incremento de la GLUCONEOGENESIS inducida por

los glucocorticoides5) FACIES DE LUNA LLENA6) Trastornos menstruales7) HIRSUTISMO

• Como diagnosticas el sx de Cushing?1) Inicialmente se debe demostrar HIPERCOLESTEROLISMO2) Determinación de CORTISOL LIBRE URINARIO con sensibilidad de 90%

3) SUPRESIÓN CON DOSIS BAJAS DE DEXAMETASONA (prueba de NUGENT)4) Cuando un paciente tiene un cortisol urinario y serico a las 8:00 am > 1.8

después de 1 mg/dl de dexametasona a las 23 hrs de la noche previa, el paciente tiene síndrome de Cushing.

Page 456: ENARM  COMPENDIO

SX DE CUSHING• En cuanto al origen del síndrome de Cushing cuales son los estudios a realizar?1) Se determina la hormona ADRENOCORTICOTROPICA plasmática, LOS VALORES POR DEBAJO

DEL LIMITE DE DETECCIÓN CATALOGAN AL SÍNDROME COMO DE ORIGEN SUPRARRENAL.2) La RM es la técnica de imagen de elección en el síndrome de Cushing de ORIGEN HIPOFISARIO.3) La TAC es de elección para visualizar las GLÁNDULAS SUPRARRENALES.

• En que consiste la PRUEBA DE NUGENT en el diagnostico de sx de Cushing?• R = SUPRESIÓN DE ACTH con administrar 1mg de dexametasona a las 23 Hrs y con nueva

medición a las 8 AM.

• En que consiste el tratamiento medico del sx de Cushing?• R = KETOCONAZOL, pues INHIBE LA SÍNTESIS DE CORTISOL pero se emplea por periodos cortos

pues es hepatotoxico.

• En que consiste el tratamiento quirúrgico del sx de Cushing?1) CIRUGÍA HIPOFISARIA2) RADIOTERAPIA HIPOFISARIA que ha mostrado ser efectiva en los casos que no se pueda

realizar la CIRUGÍA TRANSESFENOIDAL

Page 457: ENARM  COMPENDIO

ENFERMEDAD DE ADDISON• Cual es la principal causa de la enfermedad de Addison/insuficiencia suprarrenal?1) 90% adrenalitis AUTOINMUNE, 2) En países subdesarrollados TB.

• Cual es el cuadro clínico y DHE de la enfermedad de Addison?

1) HIPERPIGMENTACION de la piel (debido a que la deficiencia de cortisol, provoca un amento compensatorio de la PROPIOMELANOCORTINA que es precursora de la ACTH y de la hormona estimulante de melanocitos)

2) Vomito3) Fatiga4) Hipoglucemia5) HIPOTENSIÓN

6) HIPONATREMIA7) HIPERKALEMIA.

• Cual es el desequilibrio hidroelectrolitico principal en la enfermedad de Addison?

• R = ACIDOSIS METABOLICA con HIPERCALCEMIA.

Page 458: ENARM  COMPENDIO

ENFERMEDAD DE ADDISON• Cuales son las manifestaciones clínicas de una CRISIS ADISONIANA o suprarrenal?

1) FIEBRE, 2) Deshidratación,

3) DOLOR ABDOMINAL4) HIPOTENSIÓN O CHOQUE HIPOVOLEMICO, 5) Urea elevada

6) HIPONATREMIA,

7) HIPERKALEMIA,

8) HIPERCALCEMIA y

9) ACIDOSIS METABÓLICA.

• Cual es el tratamiento de la crisis Adisoniana?• R = Administración inmediata de HIDROCORTISONA IV C-8

• Como realizas el diagnostico de enfermedad de Addison?

1) ESTANDAR DE ORO es la prueba de estrés con INSULINA IV la cual hace el diagnostico de adrenalitis secundaria, DONDE EL CORTISOL DEBE AUMENTAR, SINO ENTONCES SE HACE EL DIAGNOSTICO.

2) Medición de CORTISOL DURANTE LA MAÑANA y ADRENOCORTICOTROPINA con NIVELES BAJOS.3) ACTH IM y MEDIR CORTISOL A LOS 60 MIN, EL CUAL DEBE AUMENTAR. 4) En los pacientes con adrenalitis autoinmune se encuentran AUTOANTICUERPOS ANTI-21HIDROXILASA.

• Cual es el manejo de la enfermedad de Addison?• R = Hidrocortisona o PREDNISONA.

Page 459: ENARM  COMPENDIO

HIPERALDOSTERONISMO• Cual es la etiología del hiperaldosteronismo?1) Primario: Hipersecreción de aldosterona la cual NO SE SUPRIME EN RESPUESTA A LA EXPANSIÓN DE VOLUMEN.2) Secundario: Altos niveles de renina la cual se reprime en respuesta a la expansión de volumen.

• Cuales son los datos clínicos del hiperaldosteronismo?1) Puede cursar asintomático o 2) Cefalea3) HAS4) DEBILIDAD MUSCULAR5) PARESTESIAS6) Tetania 7) Parálisis

• Cuales son los datos de laboratorio encontrados en Hiperaldosteronismo primario?• R = HIPERNATREMIA, HIPOKALEMIA y ALCALOSIS METABÓLICA

• -A 30-year-old man is having symtoms of muscle weakness, fatigue, and headaches. On examination, his blood pressure is 180/100 mm Hg, pulse 80/min, JVP 4 cm, heart sounds normal, and lungs clear. His serum potassium level is 2.5 mEq/L and bicarbonate 30 mEq/L. The urine potassium is 40 mEq/L. Plasma renin is low and aldosterone is high.

• -Primary hyperaldosteronism is characterized by hypertension with high plasma aldosterone and low plasma rennin

Page 460: ENARM  COMPENDIO

HIPERALDOSTERONISMO• Como diagnosticas hiperaldosteronismo?• R = ELEVACIÓN DE ALDOSTERONA PLASMÁTICA POSTERIOR A LA INGESTIÓN DE

CLORURO DE SODIO por cada comida durante dos a tres días o un NIVEL DE ALDOSTERONA EN PLASMA ALTO DESPUÉS DE LA INFUSIÓN INTRAVENOSA DE SOLUCIÓN SALINA (1 lt x hr durante 4 hrs).

• Cual es el manejo del hiperaldosteronismo?1) CIRUGÍA A PACIENTES CON ALDOSTERONOMA con adrenalectomía laparoscópica2) En el caso de HIPERPLASIA SUPRARRENAL BILATERAL IDIOPÁTICA se debe recibir

TRATAMIENTO MEDICO CON ESPIRONOLACTONA3) En el caso de hiperaldosteronismo tratable con esteroides se utiliza la

DEXAMETASONA para mantener una supresión del eje hipotálamo hipófisis.

• Cuales son los datos que caracterizan al síndrome de Conn en hiperaldosteronismo?

• R = Hipotensión, hipokalemia, aumento de aldosterona y supresión secundaria de renina.

Page 461: ENARM  COMPENDIO

HIPOALDOSTERONISMO• Cual es la etiología del hipoaldosteronismo?1) Puede producirse por un estimulo insuficiente por parte de la renina

(HIPORRENINEMICO) “NEFROPATÍA DIABÉTICA”

2) Por un fallo primario en la secreción de aldosterona (HIPERRENINEMICO) “ENFERMEDAD DE ADDISON”

3) Por resistencia a la acción de la aldosterona (SEUDOHIPOALDOSTERONISMO).4) Algunos medicamentos pueden ocasionarlo como los AINES, heparina (suprime la

secreción de renina), IECAS.

• Cual es el cuadro clínico del hipoaldosteronismo?1) CALAMBRES2) Debilidad muscular

• Cuales son los datos de laboratorio del hipoaldosteronismo?

1) HIPERKALEMIA 2) HIPONATREMIA3) ACIDOSIS METABOLICA HIPERCLOREMICA4) Deshidratación5) Se debe hacer diagnostico diferencial de enfermedad de Addison

Page 462: ENARM  COMPENDIO

HIPOALDOSTERONISMO• Como se diagnostica el hipoaldosteronismo?1) Analíticamente se deben obtener los niveles plasmáticos de aldosterona,

cortisol y actividad de renina2) Si se sospecha de enfermedad de ADDISON SE DEBE REALIZAR LA PRUEBA DE

ESTIMULACIÓN CON HORMONA ACTH.3) Si existe aldosterona baja con actividad de renina (HIPORRENINEMICO),

debemos pensar en NEFROPATIA DIABETICA,4) Si obtenemos aldosterona baja y renina alta (HIPERRENINEMICO), debemos

sospechar ADISON o una forma grave de hiperplasia suprarrenal congénita.5) Si lo que nos reportan es ALDOSTERONA ELEVADA Y ACTIVIDAD DE RENINA

ALTA, debemos pensar en SEUDOHIPOALDOSTERONISMO.

• Cual es el manejo del hipoaldosteronismo1) INGESTIÓN LIBRE DE SAL como TRATAMIENTO MEDICO no farmacológico2) El tratamiento medico CON FÁRMACOS es con FLUDROCORTISONA.

Page 463: ENARM  COMPENDIO

ALTERACIONES POR DHE, Na y K • ENFERMEDAD DE ADDISON:

• Acidosis Metabolica con hipercalcemia• DHE: Hiponatremia – Hiperkalemia• Cual es el manejo de la enfermedad de Addison?• R = Hidrocortisona o PREDNISONA.

• HIPOALDOSTERONISMO:

• Acidosis Metabolica Hipercloremica• DHE: Hiponatremia – Hiperkalemia• Cual es el manejo del hipoaldosteronismo1) INGESTIÓN LIBRE DE SAL como TRATAMIENTO MEDICO no farmacológico2) El tratamiento medico CON FÁRMACOS es con FLUDROCORTISONA.

• HIPERALDOSTERONISMO:

• Alcalosis Metabolica• DHE: Hipernatremia - Hipokalemia• Cual es el manejo del hiperaldosteronismo?1) CIRUGÍA A PACIENTES CON ALDOSTERONOMA con adrenalectomía laparoscópica2) En el caso de HIPERPLASIA SUPRARRENAL BILATERAL IDIOPÁTICA se debe recibir TRATAMIENTO MEDICO

CON ESPIRONOLACTONA3) En el caso de hiperaldosteronismo tratable con esteroides se utiliza la DEXAMETASONA para

mantener una supresión del eje hipotálamo hipófisis.

Page 464: ENARM  COMPENDIO

HIPERCALCEMIA• Cuales son los eventos metabólicos implicados en la regulación del calcio?

1) En presencia de hipercalcemia se estimula la secreción de CALCITONINA, la cual INHIBE LA OSTEOLISIS y ESTIMULA LA CALCIURIA.

2) Cuando la hipercalcemia permanece, la CALCITONINA INHIBE LA REABSORCIÓN OSTEOCLASTICA, LA SECRECIÓN DE PARATHORMONA Y LA SÍNTESIS DE VITAMINA D.

• Cual es la etiología de la hipercalcemia?1) HIPERPARATIROIDISMO PRIMARIO: Puede deberse a ADENOMA ÚNICO o HIPERTROFIA GLANDULAR. Se

caracteriza por ELEVACIÓN DE LA PTH, HIPERCALCEMIA E HIPOFOSFATEMIA.2) Hiperparatiroidismo secundario: Puede deberse a IR, sx de mal absorción y raquitismo. 3) El adenoma y las neoplasias malignas causan 90% de las hipercalcemias.4) Es frecuente en el CÁNCER DE MAMA.

• Cual es el cuadro clínico de la hipercalcemia?1) Ca entre 10.5-12 g/dL es asintomático2) Niveles mayores forman TRIADA CLÁSICA DE ESTREÑIMIENTO, NAUSEA Y VOMITO.3) Fatiga, letargo, cefalea, sed, nausea, vomito, NEFROLITIASIS,4) ACORTAMIENTO DEL INTERVALO QT EN EL EKG.

• Cual es el manejo de la hipercalcemia?1) HIDRATACION2) FUROSEMIDE es eficaz para evitar la reabsorción tubular de calcio

• Cual es el fármaco que se indica en hipercalcemia crónica?• R = TIAZIDAS pues previenen cálculos renales.

Page 465: ENARM  COMPENDIO

HIPOCALCEMIA• Cual es la etiología de la hipocalcemia?1) Ca sérico menor 8.5. Previamente se debe confirmar que la albumina se encuentre en niveles séricos

normales, ya que el descenso de 1 g/dL de albumina se acompaña de un descenso de 0.8 mg/dL de calcio.2) La causa mas común de HIPOPARATIROIDISMO ES LA CIRUGÍA TIROIDEA Y LA SEGUNDA ES LA RADIACIÓN.3) La hipomagnesemia < 1 mg/dL puede producir hipocalcemia4) La hipermagnesmia reduce la secreción de PTH.5) La hiperfosfatemia causa hipercalcemia por formación de complejos con el calcio.6) En pacientes transfundidos se produce hipocalcemia ya que el citrato del paquete globular actúa como

paquete globular.7) EL SINDROME DE DIGEORGE se caracteriza por AUSENCIA DE GLÁNDULAS PARATORIDEAS, con DÉFICIT DE

PTH, ASOCIADO A DISPLASIA TIMICA, y malformaciones cardiacas por alteración en el desarrollo de LAS TERCERAS Y CUARTAS BOLSAS FARÍNGEAS.

8) Las formas adultas de hipoparatiroidismo se asocian a enfermedades autoinmunes como insuficiencia suprarrenal, anemia perniciosa e hipotiroidismo.

9) En la PANCREATITIS, el calcio se deposita en áreas con necrosis grasa.10) EL SINDROME DE HUESO HAMBRIENTO se presenta en paratiroidectomia.11) En enfermedades del APARATO DIGESTIVO se favorece el déficit de vitamina D por disminución en la

absorción.12) La DIFENILHIDANTOINA inhibe la absorción de calcio.

• Cual es el cuadro clínico de la hipocalcemia?1) HIPEREXITABILIDAD neuromuscular.2) Parestesias, HIPERREFLEXIA, ESPASMO CARPOPEDAL, irritavilidad, SIGNO DE CHEVOSTEK Y TROSSEAU.3) El EKG MUESTRA PROLONGACIÓN DEL ST.

Page 466: ENARM  COMPENDIO

OSTEOPOROSIS• Cual es el manejo de la osteoporosis?1) ALENDRONATO, RISENDRONATO.2) PTH a dosis bajas, al igual que los bifosfonatos han demostrado aumento en la masa

ósea,3) Terapia de reemplazo hormonal con estrógenos previene fracturas vertebrales.4) RALOXIFENO Y TAMOXIFENO ya que AUMENTAN LA DENSIDAD ÓSEA, DISMINUYEN

RIESGO DE FRACTURAS VERTEBRALES Y EL CÁNCER DE MAMA5) La TIBOLONA es una sustancia sintética con acción tejido específica que tiene

efecto ESTROGÉNICO, PROGESTACIONAL Y ANDROGÉNICO según el lugar de acción. Esta molécula mejora los síntomas vasomotores, el estado anímico, la libido, la atrofia urogenital y previene la osteoporosis postmenopáusica; no produce mastalgia ni modifica la densidad mamográfica. Se observan además efectos beneficiosos sobre algunos parámetros cardiovasculares y hemostáticos. EN EL ENDOMETRIO, LA TIBOLONA SE CONVIERTE EN SU METABOLITO PROGESTAGÉNICO/ANDROGÉNICO Y EN CONSECUENCIA NO PRODUCE ESTIMULACIÓN ENDOMETRIAL NI SE NECESITA AGREGAR UN PROGESTÁGENO. Esta ventaja mejora el cumplimiento de la terapia hormonal de reemplazo (THR) debido que no provoca sangrados por disrupción

Page 467: ENARM  COMPENDIO

DENSITOMETRIA OSEA

Page 468: ENARM  COMPENDIO

OSTEOMALACIA• Cual es la etiología de la osteomalacia?• R = Déficit de VITAMINA D EN ADULTOS

• Cual es el cuadro clínico de la osteomalacia?1. DEBILIDAD MUSCULAR PROXIMAL, asociada con desgaste muscular, hipotonía y dificultad para la movilización.2. EL OSTEOCOPO (dolor óseo), es mas notorio en LA REGION LUMBAR, PELVIS Y EXTREMIDADES PELVICAS DONDE

SE PRESENTAN FRACTURAS.3. Las fracturas suceden con un mínimo de traumatismo.4. Los síntomas deben incluir: DOLOR OSEO, DEFORMIDADES DEL ESQUELETO Y/O CRANEO, piernas arqueadas o

rodillas malformadas, PECHO EN PALOMA, Deformaciones dentales.

• Como se diagnostica la osteomalacia?1. MARCLAJE CON TETRACICLINAS. Este medicamento se deposita en forma de bandas en el sitio de

mineralización y al ser fluorescentes pueden ser visualizados al microscopio. La tasa de crecimiento del esqueleto puede estimarse en las biopsias de las crestas iliacas al medir la distancia entre las bandas de tetraciclina. Si disminuye la distancia entre las bandas se hace el diagnostico.

2. EL HALLAZGO RADIOGRÁFICO CARACTERÍSTICO ES LA REDUCCIÓN EN LA DENSIDAD ÓSEA CON ADELGAZAMIENTO DE LA CORTEZA.

3. Laboratorio: Los pacientes con déficit de vitamina D se presentan con HIPOFOSFATEMIA, CALCIO SÉRICO BAJO E HIPOPARATIROIDISMO SECUNDARIO.

• Cual es el manejo de la osteomalacia?• R = Calcitriol o CALCIO + VITAMINA D en combinación.

Page 469: ENARM  COMPENDIO

DIABETES INSIPIDA CENTRAL• A que se debe la diabetes insípida central primaria y secundaria?

1) Primaria : Ac vs ARGININA-VASOPRESINA.2) LESIÓN DEL HIPOTÁLAMO, del tallo hipofisario o por cualquier CAUSA INFECCIOSA,

TRAUMÁTICA etc..

• Cuales son las causas de DI central y cuales son los hallazgos en la TAC?• R = DI IS USUALLY CAUSED BY DESTRUCTION, OR AGENESIS, OF THE POSTERIOR

PITUITARY, its normal signaling is lost. PITUITARY DI CAN ALSO RESULT FROM TRAUMA, TUMORS (BOTH PRIMARY AND SECONDARY), granulomas, infections, inflammatory diseases, chemical toxins, congenital malformations, and genetic disorders. Depending on the cause, the MRI may demonstrate other associated findings.

• Que causa diabetes insípida en el ultimo trimestre de embarazo-puerperio?• R = Es inducido por VASOPRESINA en el ultimo trimestre de embarazo, relacionada

frecuentemente con oligohidramnios, preeclampsia o disfunción hepática. RESPONDE A DESMOPRESINA

Page 470: ENARM  COMPENDIO

DIABETES INSIPIDA NEFROGENICA• Como diagnosticas DM insípida?1) JUICIO CLÍNICO, no hay prueba de laboratorio contundente. 2) RECOLECCIÓN DE ORINA DE 24 HRS MIDIENDO VOLUMEN <2LTS/24 HRS SIN HIPERNATREMIA. 3) Desmopresina: MIDIENDO VOLUMEN URINARIO 12 HRS ANTES Y 12 HRS DESPUÉS, EN EL

CUAL LOS PACIENTES CON DIABETES INSÍPIDA CENTRAL HABRÁ DISMINUCIÓN DE SED Y POLIURIA,

4) Realizándose IRM analizando hipófisis e hipotálamo, se observa saco hipofisario engrosado: Células de langerhans, sarcoidosis, hipofisitis linfocitica, Histiocitosis (proliferación de macrófagos en diferentes aparatos y sistemas).

5) Desmopresina en DM nefrogenica valorando su concentración con restricción de líquidos aumentando sus valores.

• Cual es el manejo de la DM insípida nefrogenica o central?

1) CENTRAL: DESMOPRESINA tmb se utiliza en DM INSÍPIDA RELACIONADA CON EMBARAZO o puerperio, evitar factores de agravamiento como administración de glucocorticoides.

2) NEFROGENICA: HIDROCLOROTIAZIDA con suplemento de potasio, INDOMETACINA en fase aguda.

Page 471: ENARM  COMPENDIO

ACROMEGALIA• Cual es la causa principal de acromegalia?• R = ADENOMA HIPOFISIARIO, también por secreción ectópica de GnRH o GH por tumores carcinoides hipotalámicos, bronquiales o

pancreáticos.

• Que características forman parte del SÍNDROME DE CARNEY?• R = MIXOMA AURICULAR, NEUROMA DEL ACÚSTICO Y PIGMENTACIÓN PUNTEADA DE LA PIEL

• Cuales son los signos y síntomas de la acromegalia o gigantismo?1) ESTATURA ELEVADA, gigantismo antes del cierre hiposifiario. 2) Acromegalia significa crecimiento de extremidades. 3) SÍNDROME DEL TÚNEL CARPIANO. 4) RASGOS FACIALES TOSCOS, PROGNATISMO CON MALA OCLUSIÓN.

5) HAS 50%. DM 30%

• Cuales son los lab para dx acromegalia o gigantismo?1) AYUNO. > IGF-1 5 VECES DE LO NORMAL. 2) Prolactina aumentada algunas veces. 3) Glucosa: la DM ES COMÚN. 4) Calcio: Para descartar hiperparatiroidismo. 5) Fosforo: Se encuentra aumentada. 6) T4 y TSH: HIPOTIROIDISMO COMÚN. 7) Se recomienda ADMINISTRAR UNA SOLUCIÓN DE DEXTROSA MIDIENDO LA GH 1 HR DESPUÉS, SI LOS VALORES ESTÁN < O NORMALES AL

IGUAL QUE LA IGF-1, YA QUE ESTA HORMONA RESPONDE CON DISMINUCIÓN A LA INGESTA DE GLUCOSA.

• Que medicamentos se utilizan en caso de no remitir > GH o IGF-1 con adenectomía transesfenoidal hipofisiaria?1) ANÁLOGOS DE LA SOMATOSTATINA COMO OCTREOTIDE para conservar disminución del tamaño del adenoma. GH sérica 1 mg y 2.5 mg,

pero no responde bien si previos al tratamiento la GH > 20 ng/ml. 2) PEGVISOMANT es un antagonista del receptor de la GH bloqueando sus efectos, vía SC, pero no reduce el tamaño de los adenomas.3) RADIOCIRUGÍA: en los pacientes no responden al tratamiento quirúrgico o medico, no se realiza si hay extensión suprecelar por que puede

dañar el quiasma óptico.

Page 472: ENARM  COMPENDIO

NEM I• Cuales son las características de la NEM I O SÍNDROME DE WERMER?1) Se caracteriza por neoplasias en PARATIROIDES, PÁNCREAS y ADENOHIPOFISIS.

2) Se hereda de forma AD en el CROMOSOMA 11

• Cual es la clínica del NEM I o Sindrome de Wermer?1) HIPERPARATIROIDISMO PRIMARIO: Que puede manifestarse con MIALGIAS, DEBILIDAD, HIPERCALCEMIA,

ALTERACIONES MENTALES, NEFROLITIASIS Y ANORMALIDADES ÓSEAS. La hipercalcemia puede provocar aumento de la secreción de gastrina y un Sx Zollinger-Ellison secundario. EL TRATAMIENTO CONSISTE EN LA EXTIRPACIÓN QUIRÚRGICA DE LAS PARATIROIDES.

2) TUMORES DE LAS CÉLULAS DE LOS ISLOTES PANCREÁTICOS: Los GASTRINOMAS SON LA CAUSA MAS IMPORTANTE de morbimortalidad y la mayoría son multiples. Los insulinomas son generalmente benignos y únicos.

3) NEOPLASIA DE LA ADENOHIPOFISIS: La neoplasia MAS FRECUENTE ES EL PROLACTINOMA seguida del tumor secretor de hormona del crecimiento que causa acromegalia, pueden presentarse como enfermedad de Cushing secundaria a adenoma hipofisario.

• Como realizas el diagnostico de NEM I?1) Con 2 de las 3 neoplasias características del síndrome2) El MEN I familiar se define como la presencia de un paciente afectado con esta enfermedad y un familiar de

primer grado que presente al menos una de las tres neoplasias características

• Cual es el manejo del NEM I?1) CIRUGÍA2) PARA CADA MANIFESTACIÓN de sobre producción hormonal se da el medicamento individual

Page 473: ENARM  COMPENDIO

MEN II• Cual es la etiología cromosómica del MEN II?

• R = Se hereda de forma AD, en el CROMOSOMA 10.

• Cual es la clasificación del MEN II? MEN II A o Síndrome de Sipple: Con la presencia de cáncer medular de tiroides,

FEOCROMOCITOMA e hiperparatiroidismo primario. MEN II B: Con la presencia de cáncer medular de tiroides, feocromocitoma,

NEUROMAS MUCOSOS, ganglioneuromatosis intestinal y HABITO MARFANOIDE.

• Como diagnosticas el MEN II?1) 2 a 3 tumores de los antes mencionados2) La mutación del protoncogen RET esta indicada en todos los familiares de primer

grado.3) EL CARCINOMA MEDULAR DE TIROIDES SUELE SER LA PRINCIPAL CAUSA DE

MUERTE.

Page 474: ENARM  COMPENDIO

GASTRINOMA• Cuales son los tumores de las células de los islotes pancreáticos?

• R = Gastrinoma, insulinoma, glucagonoma, vipoma y somatostinoma. GIGVS

• Cual es el cuadro clínico del gastrinoma?1) Enfermedad ulcerosa con dolor abdominal2) Diarrea3) ERGE.4) Se debe sospechar en ULCERAS PÉPTICAS MÚLTIPLES.

• Como diagnosticas el gastrinoma?1) Los niveles séricos de GASTRINA EN AYUNO MAYORES DE 1000 Y PH < 2.5 EN LA SECRECIÓN GÁSTRICA

HACEN EL DIAGNOSTICO DE GASTRINOMA.2) Se debe SUSPENDER LA ADMINISTRACIÓN DE ANTAGONISTAS DE H2 UNA SEMANA ANTES Y DE

OMEPRAZOL.3) En paciente con niveles séricos menores a 1000 y PH < 2.5 se debe pensar en H. Pilory4) La aclorhidria por gastritis atrófica o anemia perniciosa es una de las causas mas frecuentes de

hipergastrinemia.

• Cual es el manejo del gastrinoma?1) Para la hipersecreción acida OMEPRAZOL2) Gastrinoma RESECCIÓN QUIRÚRGICA PARA EVITAR METÁSTASIS

Page 475: ENARM  COMPENDIO

INSULINOMA• Cual es el cuadro clínico del insulinoma?

1) SECUNDARIOS AL EFECTO HIPOGLUCEMICO como cefalea, mareo, debilidad, confusión, desorientación y coma.

2) Síntomas secundarios al aumento de catecolaminas circulantes provocado por la hipoglucemia como sudación, temblor y palpitación.

3) La mayoría de las crisis se relaciona con el ayuno.

• Como realizas el diagnostico de insulinoma?

1) Se basa en la TRIADA DE WIPPLE la que consiste en síntomas de neuroglucopenia, niveles de glucosa en sangre por debajo de 45 y mejoría inmediata de los síntomas después de la administración de glucosa

2) Si se sospecha de insulinoma se debe realizar una PRUEBA DE AYUNO DURANTE 72 HRS, considerado como el método diagnostico de elección, donde se miden los niveles de glucosa, péptido C e insulina cada 6 hrs y siempre que exista clínica de hipoglucemia la prueba se suspende si aparece una glucemia inferior a 45 o si se presentan síntomas neuroglucopenicos. En sujetos normales la insulina debe disminuir en presencia de hipoglucemia. Los criterios diagnósticos de insulinoma incluyen insulina plasmática mayor de 6 cuando la glucemia es menor de 40, proinsulina > .2, péptido C > 200 y cociente de insulina/glucosa > .3.

3) Existen AINTICUERPOS ANTIINSULINA Y NIVELES ELEVEADOS DE PROINSULINA.

• Cual es el manejo del insulinoma?1) Para las crisis de hipoglucemia se realizan comidas frecuentes y de escasa cantidad2) En el caso de hipoglucemia se administra 50 ml de dextrosa al 50%, si no hay respuesta dentro de los primeros 15

minutos, la dosis puede repetirse, en caso de que no se pueda canalizar al paciente se administra 1 mg de guagón por vía IV, IM o subcutánea.

3) El DIAZOXIDO inhibe la secreción de insulina, pero provoca retención de líquidos e hirsutismo con frecuencia.4) La resección completa del tumor cura a la mayoría de los pacientes. Cuando no se logra localizar el tumor se realiza

una pancreatectomia distal gradual hasta localizarlo dejando por lo menos 20-30% del páncreas hasta localizarlo.

5) EN EL CASO DE TUMOR MALIGNO CON METÁSTASIS O TUMOR IRRESECABLE SE RECOMIENDA EL TRATAMIENTO A LARGO PLAZO CON DIAZOXIDO, SI NO RESPONDE SE INICIA QUIMIOEMBOLIZACION.

Page 476: ENARM  COMPENDIO

VIPOMAS• Que son los vipomas?• R = Son tumores derivados de las CÉLULAS DE LOS ISLOTES DEL PÁNCREAS que producen grandes cantidades de

PEPTIDO INTESTINAL VASOACTIVO, el cual AUMENTA LA CONTRACCIÓN DEL MUSCULO LISO, INHIBE LA SECRESION GÁSTRICA Y TIENE EFECTOS VASODILATADORES.

• Cual es el cuadro clínico asociado a vipomas?

1) El síntoma predominante es una DIARREA ACUOSA, que produce deshidratación, hipokalemia y debilidad.

2) La DIARREA ES SECRETORA, persiste durante el ayuno y NO RESPONDE A ANTIDIARREICOS.3) Acidosis metabólica

• Como realizas el diagnostico del vipoma?1) Presencia de DIARREA SECRETORA2) Niveles elevado de PÉPTIDO INTESTINAL VASOACTIVO3) TUMORACION pancreática

• Como se maneja el vipoma?

1) Corregir la deshidratación y desequilibrio hidroelectrolitico, en especial de la HIPOKALEMIA.2) Resección del tumor y en presencia de metástasis se reduce primero el tumor.3) Si no se encuentra el tumor se busca en las glándulas suprarrenales y las cadenas simpáticas, en caso de no

identificarse en ninguno de estos sitios se realiza resección del 80% del tumor.4) En el caso de tumor maligno inoperable o presencia de metástasis se recomienda el uso del análogo de la

somatostatina de acción prolongada, también puede utilizarse la embolizacion de la arteria hepática.

Page 477: ENARM  COMPENDIO

GLUCAGONOMAS• Que son los glucagonomas?

1) Son tumores derivados de las CELULAS ALFA DE LOS ISLOTES DEL PANCREAS los cuales producen una gran cantidad de la HORMONA GLUCAGON y puede secretar otros péptidos (polipeptido pancreático, gastrina y somatostatina).

2) Es mas frecuente en mujeres PERI O POSTMENOPAUSICAS

• Cual es el cuadro clínico de los glucagonomas?1) DERMATITIS NECROLITICA MIGRATORIA es característica de los glucagonomas. Inicia como una zona

eritematosa y escamosa en la ingle y glúteo, se extiende al perineo y extremidades inferiores, en ocasiones hay lesiones peri orales, posteriormente se hace ampolloso y deja erosiones cuando las ampollas se rompen.

2) Intolerancia a la glucosa o DIABETES

3) Anemia

4) QUEILITIS Y ESTOMATITIS

5) Diarrea

6) Hipercolesterolemia

• Como diagnosticas los glucagonomas?1) Con el cuadro clínico y se confirma al encontrar niveles plasmáticos de GLUCAGON EN AYUNO > 1000 (NORMAL

150) Y UNA TUMORACIÓN PANCREÁTICA.

2) NO SE SUPRIME LA LIBERACIÓN DEL GLUCAGON CON LA ADMINISTRACIÓN DE GLUCOSA Y TAMPOCO SE ESTIMULA CON LA ADMINISTRACIÓN DE ARGININA.

• Cual es el manejo del glucagonoma?1) Debido a la presencia de METASTASIS EN LA MAYORIA DE LOS PACIENTES, no es posible una resección

quirúrgica curativa, pero en estos casos se extirpa el tumor primario y se reduce el volumen de las metástasis.

2) Los SUPLEMENTOS DE ZINC mejoran la dermatitis

3) Los ANALOGOS DE LA SOMATOSTATINA normalizan los valores de glucagon y aminoácidos séricos, mejoran la dermatitis y la diarrea, promueven el aumento de peso, pero NO corrigen la intolerancia a la glucosa.

4) La quimioterapia NO es útil pero cuando se decide utilizarla, la combinación que ha demostrado mas eficacia es la de ESTREPTOZOCINA +DACARBACINA.

5) EL 50% DE LAS MUERTES ES CAUSADO POR EVENTOS TROMBOEMBOLICOS.

Page 478: ENARM  COMPENDIO

SOMATOSTINOMAS• Que son los somatostatinomas?

1) Son tumores derivados de las CELULAS DELTA DEL PANCREAS productores de somatostatina, al igual que los VIPomas y glucagonomas

2) 70% SON MALIGNOS3) La somatostatina es un inhibidor potente de la secreción gástrica y pancreática

• Cual es el cuadro clínico de los somatostatinomas?1) Es causado por la inhibición de la secreción tanto exocrina como endocrina del páncreas

2) LA TRIADA CLASICA CONSISTE EN DIABETES, DIARREA-ESTEATORREA Y COLELITIASIS.

• Como diagnosticas los somatostatinomas?• R = La mayoría se localiza incidentalmente al realizar cirugías abdominales o endoscopias. EL

diagnostico se realiza al demostrar la somatostatina elevada en plasma.

• Cual es el manejo de los somatostatinamas?1) ANÁLOGOS DE LA SOMATOSTATINA con el fin de inhibir la activación de los receptores por parte

de la somatostatina secretada por el tumor. 2) EXTIRPACIÓN QUIRÚRGICA si se localiza el tumor3) Para enfermedad metastasica se usa QUIMIOTERAPIA

Page 479: ENARM  COMPENDIO

FEOCROMOCITOMA• Cual es la etiología del feocromocitoma?1) Tumores de origen NEUROECTODERMICO desarrollado a partir de las CÉLULAS CROMAFINES DEL SNS (ganglios simpáticos y

medula suprarrenal) productores de CATECOLAMINAS y la mayoría de ellos libera epinefrina y norepinefrina.2) Los feocromocitomas ASOCIADOS A NEM SECRETAN SOLO EPINEFRINA.

• Cual es la clasificación y localización mas común de los feocromocitomas?1) Feocromocitomas si provienen de las células cromafines y paragangliomas si son de origen extra renal.2) La localización mas frecuente es la adrenal 85%, siempre es unilateral, en caso de ser bilateral debe sospecharse de NEM

• Cual es el cuadro clínico del feocromocitoma?1) HAS 90%2) Crisis adrenal caracterizada por triada clásica de: CEFALEA, PALPITACIONES Y SUDORACIÓN.

• Cuales son los métodos diagnósticos de feocromocitoma?1) Demostración de niveles elevados de catecolaminas o sus metabolitos en sangre o en orina, lo que establece el diagnostico.2) Se determina el nivel de catecolamina libre o no metabolizadas (epinefrina y norepinefrina) o sus metabolitos

( metanefrina, normetanefrina y ACIDO VANILMANDELICO) en orina de 24 hrs.

• Cuales son los métodos de imagen para dx de feocromocitoma?1) TAC2) Gamagrafia para localizar paragangliomas

• Cual es el manejo del feocromocitoma?1) Cirugía, pero antes de realizarla es necesario realizar un bloqueo alfadrenergico, generalmente se utiliza

FENOXIBENZAMINA durante 10-14d previos a la cirugía.2) Durante la anestesia o manipulación del tumor SUELE APARECER HIPERTENSIÓN Y ARRITMIAS, LA PRIMERA SE TRATA CON

FENTOLAMINA Y LA SEGUNDA CON PROPANOLOL.

Page 480: ENARM  COMPENDIO

TUMOR CARCINOIDE• Cual es el cuadro clínico producido por el tumor carcinoide?• R = Producen síntomas inespecíficos, vagos, relacionados con el órgano afectado tales como:1) Carcinoide apendicular: El la LOCALIZACIÓN MAS FRECUENTE. Si el tumor mide mas de 2 cm se debe realizar una

hemicolectomia derecha, si mide entre 1-2 cm e invade la serosa se debe realizar una hemicolectomia derecha y linfadenectomia regional. LA APENDICETOMÍA SE CONSIDERA CURATIVA CUANDO MIDEN MENOS DE 1 CM.

2) Carcinoide rectal: Estos secretan mas glucagon mas que serotonina. Puede manifestarse como SANGRADO RECTAL, DOLOR O CONSTIPACION. Los tumores mayores de > 2 cm se tratan con una resección anterior baja o una abdominoperineal.

3) Carcinoide del intestino delgado: La mayoría se encuentra el ILEON. Pueden provocar desde molestias abdominales leves hasta dolor abdominal intermitente causado por la obstrucción intestinal, la cual puede ser secundaria a la fibrosis que producen estos tumores o con menor frecuencia se manifiestan con diarrea, hemorragia por ulceración de la mucosa o intususcepción. SON LA CAUSA MAS FRECEUNTE DE SX CARCINOIDE.

4) Carcinoides pulmonares y bronquiales: Neumonías recurrentes, tos hemoptisis, dolor pleurítico. La resección en cuna o segmentaria se recomienda para los tumores localizados en caso de los carcinoides atípicos se utilizan resecciones mas amplias.

5) Carcinoides gástricos: Se originan de las células enterocromafines de la mucosa gástrica, se manifiestan como anemia o dolor abdominal. EL tipo 1 y mas frecuente en mujeres se asocia a hipergastrinemia y gastritis atrófica.

6) Carcinoides del colon: La mayoría en COLON DERECHO, siendo el CIEGO el sitio mas afectado. Pueden producir dolor, anorexia y perdida de peso.

• Como diagnosticas los tumores carcinoides?1) Cuadro clínico2) Concentración de serotonina o de sus metabolitos en orina3) Imagen4) Se confirma por biopsia

5) Medición del ACIDO 5-HIDROXINDOLACETICO en una muestra de ORINA de 24 hrs, el cual si es mayor de 10

mg indica EXCESO DE SEROTONINA.6) En SANGRE se miden los niveles de CROMOGRANINA A, glucoproteina producida por las células derivadas de la cresta

neural y secretada en los tumores neuroendocrinos, relacionada directamente con el volumen tumoral

Page 481: ENARM  COMPENDIO

HIPERPLASIA SUPRARRENAL CONGÉNITA• Cual es la etiología del hiperandrogenismo suprarrenal?• R = Déficit de 21-hidroxilasa, el gen responsable se denomina CYP21B localizado en el brazo corto del

cromosoma 6

• Cual es el cuadro clínico del hiperandrogenismo renal por déficit de 21-hidroxilasa?A. FORMA CLÁSICA (PERDEDORA DE SAL O VIRILIZANTE SIMPLE): Implica la existencia de

hiperandrogenismo in útero que condiciona la aparición de macrogenitosomia en el varón y un grado variable de virilización en la mujer. En la forma perdedora de sal existe un DÉFICIT TANTO DE CORTISOL COMO DE ALDOSTERONA y se manifiesta en ambos sexos por perdida salina en la etapa neonatal. En la virilizante simple no esta afectada la síntesis de aldosterona por lo que se mantiene la homeostasis del sodio.

B. FORMA NO CLÁSICA: Existe HIPERANDROGENISMO DE APARICIÓN POSNATAL. Los síntomas mas frecuentes durante la infancia son pubarquia prematura, piel grasa con acné, aceleración del crecimiento y maduración ósea y en las niñas aparece hipertrofia de clítoris, en la adolescencia y la edad adulta las mujeres pueden presentar trastornos menstruales, hirsutismo, acné, calvicie, ovarios poliquísticos e infertilidad, los varones afectados pueden cursar con oligoespermia, acné e infertilidad.

• Como diagnosticas el déficit de la enzima 21-hidroxilasa?• R = Demostración de VALORES ELEVADOS DE 17-HIDROXIPROGESTERONA

• Cual es el manejo del déficit de enzima 21-hidroxilasa?• R = GLUCOCORTICOIDES y en niños recién nacidos es FLUOROHIDROCORTISONA y en adultos es a base

de PREDNISONA.

Page 482: ENARM  COMPENDIO

ENFERMEDAD DE PAGET

• A 63-year-old asymptomatic woman is investigated for a HIGH ALKALINE PHOSPHATASE (ALP) level. X-rays of the pelvis show multiple porotic and sclerotic lesions with characteristic whorls of trabeculation. HER EXCRETION OF URINARY HYDROXYPROLINE IS ALSO ELEVATED. Which of the following is the most likely diagnosis?

• R = Enfermedad de Paget del Hueso

• The elevated ALP AND HYDROXYPROLINE are diagnostic for Paget’s disease. The bony lesions are blastic and the sacrum and pelvis are most frequently involved, followed closely by the tibia and femur. Hypercalcemia can complícate immobilization. The etiology is unknown, but a viral agent has been postulated. Symptoms may be absent or severe (pain, deformity). In metastatic cancers of most types the lesion are lytic, and the other metabolic abnormalities do not have an elevation in hydroxyproline.

Page 483: ENARM  COMPENDIO

Insulina e Hipoglucemiantes orales.Hipoglucemiantes orales.

• Sulfonilureas:– 1a generación: tolbutamida (TBT), clopropramida, tolazamida,

acetohexamida.– 2a generación: gliburida, glipizida, glibenclamida, glicazida. Son más

potentes.

Page 484: ENARM  COMPENDIO

Insulina e Hipoglucemiantes orales.Hipoglucemiantes orales.

• Sulfonilureas.– Mecanismos:

• Estimulan liberación de insulina, disminuyen eliminación hepática, aumentan somatostatina (disminuye glucagon).

• Regulación a la baja de receptores: al susp pueden tener nuevam respuesta.

• Bloq canal K sensible a ATP: despolarización: aumenta Ca.

Page 485: ENARM  COMPENDIO

Insulina e Hipoglucemiantes orales.Hipoglucemiantes orales.

• Sulfonilureas.– Farmacodinamia.

• VO mejor 30´preprandial.• Unión PP 90-99% (alb).• T 1/2 tolazamida 4-7hrs, clorpropramida 24-48hrs. 2da generación 1.5-

5hrs (efecto disminuye glucosa x 12-24hrs).• Metab hígado. Excreción renal. Cuidado en insuf hepática y renal.

Page 486: ENARM  COMPENDIO

Insulina e Hipoglucemiantes orales.Hipoglucemiantes orales.

• Sulfonilureas.– Efectos colaterales.

• 1a generación:40%. Menos frec 2da generación.• Hipoglucemia.• Medicamentos: desplazan de PP: sulfonamidas, clofibrato, dicumarol,

salicilatos, fenilbutazona. El alcohol:aumenta su acción..• Náusea, vómito, ictericia colestásica, agranulocitosis, A aplásica, A

hemolítica, alergia.• Clorpropramida: dosulfiram 10-15%, hiponatremia 5% x aumentar ADH en

túbulo colector.• TBT: aumenta 2v riesgo de muerte cardiovascular.

Page 487: ENARM  COMPENDIO

Insulina e Hipoglucemiantes orales.Hipoglucemiantes orales.

• Sulfonilureas.– Usos.

• DMIND.• Contraindicada en DMID, embarazo, lactancia, insuf renal y hepática

significativa.

Page 488: ENARM  COMPENDIO

Insulina e Hipoglucemiantes orales.Hipoglucemiantes orales.

• Biguanidas.– Metformin y fenformin.– Fenformin asociado a ác láctica.– Metformin T1/2 1.3-4.5 hrs. Máx 3g/d. No produce hipoglucemia, no

libera insulina.– Aumenta acción perif insulina, inhibe gluconeogénesis, disminuye

absorción gluc intestinal.– VO.– No en insuf renal o hepática: ác láctica, hipoxemia cr, insuf cardiaca.– Efectos colat: 20%. Diarrea, náusea, malestar abd, sabor metálico,

anorexia, disminuye absorción vit B12 y folatos.

Page 489: ENARM  COMPENDIO

Insulina e Hipoglucemiantes orales.Hipoglucemiantes orales.

• Otros.– Ciglitazona, pioglitazona, troglitazona: thiazolidinedionas. No dan

hipoglucemia. Disminuyen resistencia a insulina x aumentar transportadores glucosa.

– Inhib alfa glucosidasa: acarbosa. La inhiben en el borde en cepillo: inhiben absoción CHOS.

– Efectos colaterales: malaabsorción, flatulencia, distensión abd.

Page 490: ENARM  COMPENDIO

NEUMOLOGIA

ENARM

Page 491: ENARM  COMPENDIO

REACCIONES DE HIPERSENSIBILIDADTIPO NOMBRE ALTERNATIVO ALTERACIONES NOMBRADAS

FRECUENTEMENTE MEDIADORES

1 ALERGIA (IMMEDIATA)ATOPIA

ANAFILAXIA

ASMA

IGE

2 ANTICUERPO DEPENDIENTE

ANEMIA HEMOLÍTICA AUTOINMUNE

TROMBOCITOPENIA

ERITROBLASTOSIS FETALICOS

SÍNDROME DE GOODPASTURE

IGM O IGG

(COMPLEMENTO)

3 ENFERMEDAD DE COMPLEJO INMUNE

ENFERMEDAD DEL SUERO

REACCIÓN DE ARTHUS

LUPUS ERITEMATOSO SISTÉMICO

IGG

(COMPLEMENTO)

4 CITOTÓXICA HIPERSENSIBILIDAD RETARDADA

DERMATITIS DE CONTACTO

TEST DE MANTOUX

RECHAZO CRÓNICO DE ÓRGANO TRASPLANTADO

ESCLEROSIS MÚLTIPLE4

CÉLULA T

Page 492: ENARM  COMPENDIO

SARCOIDOSIS PULMONAR• En que consiste la sarcoidiosis?• R = Enfermedad sistémica de ETIOLOGÍA DESCONOCIDA, caracterizada en 90% por INFLAMACIÓN

GRANULOMATOSA PULMONAR

• Que datos clínicos o de laboratorio son mas frecuentes a encontrar en sarcoidiosis?

1) Clínicos: Tos crónica, ERITEMA NODOSO, UVEITIS, parotiditis, hepato-esplenomegalia + LINFADENOPATIA.

2) Laboratorio: AUMENTO DE ECA 40-80%, hipercalciuria. 3) ANERGIA a pruebas cutáneas 70%. 4) LAVADO BRONCOALVEOLAR con alto contenido de CD4 Y CD8.

• Cuales son los datos radiológicos que te da sarcoidosis?1) AUMENTO DE TAMAÑO DE GANGLIOS LINFÁTICOS HILIARES BILATERALES en pacientes asintomáticos con

exploración física normal y exámenes de rutina normales se presenta en el 95% de pacientes con sarcoidosis pulmonar

2) Adenopatia hiliar bilateral/ afección parenquimatosa bajo la forma de infiltrado reticular difuso.

• Cuales son los signos espirometricos mas frecuentes en sarcoidosis?• R = CAMBIOS RESTRICTIVOS con disminución de los volúmenes pulmonares y capacidad de difusión.

• Que te indica la biopsia de un paciente con sarcoidosis?

• R = GRANULOMAS NO CASEIFICANTES PATOGNOMONICOS

• Cual es el tratamiento de sarcoidosis?• R = ESTEROIDES E INMUNOSUPRESORES

Page 493: ENARM  COMPENDIO

NEUMONIA NOSOCOMIAL• Como se define la neumonía nosocomial (NNC)?• R = Se define la neumonía adquirida en hospital como aquella que se desarrolla después de una estancia

intrahospitalaria de 48 hrs.

• Cual es la etiología de la NNC?• R = Los patógenos mas comunes incluyen BACILOS GRAMNEGATIVOS, como PSEUDOMONAS AUROGINOSA, E. coli,

Klebsiella y Acinetobacter.

• Cual es la sospecha diagnostica de la NNC?1) Px presenta infiltrados nuevos o progresivos2) Hallazgos clínicos como fiebre, leucocitosis o leucopenia, esputo purulento.3) Los hallazgos radiológicos pueden variar DESDE INFILTRADOS AÉREOS EN FORMA DE PARCHE HASTA

CONSOLIDACIONES LOBARES CON BRONCOGRAMA AÉREO O INFILTRADOS DIFUSOS ALVEOLARES O INTERSTICIALES.

• Cual es el manejo de la NNC?1) DE INICIO EMPÍRICO2) Esquema antibiótico de espectro limitado: px con < 5d de hospitalización con Ceftriaxona, levofloxacina, Ampicilina y

Ertrapenem3) Esquema antibiótico de espectro amplio: px con >5d de hospitalización Cefalosporinas antipseudomonica como

CEFEPIME, Fluoroquinolona antipseudomonica como CIPROFLOXACINO, Aminoglucosido como AMIKACINA.4) La terapia es normalmente de 14 a 21d.

Page 494: ENARM  COMPENDIO
Page 495: ENARM  COMPENDIO
Page 496: ENARM  COMPENDIO

DERRAME PLEURAL• Cual es la diferencia entre derrame pleural de tipo exudativo y de tipo

trasudativo?

• TRASUDADO: se define como aquel derrame pleural ORIGINADO POR FACTORES SISTÉMICOS que producen alteraciones en la formación o reabsorción del líquido pleural. La principal causa es ICC y cirrosis.

• EXUDADO: Es originado por varios factores locales que perturban la formación y absorción de líquido pleural. SON DEBIDOS PRINCIPALMENTE A NEUMONÍA BACTERIANA, NEOPLASIAS MALIGNAS, INFECCIONES VIRALES Y EMBOLIA PULMONAR.

Page 497: ENARM  COMPENDIO

DERRAME PLEURAL• Cuales son las características de laboratorio del EXUDADO pleural?1) Relación entre proteínas del liquido pleural y proteína serica >.5, 2) Relación entre DHL de liquido pleural y DHL sérica >.6, 3) DHL >2/3 partes de la DHL sérica.

• Cuales son los datos de laboratorio de un TRASUDADO pleural?• R = Glucosa igual a glucosa sérica, PH 7.40-7.55, < 1000 leucocitos

• Que características tiene el derrame pleural reumatoide?• R = Liquido turbio VERDE - AMARILLENTO, FR aumentado, CRISTALES DE

COLESTEROL.

• Que fármacos se utilizan en la pleurodiesis?• R = DOXICICLINA y talco

Page 498: ENARM  COMPENDIO
Page 499: ENARM  COMPENDIO

PATOLOGIA MEDIASTINAL• En lo que respecta a la patología mediastinal, cual es la división anatómica?1) Mediastino ANTERIOR: Compuesto por tráquea, ganglios linfáticos, arco aórtico y sus grandes vasos, venas

inominadas y TIMO2) Mediastino MEDIO: CORAZON, pericardio, ganglios linfáticos, hilios pulmonares, vena cava superior, nervio

frénico y vago.3) Mediastino POSTERIOR: ESOFAGO, aorta descendente, vena ácigos, conducto torácico, nervios vagos y

cadena simpática

• Cuales son las lesiones que ocupan el MEDIASTINO ANTERIOR y sus principales características?1) TIMOMA: Mas común en ADULTOS que en niños. Los síntomas mas comunes son dolor torácico, tos, disnea.

La IMAGEN RADIOGRAFICA TÍPICA ES UNA MASA LOBULADA, bien definida y la TAC muestra una masa de tejido blando, encapsulada, bien definida y en ocasiones con necrosis o hemorragia. Se desarrollan síndromes PARATIMICOS como la miastenia grave, hipogammaglobulinemia y aplasia pura de glóbulos rojos. El síndrome de Good es una inmunodeficiencia por déficit de anticuerpos asociada a timoma, generalmente benigno.

2) TUMORES DE CELULAS GERMINALES: Como teratomas, seminomas y tumores no seminomatosos.3) TERATOMAS: Mezcla de tejidos provenientes de las 3 capas germinativas con potencial de transformación

maligna.4) SEMINOMAS: Los síntomas mas frecuentes son disnea, dolor retroesternal, tos, fiebre, GINECOMASTIA,

perdida de peso y síndrome de vena cava superior por compresión tumoral. 5) NO SEMINOMATOSO: Se asocian a KLINEFELTER. Tiene elevación de hormonas como la ALFAFETOPROTEINA y

gonadotropina corionica humana.6) BOCIO MEDIASTINAL: Suelen ser EUTIROIDEOS. Con disnea, estridor y disfagia.7) LINFOMA PRIMARIO: Linfoma Hodking siendo el no Hodking el menos común. Con sintomatología de FIEBRE,

SUDORACION NOCTURNA, perdida de peso, asociada a síntomas por compresión de estructuras vecinas.8) LINFOMA NO HODKING: Sus subtipos mas comunes son el linfoma linfoblastico y linfoma células B de células

grandes.

Page 500: ENARM  COMPENDIO

PATOLOGIA MEDIASTINAL• Cuales son las masas del MEDIASTINO MEDIO mas frecuentes y sus

principales características:1) QUISTES MEDIASTINALES: Los principales síntomas son tos, disnea,

fiebre, expectoración purulenta y dolor torácico.2) LINFANGIOMAS: Anormalidades de los vasos linfáticos.

• Cuales son las masas del MEDIASTINO POSTERIOR mas frecuentes y sus principales características:

1) TUMORES NEUROGENICOS: Se encuentra los tumores benignos y malignos de la vaina nerviosa

2) TUMORES DE LA VAINA NERVIOSA: Son tumores de crecimiento lento y en su mayoría sintomáticos siendo los Schwannomas

3) TUMORES MALIGNOS DE LA VAINA NERVIOSA: Se asocia a neurofibromatosis

Page 501: ENARM  COMPENDIO

MEDIASTINITIS

• Cual es la etiología de la mediastinitis?• R = Infecciosa

• Cual es el cuadro clínico de la mediastinitis?1) Escalofríos, FIEBRE, mal estado general y dolor torácico.2) Al examen físico puede haber dolor en el esternón,

crepitación y el característico pero inconstante SIGNO DE HAMMAN (sonido crujiente que se escucha en la pared anterior del tórax durante la sístole cardiaca).

Page 502: ENARM  COMPENDIO

ASMA• En que casos se eleva la CAPACIDAD TOTAL DE DIFUSIÓN

PULMONAR?• R = ICC, ASMA y hemorragia pulmonar debido a un incremento del

volumen sanguíneo en capilares pulmonares.

• En que casos se emplea la broncoscopia rígida?• R = Hemorragia masiva, extracción de grandes objetos y biopsia de

tumores traqueales.

• Que parámetros evalúan la obstrucción respiratoria de aire?• R = La reducción del VEF1/CVF < 75%.

• Con que medicamentos se realizan las pruebas de estimulación bronquial?

• R = Histamina, METACOLINA.

Page 503: ENARM  COMPENDIO
Page 504: ENARM  COMPENDIO
Page 505: ENARM  COMPENDIO

EPOC• Como se define la bronquitis crónica?• R = Secreción excesiva de moco bronquial manifestada por tos productiva durante 2-3 meses en

2 años consecutivos.

• Que enzima liberada es la que produce el enfisema pulmonar?• R = PROTEASA de PMN

• Cual es la clasificación HISTOLOGICA del enfisema pulmonar y con que se relacionan cada una mas frecuentemente?

1) CENTROLOBULILLAR: Se caracteriza porque el área afectada está en el lobulillo proximal, en especial por destrucción de los bronquiolos respiratorios y dilatación de los lóbulos superiores sin afectar a los alvéolos distales. Representa el 95% de los casos de enfisema y la principal manifestación en los FUMADORES asociándose comúnmente con una BRONQUITIS CRÓNICA.

2) PANACINAR: Es la forma más comúnmente asociada a una DEFICIENCIA DE ALFA-1 ANTITRIPSINA y se caracteriza por involucrar al extremo ciego de los alvéolos de manera homogénea, mas que a los bronquiolos respiratorios y acompañado de los característicos cambios destructivos. Es más frecuente en la base de los pulmones.

3) PARASEPTAL: La enfisema paraseptal interesa prevalentemente a la parte periférica del lobulillo, vecina a la pleura creando grandes espacios aéreos en la región interlobulillar. Es más frecuente en el ápice pulmonar que en las bases y ocasionalmente se ASOCIA CON NEUMOTÓRAX ESPONTÁNEO.

4) IRREGULAR: Es una enfisema cicatrizante, comprometido de manera irregular al acino asociado a géneros asintomáticos. Se le llama también enfisema paracicatrizal o paraseptal.

Page 506: ENARM  COMPENDIO

EPOC• Como se encuentra la capacidad vital forzada pulmonar en EPOC

grave?• R = Disminuida

• Como se encuentra la CTP en EPOC sin complicaciones?• R = Aumentada

• Como se encuentra el volumen residual en EPOC?• R = Aumentado (VR)

• Como se encuentra la relación VR/CPT en EPOC?• R = Aumentada

Page 507: ENARM  COMPENDIO

EPOC• Cual es la principal característica del soplador ROSADO en EPOC (enfisema

predominante)?• R = DISNEA, edad de presentación a los 50ª, pacientes con tos y esputo mucoide,

pacientes delgados con bajo peso y no hay edema periférico.

• Cual es el principal síntoma del AZUL abotagado EPOC (bronquitis predominante)?• R = TOS CRÓNICA con ESPUTO PURULENTO, edad de presentación 30-40ª, disnea leve y

edema periférico.

• Que muestra la Rx de tórax en el soplador rosado (enfisema)?• R = Hiperaereacion con hemidiafragmas aplanados.

• Que muestra la Rx de tórax en el azul abotagado (bronquitis predominante?• R = Sombras intersticiales.

• Que iniciales controlan el tratamiento de EPOC?• R = GOLD

Page 508: ENARM  COMPENDIO
Page 509: ENARM  COMPENDIO

FIBROSIS QUISTICA• Que es la fibrosis quística?• R = Es una enfermedad multisistemica que se transmite de forma AR y es letal.

• Cual es el tipo de mutación que se encuentra en 60% de los casos en FQ?

• R = Brazo corto de cromosoma 7 posición AF508

• Que dato te sugiere fibrosis quística en el RN?• R =Impactación fecal

• Cual es la fisiopatología de la fibrosis quística?• R = Anomalías en la proteína de canal de cloruro de la membrana celular

• Cual es el resultado de la prueba del sudor en fibrosis quística?1) Concentración de CLORURO mediante iontoforesis > 60 mEq en 2 ocasiones en

la prueba de Gibson.2) TRIPSINA INMUNORREACTIVA en neonatos, la cual se basa en el hecho de que

en etapas tempranas, esta proteína se encuentra anormalmente elevada por la obstrucción de conductos pancreáticos.

Page 510: ENARM  COMPENDIO

FIBROSIS QUISTICA• Que tienen en común los pacientes que tienen FQ?• R = Casi todos los varones tienen AUSENCIA BILATERAL CONGÉNITA DEL CONDUCTO

DEFERENTE CON AZOOSPERMIA EN 95%.

• Que signos y síntomas es común encontrar en FQ?1) Antecedentes de enfermedad pulmonar crónica, pancreatitis recurrentes e

infertilidad. 2) Esteatorrea, dolor abdominal y diarrea son comunes.3) La malabsorción se manifiesta por deposiciones abundantes, fétidas y de

características grasas.

• Cuales son los agentes etiológicos de las bronquiectasias infectadas en FQ?• R = Haemophilus influenzae, Staphylococcus Aureus o Pseudomonas aeruginosa

• Cual es el manejo terapéutico de la FQ?• R = Comprende a la nutrición, prevención de infecciones, inhaloterapia y manejo de

exacerbaciones inflamatorias.

• Que medicamentos en FQ disminuyen las exacerbaciones respiratorios, consumo de antibiótico y aumento de la VEF1?

• R = Desoxirribonucleasa humana recombinante inhalada.

Page 511: ENARM  COMPENDIO

BRONQUIECTASIAS• Que enfermedad es la responsable de mas del 50% de bronquiectasias?• R = FIBROSIS QUÍSTICA y el resto se debe secundario a infecciones

• Cual es la sintomatología de las bronquioectasias?• R = Tos crónica, aumento del esputo purulento diario de cantidad

significativa, hemoptisis y neumonías recurrentes.

• Que datos se observan en una radiografía de tórax y una TAC con bronquiectasias?

1) Radiografía de tórax: Incremento inespecífico de la trama vascular, atelectasias u opacidades irregulares

2) TAC: imagen en “RIELES DE TRANVIA” que refleja el engrosamiento de las paredes y la fibrosis peribronquial.

• Cual es el manejo de las bronquiectasias?1) Agentes mucoliticos como acetilcisteina, salina hipertónica2) Antimicrobianos en prevención y para tratamiento

Page 512: ENARM  COMPENDIO

TUBERCULOSIS• Efecto adverso más común de la ISONIACIDA?• R = NEUROPATÍA PERIFÉRICA

• Efecto adverso más común de la RIFAMPICINA?• R = Hemorrágicos, HEPATITIS.

• Efecto adverso del ETAMBUTOL?• R = NEURITIS ÓPTICA

• Efecto adverso de ESTREPTOMICINA?• R = DAÑO AL VIII PAR CRANEAL, nefrotoxicidad.

• Medicamentos para Tb en EMBARAZADA?• R = Isoniacida, Rifampicina y Etambutol por 4-8 semanas o 9 meses.

• En una paciente con VIH + en el que usas isoniacida se debe usar que medicamento para evitar sus reacciones adversas mas comunes?

• R = VITAMINA B6

Page 513: ENARM  COMPENDIO

TUBERCULOSIS• Por que esta contraindicada la estreptomicina en el embarazo?• R = Puede causar sordera congénita.

• Están contraindicados los antituberculosos durante la lactancia?• R = No

• Antes de usar el ETAMBUTOL que pruebas debes realizar?• R = AGUDEZA VISUAL, valorando colores rojo y verde.

• Que es necesario medir en suero antes del uso de PIRAZINAMIDA?• R = ACIDO ÚRICO

• Que prueba es necesesaria realizar antes del uso de ESTREPTOMICINA?• R = AUDIOMETRICA.

• Que agente etiológico es el mas común en enfermedad pulmonar por MNT?• R = M. Avium y M. Kanassi

• Que fármacos utilizas en MNT mas antituberculosos?• R = Claritromicina o azitromicina.

Page 514: ENARM  COMPENDIO

CANCER PULMONAR• Cual es la neoplasia pulmonar benigna mas frecuente?• R = Hamartroma

• Cuales son las neoplasias pulmonares malignas en cáncer pulmonar?1) Cáncer broncogénico de células pequeñas (CPCP): representa el 20% de los canceres

pulmonares2) Cáncer broncogénico de células no pequeñas (CPCnP): representa el 80% de los canceres

pulmonares y abarca los subtipos de ADENOCARCINOMA 60%, escamoso o epidermoide 30% y el carcinoma indiferenciado de células grandes < 10%

• Que es el tumor de Pancoast en el cáncer pulmonar?• R = Los carcinomas que se localizan en el surco pulmonar superior. Tiene una evolución lenta,

tendencia a infiltrar los linfáticos subpleurales y por contigüidad las raíces nerviosas del 8vo nervio cervical y del 2do y 3er dorsales.

• Cuales son las manifestaciones clínicas del tumor de Pancoast?

1) DOLOR LOCALIZADO EN EL HOMBRO DE CARACTERÍSTICAS RADICULARES, IRRADIADO A LO LARGO DEL BRAZO Y PARESTESIAS

2) SÍNDROME DE HORNER con debilidad y atrofia de los músculos extrínsecos

• Cual es la incidencia de ca de células escamosas en cáncer pulmonar primario?• R = 25-35%, se presenta con hemoptisis y se puede diagnosticar mediante citología de esputo.

Page 515: ENARM  COMPENDIO

CANCER PULMONAR• Cual es la estirpe etiológica mas común de ca pulmonar?• R = Adenocarcinoma 60 %

• Cual es la presentación típica del adenocarcinoma pulmonar?• R = Masas o nódulos periféricos

• Cual es el pronóstico del CPCP?• R = Se propaga por VÍA HEMATOGENA muy rápido con SOBREVIDA MUY CORTA, de aproximadamente 2m

después del diagnostico sin tratamiento.

• Cual es el pronostico de CPCnP?• R = Se propaga mas lentamente con BUENA RESPUESTA AL TRATAMIENTO

• Que porcentaje de pacientes con CPCP desarrolla secreción inadecuada de ADH?• R = 15% ADH

• Que porcentaje de pacientes con carcinoma de CPCnP de CÉLULAS ESCAMOSAS desarrolla hipercalcemia?

• R = 10% HIPERCALCEMIA

• Cuales son los fármacos antineoplásicos mas novedosos?• R = Pacitaxel, gemcitabina, vinorelbina.

Page 516: ENARM  COMPENDIO

CARCINOIDE PULMONAR

• Cuales son los síntomas mas comunes de los tumores carcinoides bronquiales?

• R = Hemoptisis, tos, sibilancias y neumonía RECURRENTE

• Que caracteriza al síndrome carcinoide?• R = Rubores, diarrea, sibilancia e hipotensión.

Page 517: ENARM  COMPENDIO

FIBROSIS PULMONAR• Que enfermedad pulmonar te da el patrón radiográfico llamado, en panal de abejas?• R = Fibrosis pulmonar y Enfermedad intersticial pulmonar o enfermedad parenquimatosa pulmonar difusa

• Cual es el nombre actual que recibe la fibrosis pulmonar idiopática?• R = Neumonía intersticial fibrosante idiopática

• Cual es el cuadro clínico y datos de espirometria de la fibrosis pulmonar idiopática?1) Tos seca2) Disnea que se incrementa con ejercicio3) Capacidad vital forzada (CVF) disminuida4) Relación CVF/ VEF 1 dentro de los LIMITES NORMALES O AUMENTADA

• Cuales son los criterios para fibrosis pulmonar idiopática?1) Mayores: Exclusión de otras causas conocidas de enfermedad pulmonar intersticial, como toxicidad por

fármacos, exposición ambiental y enfermedades del tejido conectivo. Pruebas de función respiratoria que sugieran restricción. Opacidades reticulares bibasales como mínimo VIDRIO DESPULIDO en la TAC. Biopsia transbronquial o lavado broncoalveolar no compatible con otro diagnostico.

2) Menores: Edad > 50ª. Disnea de esfuerzo. Mas de 3 meses con la enfermedad. Estertores tipo velcro en región subescapular bilateral.

• Cual es el manejo de la fibrosis pulmonar idiopática?• R = Prednisona y azatriopina o ciclofosfamida.

• Cual es el pronóstico de la fibrosis pulmonar idiopática?• R = Malo con promedio de sobrevida de 1ª.

Page 518: ENARM  COMPENDIO

NEUMOCONIOSIS• Que es la neumoconiosis?• R = Enfermedad pulmonar provocada por la acumulación de POLVOS MINERALES secundaria a la

inhalación crónica de los mismo

• Cuales son las patologías agrupadas en la neumoconiosis?1) ASBESTOSIS por inhalación de ASBESTO2) SILICOSIS por la inhalación de SÍLICE en su mayoría en forma de cuarzo3) SIDEROSIS por la acumulación pulmonar de HIERRO4) ANTRACOSIS por la exposición secundaria a CARBON

• Que caracteriza al mesotelioma, el cual es una complicación de la asbestosis?• R = Son tumores primarios originados a partir del recubrimiento superficial de la pleura (80%), del

peritoneo (20%) y es mas común en hombres 3:1.

• Que características tiene el liquido pleural en el mesotelioma pulmonar?• R = Exudativo y hemorrágico

• Que datos radiográficos denota la silicosis?• R = Pequeñas opacidades redondeadas en todo el pulmón, calcificación de la periferia de los ganglios

linfáticos hiliares “CALCIFICACIÓN EN CASCARA DE HUEVO”.

• Que caracteriza al síndrome de Caplan?• R = Se caracteriza por NÓDULOS NECROBIOTICOS en periferia de pulmón en los TRABAJADORES DE

CARBÓN con ARTRITIS REUMATOIDE

Page 519: ENARM  COMPENDIO
Page 520: ENARM  COMPENDIO

TROMBOEMBOLIA PULMONAR• Cual es la causa hereditaria mas común de hipercoagubilidad?• R = Resistencia a proteína C activada o FACTOR V DE LEIDEN

• Cual es la triada de Virchow identificada como factor de riesgo para desarrollo de trombosis venosa?

1) Disminución del flujo sanguíneo o estasis venosa2) Lesión o traumatismo vascular3) Estado hipercoagulable o trombofilico

• Datos que sugieren TEP?1) DISNEA SÚBITA SOSTENIDA2) TAQUICARDIA SOSTENIDA3) Hipotensiona rterial sistólica < 100 mm Hg4) SINCOPE5) DOLOR PRECORDIAL ANTERIOR QUE SIMULA IAM6) Colapso cardiorespiratorio

• 98. Cuales son las anormalidades EKG en pacientes con TEP?1) Taquicardia sinusal2) Patron S1, Q3 – T3

Page 521: ENARM  COMPENDIO

TROMBOEMBOLIA PULMONAR• Que datos de laboratorio te sugieren TEP?1) Valores plasmáticos de DIMERO D, el cual es un producto de la degradación de fibrina,

mismo que se aumenta en caso de trombos 300-500 mg/ul con ELISA.2) Hipoxemia3) ALCALOSIS RESPIRATORIA

• Cual es el signo de Westemark en TEP?• R = ARTERIA PULMONAR CENTRAL PROMINENTE CON OLIGOHEMIA LOCAL en la

radiografía de tórax. Área de vascularización y perfusión disminuida como consecuencia de una embolia pulmonar, acompañada de un ensanchamiento de la arteria pulmonar central del lado afectado. Se observa en el 2% de los casos.

• Que método sustituyo el diagnostico por imagen de TEP con gamagrama pulmonar?1) ANGIOGRAFÍA PULMONAR selectiva que se considera actualmente como ESTANDAR DE

ORO2) TAC helicoidal

• Cual es el estándar de oro en el tratamiento de la TEP?1) HEPARINA2) El manejo con acenocumarinicos (acenocumarina y warfarina) constituye el tratamiento

a largo plazo por vía oral

Page 522: ENARM  COMPENDIO

TROMBOEMBOLIA PULMONAR• Como actúa la heparina en TEP?• R = Se fija y acelera la capacidad de la antitrombina 3 para inactivar a la trombina, factores 9 y 10.

• Como actúan los acenocumarinicos en TEP?• R = EL mecanismo de acción depende de la inhibición de la carboxilacion de los factores de coagulación 2,7,

9 y 10 (vitamina K dependientes)

• Cual es la meta del INR en TEP?• R = Que se encuentre entre 2-3

• Que te indica un INR <2?• R = Aumenta el RIESGO DE TROMBOSIS

• Que te indica un INR >4?• R = Aumenta el riesgo de HEMORRAGIA.

• Como actúa la estreptoquinasa, urocinasa y el activador tisular de plasminogeno?• R = Aumentan los valores de plasmina, lisando directamente los trombos intravasculares

• Que medida adicional puede utilizarse en pacientes que esta contraindicado el uso de anticoagulantes?• R = Bloqueo de la vena cava inferior

• En pacientes quirúrgicos de riesgo tromboembolico que medida de prevención se utiliza?• R = Heparina de bajo peso molecular

Page 523: ENARM  COMPENDIO
Page 524: ENARM  COMPENDIO

HIPERTENSION PULMONAR• Cual es la definición hemodinámica de la hipertensión pulmonar (HP)?• R = PRESIÓN ARTERIAL pulmonar media MAYOR A 25 MM HG EN REPOSO O DE 30 MM HG DURANTE EL

EJERCICIO.

• Cual es la etiología de la HP?1) in situ Vasoconstricción sostenida2) Proliferación vascular pulmonar3) Trombosis 4) Disnea en reposo o de esfuerzo. 5) Disnea en reposo con dolor retroesternal semejante a angina de pecho. 6) Policitemia

• Cuales son los síntomas que puede incluir la hipertensión pulmonar, además de los datos de laboratorio mas frecuentes?

• Que refleja hipoxemia crónica.

• Cuales son los datos EKG de HP?• R = Desviación del eje a la derecha, hipertrofia ventricular derecha y crecimiento de aurícula derecha.

• Que tratamiento esta indicado tanto en la HP primaria como secundaria?• R = Inhibidor de la fosfodiesterasa 5 (sindenafil), Anticuagulación permanente, venoclisis prolongada

continua de prostaciclina +++

• Como se maneja el Cor pulmonale como complicación de HP?• R = Tratamiento de enfermedad de fondo, diuréticos, disminución de Na y O2

Page 525: ENARM  COMPENDIO

GRANULOMATOSIS DE WEGENER• En que consiste la granulomatosis de Wegener (vasculitis pulmonar)?• R = Es una enfermedad pulmonar ideopatica que se manifiesta por una combinación de GLOMERULONEFRITIS,

VASCULITIS GRANULOMATOSA NECROSANTE de vías respiratorias superiores e inferiores y vasculitis de pequeños vasos.

• Cuales son los síntomas mas frecuentes en granulomatosis de Wegener?• R = SINUSITIS CRÓNICA, artralgias, exantema y disminución de peso

• Cual es el signo pulmonar mas común en granulomatosis de Wegener?• R = Infiltrado pulmonar nodular con cavitaciones

• Que prueba de lab te confirma granulomatosis de Wegener?• R = ANCA 90% con patrón de tinción citoplasmática fluorecente cANCA

• Como diagnosticas granulomatosis de Wegener sin laboratorio?• R = Con biopsia pulmonar, tejido sinusal o renal demostrando VASCULITIS GRANULOMATOSA NECROSANTE.

• Cual es el tratamiento de la granulomatosis de Wegener o vasculitis pulmonar?• R = Corticosteroides y ciclofosfamida

• Cual es el corticosteroide de elección en granulomatosis de Wegener?• R = PREDNISONA ORAL.

• Cual es antibiótico de elección en granulomatosis de Wegener?• R = TMP/SMZ

Page 526: ENARM  COMPENDIO

CHURG- STRAUSS• Que es el sindrome de Churg- Strauss?• El síndrome de Churg Strauss, también llamado angeítis y granulomatosis alérgica, es un

síndrome multisistémico CARACTERIZADO POR RINITIS ALÉRGICA, ASMA Y EOSINOFILIA importante en el extendido periférico. El órgano más comúnmente comprometido es el pulmón, seguido de la piel.

• Sin embargo, el síndrome de Churg Strauss puede afectar cualquier órgano sistema, incluyendo el sistema cardiovascular, el gastrointestinal, el renal y el sistema nervioso central.

• Cuales son manifestaciones clínicas del síndrome de Churg-Strauss? Se manifiesta en tres fases:1) Asma y rinitis alérgica.2) Eosinofilia periférica e infiltración a órganos.3) Vasculitis con afectación sistémica:• a. Pulmonar: asma grave, tos, disnea, sinusitis.• b. Neurológica: mononeuritis, polineuropatía.• c. Cutánea: púrpura, urticaria.• d. Cardiovascular: pericarditis, IAM.• e. Gastrointestinal: gastroenteritis eosinofílica, dolor abdominal, diarrea, hemorragia.

• Que laboratorio se mantiene constantemente presente en el sx de Churg-Strauss?• R = Eosinofilia

Page 527: ENARM  COMPENDIO

CHURG- STRAUSS• Para su clasificación el American College of Rheumatology (1990) publicó

los siguientes criterios diagnósticos contando con al menos cuatro de los siguientes seis:

1) Asma: historia de dificultad respiratoria o sibilancias espiratorias difusas.2) Eosinofilia: eosinofilia >10% en conteo diferencial de glóbulos blancos.3) Mono o polineuropatía: desarrollo de mononeuropatía, mononeuropatía

múltiple, o polineuropatía (distribución en guante o en bota) atribuible a vasculitis sistémica.

4) Infiltrados pulmonares migratorios: infiltrados pulmonares migratorios o transitorios (no incluyendo infiltrados fijos). Atribuibles a vasculitis.

5) Anormalidades de los senos paranasales: historia de dolor agudo o crónico de los senos paranasales u opacificación radiográfica de los senos paranasales.

6) Eosinófilos extravasculares: biopsia incluyendo arterias, arteriolas o vénulas mostrando acúmulos de eosinófilos en áreas extravasculares.

Page 528: ENARM  COMPENDIO

CHURG- STRAUSS

• Cual es el manejo del sindrome de Churg Strauss?A. La combinación entre altas dosis de

corticoesteroides y ciclofosfamida son significativas en el tratamiento de esta patología con una baja tasa de resistencia;

B. Los corticoesteroides pueden ser usados solos en casos del síndrome de Churg Strauss, en cambio la ciclofosfamida se utiliza solo en casos de pacientes con ANCA positivo.

Page 529: ENARM  COMPENDIO

SÍNDROME DE GOOD PASTEUR• Menciona cuales son las causas de hemorragia alveolar difusa?1) Enfermedad de ANTICUERPO ANTIMEMBRANA BASAL (sx de good pasteur),2) Vasculitis y enfermedad de la colagena vascular (lupus eritematoso sistémico, granulomatosis de

Wegener).

• Cuales son las causas de hemorragia alveolar difusa de causa no inmunitaria?• R = Coagulopatia, estenosis mitral, fármacos como la penicilamina y hemosiderosis pulmonar

idiopática.

• Que es el síndrome de Good pasteur?• R = HEMORRAGIA ALVEOLAR recurrente idiopática y GLOMERULONEFRITIS rápidamente progresiva

que se manifiesta en varones de 30-40ª.

• Como diagnosticas el síndrome de Good pasteur?

• R = En la biopsia se observan depósitos lineales de IgG tanto en glomérulos como en alveolos por medio de inmunofluorecencia y PRESENCIA DE AC CONTRA MEMBRANA BASAL glomerular en suero.

• Que tratamiento a dado resultado en el síndrome de Goodpasteur?• R = PREDNISONA, CLICLOFOSFAMIDA Y PLASMAFERESIS

Page 530: ENARM  COMPENDIO

SÍNDROME DE INSUFICIENCIA RESPIRATORIA AGUDA

• Como se define el síndrome de insuficiencia respiratorio (SIRA antes SIRPA)?• R = Pacientes con disnea, hipoxemia refractaria a tratamiento, disminución de la

complianza pulmonar e infiltrados pulmonares difusos en la radiografía de tórax.

• Cual es la etiología del SIRA?1) Causas de lesión pulmonar de manera indirecta como sobredosis de

fármacos2) Causas de lesión pulmonar de manera directa como la broncoaspiracion o

politraumatismo

• Cual es el cuadro clínico del SIRA?1) Deterioro progresivo de la función respiratoria2) Disnea en reposo3) Hipoxemia grave Pa O2 < 50 mm Hg refractaria a la oxigenoterapia4) ALCALOSIS RESPIRATORIA5) Taquipnea6) CIANOSIS

Page 531: ENARM  COMPENDIO

SÍNDROME DE INSUFICIENCIA RESPIRATORIA AGUDA

• Cuales son los criterios diagnósticos de SIRA?R = Se diagnostica SIRA si cumple con 2 de los siguientes criterios1) Inicio agudo o súbito de la sintomatología2) Opacidades pulmonares alveolares o en parches difusas bilaterales3) Hipoxemia PaO2 < 60 mm Hg y relación PaO2 / FiO2 < 2004) Presión capilar pulmonar < 18 mm Hg o ausencia de signos de hipertensión auricular

izquierda en caso de no poder medir la presión en cuna pulmonar.

• Cual es el manejo del SIRA?1) Ventilación mecánica invasiva de elección2) Los fármacos no han demostrado beneficio en SIRA

• Cual es el pronostico del SIRA?• R = Mortalidad entre 40-60%

• Que maniobras han demostrado disminuir la mortalidad den SIRA?1) El volumen corriente bajo (<6 ml/kg de peso ideal)2) Mantenimiento de una presión meseta o plateau < 30 cm H2O

Page 532: ENARM  COMPENDIO

MICOSIS BRONCOPULMONAR ALERGICA

• Que es la micosis broncopulmonar alérgica (MBA)?• R = Trastorno de hipersensibilidad por alergia a los antígenos micóticos.

Aspergilosis broncopulmonar alérgica.

• Agente causal de MBA?• R = Especies de aspergilus.

• Como dx MBA?• R = Tiene que reunir 6-7 criterios. 1) antecedente de asma 2) eosinofilia

periférica 3)reactividad cutánea a aspergilus 4)Ac vs aspergilus 5) aumento de IgE 6)bronquiectasias 7)infiltrados pulmonares.

• Cual es el tratamiento de elección en MBA?• R = Prednisona

Page 533: ENARM  COMPENDIO

BIBLIOGRAFIA

• EXARMED• PAPADAKIS• CTO• HARRISON